compilacaoModulo1Nivel2

117
DRAFT PECI 2013 – Nível 2 – Módulo 1 Robério Bacelar, Fábio Brochero, Sávio Ribas e Paulo Rodrigues 1 de Fevereiro de 2013

Transcript of compilacaoModulo1Nivel2

Page 1: compilacaoModulo1Nivel2

DRAFTPECI 2013 – Nível 2 – Módulo 1

Robério Bacelar, Fábio Brochero, Sávio Ribas e Paulo Rodrigues

1 de Fevereiro de 2013

Page 2: compilacaoModulo1Nivel2

DRAFT

2

Page 3: compilacaoModulo1Nivel2

DRAFT

Conteúdo

1 Informações sobre o Programa 7

1.1 Apresentação . . . . . . . . . . . . . . . . . . . . . . . . . . . . . . . . . . . . . . . . . . . . . . . . . . . . . . 7

1.2 Utilizando o Fórum . . . . . . . . . . . . . . . . . . . . . . . . . . . . . . . . . . . . . . . . . . . . . . . . . . 7

1.3 Avaliações e Simulados . . . . . . . . . . . . . . . . . . . . . . . . . . . . . . . . . . . . . . . . . . . . . . . . 7

1.4 Como participar da OBM? . . . . . . . . . . . . . . . . . . . . . . . . . . . . . . . . . . . . . . . . . . . . . . . 8

Álgebra 9

2 Produtos Notáveis e Fatoração 10

2.1 Produtos Notáveis . . . . . . . . . . . . . . . . . . . . . . . . . . . . . . . . . . . . . . . . . . . . . . . . . . . 10

2.2 Triângulo de Pascal . . . . . . . . . . . . . . . . . . . . . . . . . . . . . . . . . . . . . . . . . . . . . . . . . . 10

2.2.1 Exemplos . . . . . . . . . . . . . . . . . . . . . . . . . . . . . . . . . . . . . . . . . . . . . . . . . . . . 11

2.3 Fatoração . . . . . . . . . . . . . . . . . . . . . . . . . . . . . . . . . . . . . . . . . . . . . . . . . . . . . . . . 11

2.4 Potênciação em R . . . . . . . . . . . . . . . . . . . . . . . . . . . . . . . . . . . . . . . . . . . . . . . . . . . 12

2.4.1 Potência com Expoente Natural . . . . . . . . . . . . . . . . . . . . . . . . . . . . . . . . . . . . . . . . 12

2.4.2 Propriedades . . . . . . . . . . . . . . . . . . . . . . . . . . . . . . . . . . . . . . . . . . . . . . . . . . 12

2.5 Radiciação em R . . . . . . . . . . . . . . . . . . . . . . . . . . . . . . . . . . . . . . . . . . . . . . . . . . . . 13

2.5.1 Propriedades . . . . . . . . . . . . . . . . . . . . . . . . . . . . . . . . . . . . . . . . . . . . . . . . . . 13

2.6 Potência com Expoente Racional . . . . . . . . . . . . . . . . . . . . . . . . . . . . . . . . . . . . . . . . . . . 13

2.7 Radical Duplo . . . . . . . . . . . . . . . . . . . . . . . . . . . . . . . . . . . . . . . . . . . . . . . . . . . . . . 13

2.8 Problemas Propostos . . . . . . . . . . . . . . . . . . . . . . . . . . . . . . . . . . . . . . . . . . . . . . . . . 13

3 Equações do Segundo Grau 18

3.1 Equação do Segundo Grau . . . . . . . . . . . . . . . . . . . . . . . . . . . . . . . . . . . . . . . . . . . . . . 18

3.1.1 Forma Canônica do Trinômio de Segundo Grau . . . . . . . . . . . . . . . . . . . . . . . . . . . . . . 18

3.1.2 Fórmula de Báscara . . . . . . . . . . . . . . . . . . . . . . . . . . . . . . . . . . . . . . . . . . . . . . 18

3.1.3 Soma das raízes da Equação do Segundo Grau . . . . . . . . . . . . . . . . . . . . . . . . . . . . . . . 19

3.1.4 Produto das Raízes da Equação de Segundo Grau . . . . . . . . . . . . . . . . . . . . . . . . . . . . . 19

3.2 Problemas Resolvidos . . . . . . . . . . . . . . . . . . . . . . . . . . . . . . . . . . . . . . . . . . . . . . . . . 19

3.3 Forma Fatorada do Trinômio de Segundo Grau . . . . . . . . . . . . . . . . . . . . . . . . . . . . . . . . . . 20

3.3.1 Exercícios Propostos . . . . . . . . . . . . . . . . . . . . . . . . . . . . . . . . . . . . . . . . . . . . . . 20

Combinatória 23

4 Paridade 24

4.1 Problemas Propostos . . . . . . . . . . . . . . . . . . . . . . . . . . . . . . . . . . . . . . . . . . . . . . . . . 31

3

Page 4: compilacaoModulo1Nivel2

DRAFT

4 CONTEÚDO

5 Princípio da Casa dos Pombos 33

5.1 Exemplos Introdutórios . . . . . . . . . . . . . . . . . . . . . . . . . . . . . . . . . . . . . . . . . . . . . . . . 33

5.2 Pombos Geométricos . . . . . . . . . . . . . . . . . . . . . . . . . . . . . . . . . . . . . . . . . . . . . . . . . 34

5.3 Problemas Propostos . . . . . . . . . . . . . . . . . . . . . . . . . . . . . . . . . . . . . . . . . . . . . . . . . 35

5.4 Ramsey . . . . . . . . . . . . . . . . . . . . . . . . . . . . . . . . . . . . . . . . . . . . . . . . . . . . . . . . . 35

5.4.1 Problemas Propostos . . . . . . . . . . . . . . . . . . . . . . . . . . . . . . . . . . . . . . . . . . . . . . 36

5.5 Pombos e Divisibilidade . . . . . . . . . . . . . . . . . . . . . . . . . . . . . . . . . . . . . . . . . . . . . . . . 37

5.5.1 Problemas Propostos . . . . . . . . . . . . . . . . . . . . . . . . . . . . . . . . . . . . . . . . . . . . . . 40

5.6 Combinatória Aditiva . . . . . . . . . . . . . . . . . . . . . . . . . . . . . . . . . . . . . . . . . . . . . . . . . 40

5.7 Problemas Avançados . . . . . . . . . . . . . . . . . . . . . . . . . . . . . . . . . . . . . . . . . . . . . . . . . 41

5.8 Problemas Complementares . . . . . . . . . . . . . . . . . . . . . . . . . . . . . . . . . . . . . . . . . . . . . 42

5.9 Problemas Propostos . . . . . . . . . . . . . . . . . . . . . . . . . . . . . . . . . . . . . . . . . . . . . . . . . 43

6 Grafos 45

6.1 Viajando em Pecilândia . . . . . . . . . . . . . . . . . . . . . . . . . . . . . . . . . . . . . . . . . . . . . . . . 45

6.2 Entre Amigos . . . . . . . . . . . . . . . . . . . . . . . . . . . . . . . . . . . . . . . . . . . . . . . . . . . . . . 46

6.3 Triângulos em Grafos . . . . . . . . . . . . . . . . . . . . . . . . . . . . . . . . . . . . . . . . . . . . . . . . . 49

6.4 Problemas Propostos . . . . . . . . . . . . . . . . . . . . . . . . . . . . . . . . . . . . . . . . . . . . . . . . . 51

Teoria dos Números 53

7 Ferramentas Básicas 54

7.1 Aquecendo os motores . . . . . . . . . . . . . . . . . . . . . . . . . . . . . . . . . . . . . . . . . . . . . . . . 54

7.1.1 Critérios de divisibilidade . . . . . . . . . . . . . . . . . . . . . . . . . . . . . . . . . . . . . . . . . . . 54

7.1.2 Exercícios para esquentar . . . . . . . . . . . . . . . . . . . . . . . . . . . . . . . . . . . . . . . . . . . 54

7.2 Ferramentas preliminares . . . . . . . . . . . . . . . . . . . . . . . . . . . . . . . . . . . . . . . . . . . . . . . 55

7.2.1 Princípio da Indução Finita - 1ª forma . . . . . . . . . . . . . . . . . . . . . . . . . . . . . . . . . . . . 55

7.2.2 Princípio da Indução Finita - 2ª forma . . . . . . . . . . . . . . . . . . . . . . . . . . . . . . . . . . . . 55

7.2.3 Princípio da Boa Ordenação . . . . . . . . . . . . . . . . . . . . . . . . . . . . . . . . . . . . . . . . . . 56

7.2.4 Princípio da Casa dos Pombos . . . . . . . . . . . . . . . . . . . . . . . . . . . . . . . . . . . . . . . . 56

7.2.5 Exercícios de fixação . . . . . . . . . . . . . . . . . . . . . . . . . . . . . . . . . . . . . . . . . . . . . . 56

7.3 Divisibilidade . . . . . . . . . . . . . . . . . . . . . . . . . . . . . . . . . . . . . . . . . . . . . . . . . . . . . . 57

7.3.1 Múltiplos e divisores . . . . . . . . . . . . . . . . . . . . . . . . . . . . . . . . . . . . . . . . . . . . . . 57

7.3.2 O algoritmo de Euclides . . . . . . . . . . . . . . . . . . . . . . . . . . . . . . . . . . . . . . . . . . . . 57

7.3.3 O Teorema Fundamental da Aritmética . . . . . . . . . . . . . . . . . . . . . . . . . . . . . . . . . . . 58

7.4 Problemas Propostos . . . . . . . . . . . . . . . . . . . . . . . . . . . . . . . . . . . . . . . . . . . . . . . . . 60

Geometria 62

8 Problemas de Aquecimento 63

9 Bases Médias, Medianas e Tesouros 65

9.1 A Base Média . . . . . . . . . . . . . . . . . . . . . . . . . . . . . . . . . . . . . . . . . . . . . . . . . . . . . . 65

9.2 O Primeiro Tesouro . . . . . . . . . . . . . . . . . . . . . . . . . . . . . . . . . . . . . . . . . . . . . . . . . . 66

9.3 Outro Tesouro Perdido . . . . . . . . . . . . . . . . . . . . . . . . . . . . . . . . . . . . . . . . . . . . . . . . 67

9.4 Problemas Propostos . . . . . . . . . . . . . . . . . . . . . . . . . . . . . . . . . . . . . . . . . . . . . . . . . 68

9.5 Medianas . . . . . . . . . . . . . . . . . . . . . . . . . . . . . . . . . . . . . . . . . . . . . . . . . . . . . . . . 69

9.6 Mediana em um triângulo retângulo . . . . . . . . . . . . . . . . . . . . . . . . . . . . . . . . . . . . . . . . . 70

9.7 Problemas Propostos . . . . . . . . . . . . . . . . . . . . . . . . . . . . . . . . . . . . . . . . . . . . . . . . . 72

Page 5: compilacaoModulo1Nivel2

DRAFT

CONTEÚDO 5

10 Problemas Extras 73

Sessões de Problemas 74

11 Sessão 1 75

12 Sessão 2 76

13 Sessão 3 77

14 Sessão 4 78

15 Sessão 5 79

Avaliações e Simulados 80

16 Prova de Seleção 2013 81

17 Avaliação 1 83

17.1 Álgebra . . . . . . . . . . . . . . . . . . . . . . . . . . . . . . . . . . . . . . . . . . . . . . . . . . . . . . . . . 83

17.2 Combinatória . . . . . . . . . . . . . . . . . . . . . . . . . . . . . . . . . . . . . . . . . . . . . . . . . . . . . . 83

17.3 Geometria . . . . . . . . . . . . . . . . . . . . . . . . . . . . . . . . . . . . . . . . . . . . . . . . . . . . . . . . 83

17.4 Números . . . . . . . . . . . . . . . . . . . . . . . . . . . . . . . . . . . . . . . . . . . . . . . . . . . . . . . . . 84

18 Simulado 1 85

19 Simulado 2 86

20 Gabarito da Prova de Seleção 87

21 Gabarito da Avaliação 95

21.1 Álgebra . . . . . . . . . . . . . . . . . . . . . . . . . . . . . . . . . . . . . . . . . . . . . . . . . . . . . . . . . 95

21.2 Combinatória . . . . . . . . . . . . . . . . . . . . . . . . . . . . . . . . . . . . . . . . . . . . . . . . . . . . . . 96

21.3 Geometria . . . . . . . . . . . . . . . . . . . . . . . . . . . . . . . . . . . . . . . . . . . . . . . . . . . . . . . . 96

21.4 Números . . . . . . . . . . . . . . . . . . . . . . . . . . . . . . . . . . . . . . . . . . . . . . . . . . . . . . . . . 98

22 Gabarito do Simulado 1 100

23 Gabarito do Simulado 2 103

A Utilizando TEX no Fórum 105

A.1 O que é TEX? . . . . . . . . . . . . . . . . . . . . . . . . . . . . . . . . . . . . . . . . . . . . . . . . . . . . . . 105

A.1.1 Knuth inventou o TEX... . . . . . . . . . . . . . . . . . . . . . . . . . . . . . . . . . . . . . . . . . . . . 105

A.1.2 ... e Lamport criou o LATEX . . . . . . . . . . . . . . . . . . . . . . . . . . . . . . . . . . . . . . . . . . . 105

A.2 Escrevendo e desenhando no fórum . . . . . . . . . . . . . . . . . . . . . . . . . . . . . . . . . . . . . . . . . 106

A.2.1 Exemplos Básicos . . . . . . . . . . . . . . . . . . . . . . . . . . . . . . . . . . . . . . . . . . . . . . . . 107

A.2.2 Letras Gregas . . . . . . . . . . . . . . . . . . . . . . . . . . . . . . . . . . . . . . . . . . . . . . . . . . 107

A.2.3 Aritmética . . . . . . . . . . . . . . . . . . . . . . . . . . . . . . . . . . . . . . . . . . . . . . . . . . . . 107

A.2.4 Geometria . . . . . . . . . . . . . . . . . . . . . . . . . . . . . . . . . . . . . . . . . . . . . . . . . . . . 107

A.2.5 Setas . . . . . . . . . . . . . . . . . . . . . . . . . . . . . . . . . . . . . . . . . . . . . . . . . . . . . . . 108

A.2.6 Símbolos Diversos . . . . . . . . . . . . . . . . . . . . . . . . . . . . . . . . . . . . . . . . . . . . . . . 108

A.2.7 Conjuntos . . . . . . . . . . . . . . . . . . . . . . . . . . . . . . . . . . . . . . . . . . . . . . . . . . . . 108

A.2.8 Matrizes e Determinantes . . . . . . . . . . . . . . . . . . . . . . . . . . . . . . . . . . . . . . . . . . . 108

Page 6: compilacaoModulo1Nivel2

DRAFT

6 CONTEÚDO

A.2.9 Somatórios e Produtórios . . . . . . . . . . . . . . . . . . . . . . . . . . . . . . . . . . . . . . . . . . . 109

A.2.10Diversos . . . . . . . . . . . . . . . . . . . . . . . . . . . . . . . . . . . . . . . . . . . . . . . . . . . . . 109

A.2.11Construindo figuras com LATEX . . . . . . . . . . . . . . . . . . . . . . . . . . . . . . . . . . . . . . . . 110

B Competições e Prêmios 113

B.1 Torneio Internacional das Cidades . . . . . . . . . . . . . . . . . . . . . . . . . . . . . . . . . . . . . . . . . . 113

B.2 Asian Pacific Mathematical Olympiad . . . . . . . . . . . . . . . . . . . . . . . . . . . . . . . . . . . . . . . . 113

B.3 Olimpíada de Matemática do Cone Sul . . . . . . . . . . . . . . . . . . . . . . . . . . . . . . . . . . . . . . . 114

B.4 Olimpíada Iberoamericana de Matemática . . . . . . . . . . . . . . . . . . . . . . . . . . . . . . . . . . . . . 114

B.5 Romanian Masters in Mathematics . . . . . . . . . . . . . . . . . . . . . . . . . . . . . . . . . . . . . . . . . . 115

B.6 Olimpíada de Matemática dos Países de Língua Portuguesa . . . . . . . . . . . . . . . . . . . . . . . . . . . 115

B.7 Olimpíada Rioplatense de Matemática . . . . . . . . . . . . . . . . . . . . . . . . . . . . . . . . . . . . . . . . 115

B.8 Olimpíada Internacional de Matemática . . . . . . . . . . . . . . . . . . . . . . . . . . . . . . . . . . . . . . . 116

Page 7: compilacaoModulo1Nivel2

DRAFT

... 1.

Informações sobre o Programa

1.1

Apresentação

Caro aluno, parabéns por ter sido escolhido para participar das atividades virtuais do PECI 2013. O objetivo do PECI

é preparar alunos para competições internacionais de matemática. Desde 2009, quando foi criado, nossos alunos já

obtiveram mais de 35 premiações internacionais, incluindo 8 na Olimpíada Internacional de Matemática (IMO).

Para alunos do nível 2, o primeiro passo para participar de uma competição internacional é ser premiado na

Olimpíada Brasileira de Matemática – OBM. Deste modo, como participante do PECI em 2013, seu objetivo deve ser o

de ser premiado nesta competição que acontecerá ao longo do ano.

Para ajudá-lo nessa preparação você contará com um fórum exclusivo na internet, para discutir os problemas e

conteúdos propostos pela equipe de professores. Você será acompanhado por professores e realizará periodicamente

simulados. Para um bom aproveitamento do programa estimamos um mínimo de 12h semanais de dedicação.

Apresentamos em anexo os conteúdos do Módulo 1, o qual será utilizado durante 8 semanas. São conteúdos de

Álgebra, Combinatória, Geometria e Teoria dos Números. Constam também os enunciados e gabaritos dos simulados

aplicados no encontro presencial de Janeiro em Brasília e as soluções da prova de seleção do PECI 2013.

Atenciosamente,

Paulo Rodrigues

Coordenador do PECI

1.2

Utilizando o Fórum

O fórum é a nosso ambiente de ensino e aprendizagem. Sua principal utilidade é a discussão e a resolução dos proble-

mas propostos no material. Sinta-se à vontade para propor problemas, esclarecer dúvidas dos conteúdos abordados

e tratar de tudo o que auxiliar a sua aprendizagem no PECI.

Para ajudar neste estudo contaremos com os monitores. O trabalho dos monitores será o de fomentar a discussão

dos problemas, encaminhando ideias e corrigindo rumos.

Para acessar o Fórum do PECI você deverá digitar no navegador web o endereço abaixo:

http://peci.obmep.org.br/Após o carregamento do site, visualize os campos Nome de Usuário e Senha. Entre com seu Nome de Usuário

e Senha e clique no botão Entrar. Estes dados são essenciais para acesso ao fórum e são sigilosos. Clique na opção

PECI para acessar o fórum.

1.3

Avaliações e Simulados

Ao longo de todo o ano realizaremos no fórum atividades obrigatórias para todos os alunos. São simulados, avaliações

e maratonas. Estas atividades são postadas no fórum normalmente numa sexta-feira e o aluno deve postar as suas

soluções até a segunda-feira seguinte.

7

Page 8: compilacaoModulo1Nivel2

DRAFT

8 – Informações sobre o Programa – Professor

O calendário abaixo mostra todas as atividades previstas em 2013 para a turma do nível 2. As atividades marcadas

com serão realizadas nos encontros presenciais pelos alunos participantes dos mesmos na data de início e nesta mesma

data serão postadas no fórum para os alunos que participam à distância.

# Início Fim Atividade Objetivo / Assunto

1 15/02 18/02 Simulado OBM 1a Fase

2 08/03 11/03 Maratona Números

3 22/03 25/03 Avaliação 2 Módulo 1

4 10/04 13/04 Maratona Álgebra

5 26/04 29/04 Avaliação 1 Módulo 2

6 10/05 13/05 Simulado OBM 1a Fase

7 24/05 27/05 Maratona Combinatória

8 14/06 17/06 Maratona Geometria

9 04/07 07/07 Simulado OBM 2a Fase

10 08/07 11/07 Simulado OBM 3a Fase

11 02/08 05/08 Avaliação Módulo 3

12 21/08 24/08 Simulado OBM 2a Fase

13 06/09 09/09 Avaliação Módulo 4

14 20/09 23/09 Simulado OBM 3a Fase

15 04/10 07/10 Simulado OBM 3a Fase

1.4

Como participar da OBM?

Para participar da OBM, sua escola deve se inscrever diretamente pelo site www.obm.org.br. As inscrições estarão

abertas de 25 de Março a 30 de Abril de 2013.

Calendário de provas

• Primeira Fase: sábado, 15 de junho de 2013.

• Segunda Fase: sábado, 21 de setembro de 2013.

• Terceira Fase: sábado, 26 de outubro de 2013, (níveis 1, 2 e 3) e domingo, 27 de outubro de 2013, para os níveis 2

e 3 (segundo dia de prova).

Page 9: compilacaoModulo1Nivel2

DRAFT

PARTE I

ÁlgebraProfessor Robério Bacelar

..Assuntos do Módulo: Produtos Notáveis e Fatoração; Equa-

ções do Segundo Grau.

.

Pré-requisitos: Monômios; grau de um monômio; Monômios

semelhantes.

Page 10: compilacaoModulo1Nivel2

DRAFT

..

Álgebra

. 2.

Produtos Notáveis e Fatoração

2.1

Produtos Notáveis

• 1º Caso: Quadrado da Soma e Quadrado da Diferença de Dois Termos

(x± y)2 = x2 ± 2xy+ y2

• 2º Caso: Produto da Soma pela Diferença de Dois Termos

(x+ y)(x− y) = x2 − y2

• 3º Caso: Quadrado da soma de três termos

(x+ y+ z)2 = x2 + y2 + z2 + 2(xy+ yz+ zx)

• 4º Caso: Cubo da Soma e Cubo da Diferença de Dois Termos

(x± y)3 = x3 ± 3x2y+ 3xy2 ± y3

• 5º Caso: Cubo da Soma de Três Termos

(x+ y+ z)3 = x3 + y3 + z3 + 3(x+ y)(y+ z)(z+ x)

• 6º Caso: Produto de Stevin

(x+ a)(x+ b) = x2 + x(a+ b) + ab

(x+ a)(x+ b)(x+ c) = x3 + (a+ b+ c)x2 + (ab+ bc+ ca)x+ abc

• 7º Caso: Identidade de Lagrange

(ax+ by)2 + (ay− bx)2 = (a2 + b2)(x2 + y2)

2.2

Triângulo de Pascal

A disposição de números abaixo, formada somente pelos coeficientes dos termos do desenvolvimento de (x + y)n é

denominada Triângulo de Pascal (1623-1662).

10

Page 11: compilacaoModulo1Nivel2

DRAFT

OBMEP • PECI – Preparação Especial para Competições Internacionais 11

...1

.

.

1 .

.

1

.

.

1 .

.

2

.

+.

.

1

.

.

1 .

.

3

.

+.

.

3

.

+.

.

1

.

.

1 .

.

4

.

+.

.

6

.

+.

.

4

.

+.

.

1

.

.

1 .

.

5

.

+.

.

10

.

+.

.

10

.

+.

.

5

.

+.

.

1

.

.

1 .

.

6

.

+.

.

15

.

+.

.

20

.

+.

.

15

.

+.

.

6

.

+.

.

1

.

.

1 .

.

7

.

+.

.

21

.

+.

.

35

.

+.

.

35

.

+.

.

21

.

+.

.

7

.

+.

.

1

.

.

1 .

.

8

.

+.

.

28

.

+.

.

56

.

+.

.

70

.

+.

.

56

.

+.

.

28

.

+.

.

8

.

+.

.

1

.Linha 0: (x+ y)0 .

Linha 1: (x+ y)1

.

Linha 2: (x+ y)2

.

Linha 3: (x+ y)3

.

Linha 4: (x+ y)4

.

Linha 5: (x+ y)5

.

Linha 6: (x+ y)6

.

Linha 7: (x+ y)7

.

Linha 8: (x+ y)8

Nela, observamos que:

(1) O primeiro elemento é 1;

(2) O número de termos da linha n é n+ 1;

(3) Todas as linhas começam e terminam por 1;

(4) Dois elementos de uma mesma linha, equidistantes dos extremos, são iguais;

(5) Cada elemento de uma linha, a partir do segundo, é a soma do elemento que lhe fica acima com o que está a

esquerda deste último;

(6) A soma dos coeficientes da linha de ordem n é igual a 2n;

(7) No desenvolvimento de (x+ y)n, os expoentes de x decrescem de n até 0, enquanto os de y crescem de 0 até n.

2.2.1

Exemplos

1.

(x+ y)4 = 1x4 + 4x3y+ 6x2y2 + 4xy3 + 1y4

2.

(2a2 + 1)4 = 1(2a2)4 + 4(2a2)31+ 6(2a2)212 + 4(2a2)113 + 1 · 14 = 16a8 + 32a6 + 24a4 + 8a2 + 1

3. No desenvolvimento de (x− y)n, os sinais (+) e (−), alternam-se a partir do 1º termo.

(x− y)5 = 1 · x5 − 5x4y+ 10x3y2 − 10x2y3 + 5xy4 − 1 · y5

2.3

Fatoração

• 1º Caso: Fator Comum em Evidência

ax+ ay = a(x+ y)

Page 12: compilacaoModulo1Nivel2

DRAFT

12 Álgebra– Produtos Notáveis e Fatoração – Professor Robério Bacelar

• 2º Caso: Trinômio Quadrado Perfeito

a2 ± 2ab+ b2 = (a± b)2

• 3º Caso: Soma de dois cubos

a3 + b3 = (a+ b)(a2 − ab+ b2)

• 4º Caso: Diferença de dois cubos

a3 − b3 = (a− b)(a2 + ab+ b2)

• 5º Caso: xn − yn, sendo n um número natural

xn − yn = (x− y)(xn−1 + xn−2y+ xn−3y2 + · · ·+ xyn−2 + yn−1)

Exemplo:

x5 − y5 = (x− y)(x4 + x3y1 + x2y2 + x1y3 + y4)

• 6º Caso: xn + yn, quando n é ímpar

xn + yn = (x+ y)(xn−1 − xn−2y+ xn−3y2 − · · ·− xyn−2 + yn−1)

Exemplo:

x5 + y5 = (x+ y)(x4 − x3y1 + x2y2 − x1y3 + y4)

2.4

Potênciação em R

2.4.1

Potência com Expoente Natural

Se n é um inteiro positivo qualquer e x é um número real, definimos

xn =

1, se n = 0

x · x . . . x︸ ︷︷ ︸n vezes

, se n > 1

2.4.2

Propriedades

P1: xm · xn = xm+n

P2:xm

xn= xm−n, x = 0

P3: (x · y)n = xn · yn

P4:

(x

y

)m

=xm

ym, y = 0

P5: (xm)n = xmn

P6:

(x

y

)−m

=(yx

)m

Page 13: compilacaoModulo1Nivel2

DRAFT

OBMEP • PECI – Preparação Especial para Competições Internacionais 13

2.5

Radiciação em R

2.5.1

Propriedades

n√a = b⇒ a = bn, onde

√ → radical

n→ índice

a→ radicando

b→ raiz

Se n é um número natural, n ⩾ 2, x, y são reais tais que n√x e n

√y existem, então:

R1: ( n√x)n = x

R2: n√x · n

√y = n

√xy

R3:n√x

n√y= n

√x

y, y = 0.

R4: m√

n√x = mn

√x, m ∈ N, m ⩾ 2.

R5: n√xn =

x, se n é ímpar

|x|, se n é par

2.6

Potência com Expoente Racional

xmn =

n√xm;

m

n∈ Q, n ∈ N, n ⩾ 2.

2.7

Radical Duplo

√a± b =

√a+ c

2±√

a− c

2, onde c =

√a2 − b

2.8

Problemas Propostos

Problema 1 (OCM) Se x2 + x+ 1 = 0, calcule o valor numérico de(x+

1

x

)2

+

(x2 +

1

x2

)2

+

(x3 +

1

x3

)2

+ · · ·+(x27 +

1

x27

)2

.

Problema 2 (ESPANHA-ADAPTADA) Sejam a, b, c números reais não nulos com a+ b+ c = 0 tais que

1

a+

1

b+

1

c=

1

a+ b+ c.

Mostre que também se verifica a igualdade

1

a2013+

1

b2013+

1

c2013=

1

a2013 + b2013 + c2013.

Page 14: compilacaoModulo1Nivel2

DRAFT

14 Álgebra– Produtos Notáveis e Fatoração – Professor Robério Bacelar

Problema 3 Prove que se a+ b+ c = 0, então:

(a)a3 + b3 + c3 = 3abc

(b)a5 + b5 + c5

5=

a3 + b3 + c3

3· a

2 + b2 + c2

2

(c)a7 + b7 + c7

7=

a5 + b5 + c5

5· a

2 + b2 + c2

2

Problema 4 (Torneio das Cidades) Sejam a,b,c e d números reais tais que

a3 + b3 + c3 + d3 = a+ b+ c+ d = 0.

Prove que a soma de dois desses números é zero.

Problema 5 (COLÔMBIA) Sejam a,b,c reais tais que

a12 + b12 + c12 = 8

e(a− b)2 + (b− c)2 + (c− a)2

abc= −

6

a+ b+ c.

Calcule a6 + b6 + c6.

Problema 6 (China) Seja α um número real tal que α3 − α− 1 = 0. Determine o valor numérico de

3√

3α2 − 4α+ α4√2α2 + 3α+ 2.

Problema 7 Sejam x,y,z números reais dois a dois distintos. Prove que a expressão

x(y+ z)

(x− y)(x− z)+

y(x+ z)

(y− z)(y− x)+

z(x+ y)

(z− x)(z− y)

não depende de x, y e z.

Problema 8 Sejam a,b,c números reais distintos. Prove que

a2

(a− b)(a− c)+

b2

(b− c)(b− a)+

c2

(c− a)(c− b)= 1.

Problema 9 (OMG) Seja a = 2008. Determine o valor da soma

k=2007∑k=−2007

1

1+ ak.

Problema 10 Os dois menores lados de um triângulo retângulo, a e b, satisfazem à desigualdade√a2 − 6a

√2+ 19+

√b2 − 4b

√3+ 16 ⩽ 3.

Encontre o perímetro desse triângulo.

Page 15: compilacaoModulo1Nivel2

DRAFT

OBMEP • PECI – Preparação Especial para Competições Internacionais 15

Problema 11 (Canadá) Calcule:

(a)

√1

6+

√5

18−

√1

6−

√5

18

(b)

√1+

2

5·√1+

2

6·√1+

2

7·√1+

2

8· · ·√

1+2

57·√1+

2

58.

Problema 12 (Romênia) Sejam x e y números naturais tas que

2x.3y =(24

12+ 1

3+ 1

4+···+ 1

60

).(24

13+ 1

4+ 1

5+···+ 1

60

)2.(24

14+ 1

5+ 1

6+···+ 1

60

)3· · ·(24

160

)59.

Determinar o valor de x+ y.

Problema 13 (Rússia) Sejam a e b números reais não nulos que satisfazem à equação a2b2(a2b2+4) = 2(a6+b6).

Mostre que a e b não podem ser ambos racionais.

Problema 14 (REOIM) Demonstrar que1√

2+√2+

1√6+

√6>

5

6.

Problema 15 (REOIM) Dados os números

α =√13+

√10+ 2

√13

e

β =

√5+ 2

√3+

√18− 2

√3+ 2

√65− 26

√3,

mostrar que α = β.

Problema 16 (PERU) A expressão

8

√√√√√√√2207−1

2207−1

2207−1

2207− · · ·

pode ser escritana formaa+ b

√c

d, onde a, b, c e d são os menores inteiros positivos possíveis. Determine o valor de

a+ b+ c+ d.

Problema 17 (OCM) Determine qual é o maior dos dois números123456+ 10999

123457+ 10999e123457+ 10999

123458+ 10999.

Problema 18 (CANADÁ) Se os númers reais positivos a1, a2, ...an são so comprimentos dos lados de um polígono

inscrito em uma circunferência, tais que:

a21 + a2

2 + a23 + ...+ a2

n = a1a2 + a2a3 + a3a4 + ...+ ana1,

pode-se afirmar que o polígono é regular? Justifique sua resposta.

Problema 19 (OBM) As representações decimais dos números 21999 e 51999 são escritas lado a lado. Qual o número

total de algarismos escritos?

Problema 20 (PERU) Determine todos o sinteiros positivos x tais que a expressão√92 +

1

112+

92

1002−

1

1100x+ 1

é inteiro.

Page 16: compilacaoModulo1Nivel2

DRAFT

16 Álgebra– Produtos Notáveis e Fatoração – Professor Robério Bacelar

Problema 21 (AIME) Sejam x e y números reais, com x = ±y, ex3 = 13x+ 3y

y3 = 13y+ 3x

Determine (x2 − y2)2.

Problema 22 (RIOPLATENSE) Ache o valor da soma√1+

1

12+

1

22+

√1+

1

22+

1

32+

√1+

1

32+

1

42+ ...+

√1+

1

20052+

1

20062.

Problema 23 (CANADÁ) Ache os números x, y ∈ R+ tais que√x+ 2

√y = 9

x− 4y = 9

Problema 24 (Croácia) Se ax3 = by3 = cz3 e1

x+

1

y+

1

z= 1, prove que

3√

ax2 + by2 + cz2 = 3√a+

3√b+ 3

√c.

Problema 25 Sejam x,y,z reas não nulos com x+ y+ z = 0. Prove que

x2 + y2

x+ y+

y2 + z2

y+ z+

x2 + z2

x+ z=

x3

yz+

y3

xz+

z3

xy.

Problema 26 (Alemanha) Sejam x,y,a e b reais tais quex2 + y2 = 1

x4

a+

y4

b=

1

a+ b

Prove quex8

a3+

y8

b3=

1

(a+ b)3.

Problema 27 (Canadá) Determine todos os reais positivos x,y,z tais que

4x2

4x2 + 1= y

4y2

4y2 + 1= z

4z2

4z2 + 1= x

Problema 28 (Stanford) Ache os números reais x e y tais quex4 + 2x3 − y = −

1

4+√3

y4 + 2y3 − x = −1

4−√3

Page 17: compilacaoModulo1Nivel2

DRAFT

OBMEP • PECI – Preparação Especial para Competições Internacionais 17

Problema 29 Qual a soma dos algarismos do natural n, não nulo, de modo que√25

2+

√625

4− n+

√25

2−

√625

4− n

é também um inteiro?

Problema 30 (EUA) Determine o valor da expressão√1+ 2

√1+ 3

√1+ 4

√1 · · ·.

Problema 31 (Sophie Germain) Fatore x4 + 4y4.

Problema 32 Desenvolva (a+ b+ c+ d)3.

Page 18: compilacaoModulo1Nivel2

DRAFT

..

Álgebra

. 3.

Equações do Segundo Grau

3.1

Equação do Segundo Grau

Uma equação em “x” é dita do Segundo Grau, quando pode ser escrita na forma ax2 + bx + c = 0, onde a, b e c são

números reais e a = 0.

3.1.1

Forma Canônica do Trinômio de Segundo Grau

ax2 + bx+ c = a

(x2 +

b

ax+

c

a

)= a

(x2 +

b

ax+

b2

4a2−

b2

4a2+

c

a

)=

a

[(x2 +

b

ax+

b2

4a2

)−

b2

4a2+

4ac

4a2

]= a

[(x+

b

2a

)2

−b2 − 4ac

4a2

]

3.1.2

Fórmula de Báscara

Resolver uma equação do Segundo Grau significa determinar, através de processos algébricos, os valores de “x” que

verifiquem a igualdade correspondente à equação. A partir dos coeficientes a, b e c da equação algébrica ax2+bx+c =

0 (a = 0) é possível demonstrar a existência de uma relação entre as raízes e esses coeficientes. Veja:

ax2 + bx+ c = 0

multiplicando-se por 4a↓4a2x2 + 4abx+ 4ac = 0

subtraindo-se por 4ac↓4a2x2 + 4abx = −4ac

somando-se b2↓4a2x2 + 4abx+ b2 = b2 − 4ac

fatorando-se o trinômio↓

(2ax+ b)2 = b2 − 4ac

extraindo-se a raiz↓2ax+ b = ±

√b2 − 4ac

subtraindo-se b↓2ax = −b±

√b2 − 4ac

dividindo-se por 2a↓x =

−b±√

b2 − 4ac

2a

A expressão b2 − 4ac chama-se discriminante e é indicada pela letra grega ∆ (delta). Logo:

x =−b±

√∆

2aConforme o valor do discriminante, têm-se as seguintes possibilidades quanto à natureza das raízes:

• ∆ > 0 : a equação admite duas raízes reais e distintas;

• ∆ = 0 : a equação admite duas raízes reais e iguais;

• ∆ < 0 : a equação admite raízes complexas ou imaginárias (raízes não-reais).

18

Page 19: compilacaoModulo1Nivel2

DRAFT

OBMEP • PECI – Preparação Especial para Competições Internacionais 19

3.1.3

Soma das raízes da Equação do Segundo Grau

x1 + x2 =−b+

√∆

2a+

−b−√∆

2a= −

b

a

3.1.4

Produto das Raízes da Equação de Segundo Grau

x1x2 =

(−b+

√∆

2a

)(−b−

√∆

2a

)=

c

a

3.2

Problemas Resolvidos

Exemplo 1 (OBM Júnior – 1996) a, b, c e d são números reais distintos tais que a e b são raízes da equação

x2 − 3cx− 8d = 0 e c e d são raízes da equação x2 − 3ax− 8b = 0. Calcule a soma a+ b+ c+ d.

Solução: É fácil perceber que a+b = 3c e c+d = 3a. Somando e subtraindo membro a membro as duas igualdades,

obtemos b+ d = 2(a+ c)

b− d = 4(c− a).

Como a é raiz de x2 − 3cx− 8d = 0, segue que

a2 − 3ac− 8d = 0 (3.1)

Do mesmo modo, como c é raiz de x2 − 3ax− 8b = 0, temos que

c2 − 3ac− 8b = 0 (3.2)

Subtraindo as igualdades (1) e (2) e utilizando as relações anteriormente obtidas, vem:

a2 − c2 = 8(d− b)⇒ (a− c)(a+ c) = 8× 4(a− c).

Como a− c = 0, concluímos que a+ c = 32.

Portanto, a+ c = 32 e b+ d = 2(a+ c) = 64, donde a+ b+ c+ d = 96.

Exemplo 2 (Romênia) Sejam a, b, c números reais não-nulos tais que a e 4a+ 3b+ 2c têm o mesmo sinal. Mostre

que a equação ax2 + bx+ c = 0 não pode ter duas raízes no intervalo (1, 2).

Solução: Temos que 0 ⩽ 4a+ 3b+ c

a= 4+

3b

a+2c

a= 2x1x2−3 (x1 + x2)+4 = (x1 − 1) (x2 − 2)+(x1 − 2) (x2 − 1).

Se x1 e x2 pertencem ao intervalo (1, 2), então cada termo da soma acima será estritamente negativo, o que é uma

contradição.

3.3

Page 20: compilacaoModulo1Nivel2

DRAFT

20 Álgebra– Equações do Segundo Grau – Professor Robério Bacelar

Forma Fatorada do Trinômio de Segundo Grau

Se a equação ax2 + bx+ c = 0, com a = 0, possui raízes x1 e x2, então é verdadeira a igualdade:

ax2 + bx+ c = a(x− x1)(x− x2).

Prova:

a(x− x1)(x− x2) = a[x2 − (x1 + x2)x+ x1x2] = a

[x2 −

(−b

a

)x+

c

a

]= ax2 + bx+ c.

3.3.1

Exercícios Propostos

Problema 1 Resolva as seguintes equações:

(a) x2 − 5x = 0

(b) x2 − 64 = 0

(c) x2 + 5x+ 6 = 0

(d) x4 + 10x2 − 56 = 0

(e) x6 − 7x3 − 8 = 0

(f) x2 − 2mx+m2 − n2 = 0

(g)

(x+

1

x

)2

− 5

(x+

1

x

)+ 6 = 0

(h) (x2 − x+ 3) − 10(x2 − x) = 105

Problema 2 Determine m para que a equação 2x2 − 8x+m = 0 admita raízes iguais.

Problema 3 Determine m para que a equação 3x2 + 6x+m = 0 admita raízes reais distintas.

Problema 4 Determine m para que a equação x2 − 8x+m = 0 admita raízes imaginárias.

Problema 5 Determine p para que a equação x2 + p2x+ 2px− 4 = 0 admita raízes simétricas.

Problema 6 Determine m na equação x2 − 6x+ 2m = 0 de modo que uma de suas raízes seja ao dobro da outra.

Problema 7 Se a média aritmética de dois números a e b é 7 e a média geométrica entre eles é 9, escreva a equação

do segundo graus que admite a e b como raízes.

Problema 8 Achar a equação do segundo grau na qual uma das raízes é o triplo da outra e a soma dos quadrados

das raízes é 40.

Problema 9 Determine m na equação 3x2 − 2x+ 5m = 0 de modo que a difere ça de suas raízes seja 1.

Problema 10 Qual a relação que deve existir entre os coeficientes da equação ax2+bx+c = 0 para que suas raízes

sejam recíprocas? E para que sejam opostas (simétricas)?

Problema 11 Determine k na equação x2 + kx+ 36 = 0 de modo que a soma dos inversos de suas raízes seja igual

a5

12.

Problema 12 Considere a equação x2 −mx+ 1 = 0 cujas raízes são a e b, reais e distintas. Componha a equação

do segundo graus cujas raízes são a+ 1 e b+ 1.

Page 21: compilacaoModulo1Nivel2

DRAFT

OBMEP • PECI – Preparação Especial para Competições Internacionais 21

Problema 13 Sejam a e b raízes da equação 2x2 − 3x+ k = 0. Determine k de modo que a3 + b3 =−243

8.

Problema 14 (OBM) Determine todos os reais x tais que x2 + x+ 1 =156

x2 + x.

Problema 15 (OBM) Se α é raiz da equação x2 + x− 1 = 0, determine o valor de α5 − 5α.

Problema 16 Seja α uma das raízes da equação x2 − 3x+ 1 = 0. Determine os possíveis valores de 21α− α4.

Problema 17 Provar que a condição para que uma raiz da equação ax2 + bx + c = 0 seja n vezes a outra é

b2 =(n+ 1)2

nac.

Problema 18 Forme a equação biquadrada que admite 2 e 3 como raízes.

Problema 19 Resolva as equações abaixo no conjunto dos números reais:

a)√2x− 3 = 5

b)√x2 + 5x+ 1+ 1 = 2x

c) x+√25− x2 = 1

d)√16+

√x+ 4 = 5

e)√x− 2+

√x− 7 =

√x+ 5+

√x− 10

f) 3√2x2 + 3x− 1 = 2x− 1

Problema 20 Ache o valor de x na equação x =1

1+1

1+1

1+1

. . .

.

Problema 21 Resolva, para x ∈ R a equação√

(2+√3)x +

√(2−

√3)x = 4.

Problema 22 Resolva, para x ∈ R a equação x2 +x2

(x+ 1)2= 3.

Problema 23 Resolva, para x ∈ R a equação x2 +1

x2− 4

(x+

1

x

)+ 5 = 0.

Problema 24 (ESPECEX) Determine todas as raízes reais de 2x2+ 1

x2 =1024

2x+1x

.

Problema 25 Mostre que, para todos a,b,c ∈ R, sendo a = 0, a equação1

x− b+

1

x− c=

1

a2sempre possui duas

raízes reais e distintas.

Problema 26 (OCM) Encontre todas as raízes da equação

√x2 − 2x+ 2

x2 + 4x+ 2+

√x2 + 4x+ 2

x2 − 2x+ 2= 2.

Problema 27 (OCM) Determine o valor de p, para que as raízes x1 e x2 da equação 2x2 − px − 1 = 0 satisfaçam

x21 + x22 = 1.

Problema 28 (OCM) Seja b um número real não nulo de modo que a equação do segundo grau x2 + b2x+√π = 0

tenha raízes reais x1 e x2. Se x1√π = x2(bx2 −

√π), prove que b < 0.

Problema 29 (OCM) Determine o número de raízes da equaçãox2

3+

48

x2= 10

(x

3−

4

x

).

Problema 30 Prove que, para todo x real, temos 2x + 3x − 4x + 6x − 9x ⩽ 1.

Problema 31 (ITA) Sejam x1 e x2 raízes distintas de x2 + px+ 8 = 0, p ∈ R fixo. Prove que |x1 + x2| > 4√2.

Page 22: compilacaoModulo1Nivel2

DRAFT

22 Álgebra– Equações do Segundo Grau – Professor Robério Bacelar

Problema 32 (ITA) Ache a soma das raízes positivas da equação 4x2

− 5.2x2

+ 4 = 0.

Problema 33 (IME) Sejam x1 e x2 as raízes da equação x2 + (m+ 15)x+m = 0. Sabendo que x1 e x2 são naturais,

ache os possíveis valores de m.

Problema 34 (IME) Ache as soluções de√5−

√5− x = x.

Problema 35 Ache as soluções reais da equação x+√x+

√x+ 2+

√x2 + 2x = 3.

Problema 36 (MOLDÁVIA) Seja x um real. Determine todas as soluções reais de

7

√(x+ 127)6 − 8 7

√(x− 127)6 = 7 7

√(x2 − 1272)3.

Problema 37 (MOLDÁVIA) Ache as soluções reais de 4(√1+ x− 1)(

√1− x+ 1) = x.

Problema 38 (Teste Brasil/Cone Sul) Sejam a,b,c números reais tais que as equações x2+ax+1 = 0 e x2+bx+c = 0

têm exatamente uma raiz em comum e as equações x2 + x + a = 0 e x2 + cx + b = 0 também têm uma raiz real em

comum. Determine a soma a+ b+ c.

Problema 39 Sejam α e β as raízes da equação x2 + px + q = 0 e γ e δ as raízes da equação x2 + Px + Q = 0.

Expresse o produto (α− γ)(β− γ)(α− δ)(β− γ) em função dos coeficientes das equações dadas.

Problema 40 Considere as equações x2 + ax + 1 = 0 e x2 + x + a = 0. Determine todos os valores de a para que

as duas equações admitam pelo menos uma raiz comum.

Problema 41 Resolva a equação (x+ 3)(x+ 2)(x+ 1)x = 48.

Page 23: compilacaoModulo1Nivel2

DRAFT

PARTE II

CombinatóriaProfessor Fábio Brochero

..Assuntos do Módulo: Paridade, Princípio da Casa dos Pom-

bos, Grafos.

.

Pré-requisitos: Contagem Simples. Princípio Multiplicativo.

Um conjunto com n elementos possui(n2

)= n(n− 1)/2 subcon-

juntos com 2 elementos (capítulos 1 e 4 da apostila 2 do PIC).

Page 24: compilacaoModulo1Nivel2

DRAFT

..

Combinatória

. 4.

Paridade

Nesta seção o único conhecimento prévio de que precisamos é a diferença entre número par e ímpar. Em particular,

usaremos frequentemente o fato de que uma soma de dois números é ímpar se e somente se um deles é ímpar e o

outro par.

Para começar, pensemos no seguinte problema.

Exemplo 1 Os números 1, 2, 3, . . . , 2013 são escritos em um quadro-negro. É permitido apagar do quadro quaisquer

dois números e substituí-los por sua diferença positiva. Após fazer essa operação várias vezes, sobrará somente um

número no quadro. Este número pode ser igual a 10?

Solução: Vejamos que não é possível mostrando que o último número que aparece no quadro é ímpar. De fato,

cada vez que fazemos o processo temos três possibilidades:

• Trocamos dois pares por um par,

• Trocamos dois ímpares por um par,

• Trocamos um par e um ímpar por um ímpar

Nos três casos a quantidade de números ímpares continuou igual ou diminuiu em 2, portanto conservamos a

paridade da quantidade de números ímpares. Assim como na lista 1, 3, 5, . . . , 2013 existem 1007 números ímpares, ao

final obtemos um número ímpar.

Exemplo 2 É possível cobrir um tabuleiro 2009× 2009 com peças de tamanho 1× 2?

Solução: Observe que o tabuleiro tem um número ímpar de quadradinhos, mas cada peça cobre um número par de

quadradinhos. Portanto não é possível cobrir todo o tabuleiro.

Agora modifiquemos um pouco o problema anterior.

Exemplo 3 É possível cobrir com peças de tamanho 1 × 2 um tabuleiro 2010 × 2010 do qual retiramos a casa do

canto superior direita e a casa do canto inferior esquerdo?

Solução: Observemos que o argumento anterior já não funciona, porque agora temos que cobrir 20102 − 2 quadri-

nhos, que é par, com peças de tamanho 1 × 2. Porém, podemos pensar da seguinte forma: vamos pintar o tabuleiro

como um tabuleiro de xadrez, isto é, alternadamente de branco e preto. Assim, se dois quadrinhos compartilham um

lado, então eles têm cores diferente. Deste modo, cada peça que colocamos sobre o tabuleiro vai cobrir um quadrinho

preto e outro branco. Então, a quantidade de quadradinhos brancos e pretos cobertos é a mesma.

Agora, os cantos superior direito e inferior esquerdo têm a mesma cor porque estão sobre a mesma diagonal.

Logo tiramos dois quadrinhos da mesma cor e no tabuleiro restante ficaram mais casas de uma cor que de outra e,

portanto, o restante do tabuleiro não pode ser coberto pelas peças.

24

Page 25: compilacaoModulo1Nivel2

DRAFT

OBMEP • PECI – Preparação Especial para Competições Internacionais 25

Exemplo 4 Considere a expressão

1± 2± 3± 4 · · · ± 2009.

É possível escolher os sinais + ou − de tal forma que o resultado final seja 1000?

Solução: Novamente a resposta é não, por paridade. Como a quantidade de número ímpares nesta soma é ímpar,

então para qualquer escolha dos sinais o resultado final será sempre ímpar.

Exemplo 5 Uma linha poligonal fechada é composta por 7 segmentos. É possível que uma reta intersecte o interior

de cada um dos 7 segmentos?

Solução: Suponha que a reta ℓ intersecte os segmentos P1P2, P2P3, . . . , P6 P7 e P7P1. A reta divide o plano em dois

semiplanos, os quais chamaremos de α e β.

..

α

.

β

.

.....

P1

. P3.

P5

.

P7

.

P2

.

P4

.P6...

Suponha que o ponto P1 seja um ponto do semiplano α. Então P2 está em β, P3 em α e assim sucessivamente.

Deste modo P7 está em α e como o segmento P1P7 é cortado por ℓ, P1 está em β, absurdo.

Exemplo 6 É possível que um cavalo faça um percurso passando por todas as casas de um tabuleiro de xadrez 8×8,

começando no canto superior esquerdo e terminando no canto inferior direito?

NZ0Z0Z0ZZ0Z0Z0Z00Z0Z0Z0ZZ0Z0Z0Z00Z0Z0Z0ZZ0Z0Z0Z00Z0Z0Z0ZZ0Z0Z0Z0

Solução: Observe que ao movimentar uma vez o cavalo no tabuleiro, a cor da casa onde chega é diferente da cor da

casa onde sai. Se queremos que o cavalo percorra todas as casas do tabuleiro uma única vez, teremos que movê-lo 63

vezes. Portanto, teremos um número ímpar de trocas de cor, e a cor da primeira casa e a da última serão diferentes.

Desse modo, é impossível fazer tal passeio, porque a casa do canto superior esquerdo e a casa do canto inferior direito

possuem a mesma cor.

..

2

.

1

.

6

.

5

.

8

.

7

.

4

.

3

Page 26: compilacaoModulo1Nivel2

DRAFT

26 Combinatória– Paridade – Professor Fábio Brochero

Exemplo 7 (Seleção para o PECI 2013) Colocamos 15 cavalos em um tabuleiro 15 × 15 de modo que não existam

dois cavalos em uma mesma linha ou em uma mesma coluna. Cada cavalo faz então um movimento. Prove que agora

existem dois cavalos na mesma linha ou na mesma coluna.

Observação: O cavalo movimenta-se em L. Assim, um cavalo na casa marcada com

.

pode mover-se para as casas destacadas.

.Solução: Adaptada da Solução de Thiago Lucas Faustino da Silva de Itumbiara – Goiás.

Um dos motivos é que 15 é ímpar, se o número de cavalos fosse par, poderíamos colocar os cavalos de modo que

dois cavalos troquem de posição no movimento, se fizermos isto, sobra um cavalo sem movimentar-se.

Uma maneira de provar é a seguinte, vamos numerar cada cavalo pela posição, se está na linha 1 coluna 2, como

1+ 2 = 3, o número dele é 3. A soma de todos os números será

2(1+ 2+ · · ·+ 15) = 450.

Se não houver cavalos diferentes na mesma linha ou coluna, a cada movimento, o número de um cavalo se altera em:

3, 1, −1 ou −3.

São números ímpares, para que continue com apenas um cavalo em cada linha e em cada coluna, a soma dos

números dos cavalos deveria ser 450 o que implicaria uma alteração nula nesta soma, ou seja, 0. A alteração total

será a soma das alterações individuais de cada cavalo. Porém, 0 é par e a soma de 15 números ímpares não pode ser

par, por isso, é impossível que continue um cavalo em cada linha e um em cada coluna. Logo, existem dois cavalos na

mesma linha ou na mesma coluna.

Exemplo 8 Em uma urna temos 2013 bolas brancas e 2013 bolas pretas e fora dela temos uma quantidade grande

de bolas brancas e pretas. A cada jogada, tiramos duas bolas da urna sem olhar: Se as duas bolas tiradas são da mesma

cor, devolvemos uma bola preta, mas se as bolas tiradas são de cores distintas, devolvemos uma branca. No final fica

uma única bola na urna. Esta bola que fica na urna pode ser preta?

Sorteio P B

. .. −1 0.. +1 −2

. .. −1 0

Solução: Temos três situações possíveis quando retiramos duas bolas da urna:

(1) As duas bolas são pretas : Neste caso devolvemos à urna uma bola preta. Assim, o número de bolas pretas diminui

em um e o número de bolas brancas permanece igual;

(2) As duas bolas são brancas: Neste caso devolvemos à urna uma bola preta. Assim, o número de bolas pretas

aumenta em um e o número de bolas brancas diminui em dois;

(3) As duas bolas são de cores distintas: Neste caso devolvemos à urna uma bola branca. Assim, o número de bolas

pretas diminui em um e o número de bolas brancas permanece igual.

Page 27: compilacaoModulo1Nivel2

DRAFT

OBMEP • PECI – Preparação Especial para Competições Internacionais 27

Observemos que nos três casos, o número de bolas brancas permanece igual ou diminui em dois, assim a paridade

do número de bolas brancas na urna não muda em cada jogada. Portanto, o número de bolas brancas na urna nunca

pode ser zero, pois começamos com um número ímpar de bolas brancas na urna. Logo, em qualquer caso a última

bola na urna é branca.

Exemplo 9 Em um tabuleiro 8 × 8, com casas pintadas de branco ou preto, dizemos que uma linha ou coluna é

repintada quando trocamos as cores de todas as casas dessa linha ou coluna, isto é, uma casa branca vira preta e vice

versa.

Os quatro cantos de um tabuleiro totalmente branco são pintados de preto. Mostrar que é impossível, repintando linhas

e colunas, deixar todas casas do tabuleiro da mesma cor.

Solução: Suponhamos que queremos pintar todas as casas do tabuleiro de preto, já que ao obter um tabuleiro

totalmente preto é fácil obter um tabuleiro totalmente branco, bastando repintar todas as linhas do tabuleiro e vice-

versa. Consideremos as quatro casas do canto inferior esquerda do tabuleiro 8× 8 que forma um subtabuleiro 2× 2.

No começo, este subtabuleiro tem uma casa preta e três brancas, como ilustrado na figura:

..a

.b

.c

.

d

Denotamos por a e b o número de vezes que a 1a e a 2a colunas foram repintadas, e denotamos por c e d o número

de vezes que a 7a e a 8a linhas foram repintadas. Como a cada duas vezes que uma casa é repintada ela volta à cor

original, se quisermos que todo o tabuleiro fique preto, precisamos em particular que

• a+ c seja par;

• a+ d seja ímpar;

• b+ c seja ímpar;

• b+ d seja ímpar.

Como (a + c) + (a + d) + (b + c) + (b + d) = 2(a + b + c + d) é par, as condições anteriores não podem ocorrer

simultaneamente e, portanto, é impossível repintar o tabuleiro de modo que fique todo da mesma cor.

Exemplo 10 2013 cartas que tem uma face azul e a outra vermelha são colocadas em uma mesa de forma que

algumas ficam com a face vermelha para cima e as outras com a face azul. Dois jogadores podem realizar cada um

em sua vez, uma das seguintes operações:

• retirar da mesa qualquer quantidade de cartas com a mesma cor na face de cima;

• virar qualquer quantidade de cartas com a mesma cor na face de cima.

Ganha quem pegar a última carta. Qual dos dois jogadores tem uma estratégia vencedora?

Solução: Como o número inicial de cartas é ímpar, temos que inicialmente existe uma quantidade par de cartas de

uma cor viradas para acima e um número ímpar da outra cor. Vejamos como o primeiro jogador tem uma estratégia

vencedora:

Em seu turno, o primeiro jogador remove cartas de forma a ter a mesma quantidade de cartas azuis e vermelhas.

Ele sempre pode fazer isto, pois inicialmente há mais cartas de uma cor do que de outra e quando o segundo jogador

Page 28: compilacaoModulo1Nivel2

DRAFT

28 Combinatória– Paridade – Professor Fábio Brochero

recebe uma quantidade igual de cartas azuis e vermelhas, qualquer operação que ele faça vai diminuir a quantidade de

cartas de uma cor e manter ou aumentar a quantidade de cartas da outra cor, permitindo ao primeiro jogador repetir

o procedimento. Como o primeiro jogador sempre está removendo cartas, vai chegar um momento em que o segundo

jogador não terá outras opções a não ser acabar com as cartas de uma cor, permitindo ao primeiro jogador acabar

com as cartas da outra cor.

Exemplo 11 Escrevemos +1 ou −1 em cada casa de um tabuleiro 5× 5. Depois calculamos o produto dos números

em cada coluna e em cada linha obtendo no total dez números. É possível que a soma destes dez números seja igual a

0?

Solução: O problema é equivalente a começar por um tabuleiro preenchido com 25 uns e a pergunta é se é possível ir

mudando os sinais de algumas destas entradas de forma que a soma dos produtos seja 0. Suponhamos que mudamos

uma das casas de sinal. Logo, o sinal do produto da linha e a coluna à qual pertence também muda. Assim temos três

possíveis casos

• se os produtos eram +1, depois da mudança os dois produtos ficaram −1;

• se os produtos eram −1, depois da mudança os dois produtos ficaram +1;

• se um dos produtos era +1 e o outro −1, depois da mudança um dos produtos fica −1 e o outro +1.

Assim, no primeiro caso a soma diminui em 4, no segundo aumenta em 4 e no terceiro a soma não muda. Dado que a

soma inicial era 10, os possíveis valores que podem ser obtidos são 10, 6, 2,−2,−6 e −10. Deste modo, a soma nunca

pode ser 0.

Exemplo 12 Um aparelho é formado por 64 lâmpadas montadas em um tabuleiro 8× 8 com 16 interruptores, onde

cada interruptor muda o estado das lâmpadas de uma linha ou coluna do aparelho. Supondo que no começo todas as

lâmpadas estão desligadas, é possível acionar os interruptores de tal forma que ao final fique exatamente uma lâmpada

acesa? E duas lâmpadas acesas?

Solução: Podemos transformar o problema no seguinte: Colocamos em cada casa de um tabuleiro 8× 8 o número

0 ou 1 e em cada passo, podemos mudar todos os números de uma linha ou de uma coluna. Se começamos com o

tabuleiro preenchido com 64 uns, é possível obter, depois deste tipo de mudanças, exatamente um número 1. Obser-

vemos que cada vez que mudamos uma linha ou coluna, mudamos os números de oito casas, assim se em uma linha

ou coluna arbitrária temos a uns e b, zeros, depois da mudança vamos ter a zeros e b uns nessa linha ou coluna.

Como a + b = 8, temos que a e b têm a mesma paridade. Assim, a paridade do número de uns e zeros no tabuleiro

não muda depois de tal troca. Como inicialmente temos 64 uns, ao final é impossível obter exatamente um número 1

e isto resolve a primeira parte do problema.

Seguindo a mesma ideia, suponhamos que em algum momento temos A zeros e B uns no tabuleiro, ao mudar uma

linha ou coluna obtemos A+ b− a zeros e B+ a− b uns. Assim inicialmente a diferença entre o número de zeros e

de uns era B −A e depois da mudança é B −A + 2(a − b). De igual forma, como a + b = 8 temos que a − b é par, e

portanto 2(a−b) é múltiplo de 4. Portanto, como inicialmente B−A é 64, depois de algumas mudanças esta diferença

entre o números de uns e de zeros tem que ser divisível por 4, portanto não pode ser 2. Observe que este argumento

também resolve a primeira parte do problema.

Exemplo 13 Num tabuleiro de damas 8 × 8 é colocada uma pedra em cada casa das três primeiras linhas. Uma

pedra se move pulando sobre outra pedra, ficando na casa simétrica a que estava em relação à pedra pulada, desde que

esta casa esteja vazia. É possível que após uma sequência de pulos, seja obtida uma configuração com as três últimas

linhas do tabuleiro preenchidas pelas pedras?

Page 29: compilacaoModulo1Nivel2

DRAFT

OBMEP • PECI – Preparação Especial para Competições Internacionais 29

Solução: Se numerarmos as linhas do tabuleiro de 1 a 8, então no começo as linhas 1, 2 e 3 estarão totalmente

preenchidas. Observe que depois de cada movimento, os números da linha inicial e final da pedra têm a mesma

paridade (se uma pedra na linha a pula uma pedra na linha a + b, a pedra vai parar na linha a + 2b, com a mesma

paridade da linha a). Como inicialmente tínhamos 16 pedras em linhas ímpares e 8 em linhas pares, depois de qualquer

número de movimentos vamos ter a mesma configuração. Deste modo, é impossível preencher as três últimas linhas

do tabuleiro, já que neste caso teríamos 8 pedras na linha 7 e 16 nas linhas 6 e 8 juntas.

Exemplo 14 Um baralho espanhol (40 cartas) é distribuído em um arranjo de 5 linhas e 8 colunas com todas as

cartas viradas para baixo. Dois jogadores se alternam fazendo a seguinte jogada: o primeiro seleciona uma linha que

não tenha todas as cartas viradas para cima, vira a primeira carta que esteja virada para baixo e depois vira e desvira

as cartas seguintes a vontade. Perde o primeiro jogador que não consegue fazer uma jogada. Qual dos jogadores tem

uma estratégia vencedora?

Solução: Observemos que o primeiro jogador sempre tem uma estratégia vencedora. Para isto, basta que em sua

primeira jogada, o primeiro jogador vire todas as cartas da primeira linha, de modo que as cartas que continuam “em

jogo” estejam nas outras quatro linhas. Em seguida, basta para o primeiro jogador copiar as jogadas do segundo

jogador. De fato, denote as linhas por a, A, b e B; se o segundo jogador joga nas linhas a ou b, o primeiro copia a

jogada nas linhas A ou B e vice-versa. Assim, o primeiro jogador não terá jogadas apenas quando o segundo jogador

não tiver.

Exemplo 15 Sobre uma reta r tome um segmento de extremos A e B. Fora do segmento AB, mas sobre a reta r,

existem 19 pontos distintos tais que alguns estão antes do ponto A e outros depois do ponto B, mas nenhum ponto está

entre A e B. A soma das distâncias de todos os pontos que estão antes de A até o ponto A é igual à soma das distâncias

de todos os pontos que estão depois de B até o ponto B.

Prove que a soma das distâncias de todos os pontos que estão antes de A até o ponto B não pode ser igual à soma das

distâncias de todos os pontos que estão depois de B até o ponto A.

Solução: Chamemos de d a distância entre A e B, x1, . . . , xj as distâncias dos pontos que estão antes de A ao ponto

A e y1, . . . , yk as distâncias dos pontos que estão depois de B ao ponto B. Pela hipótese do problema temos que

• j+ k = 19;

• x1 + · · ·+ xj = y1 + · · ·+ yk;

• A distância dos pontos que estão depois de B ao ponto A é y1 + d, . . . , yk + d;

• A distância dos pontos que estão antes de A ao ponto B é x1 + d, . . . , xj + d.

Segue que a soma das distâncias dos pontos que estão depois de B ao ponto A é

(y1 + d) + · · ·+ (yk + d) = y1 + · · ·yk + kd = x1 + · · · xj + kd

e a soma das distâncias dos pontos que estão antes de A ao ponto B é

(x1 + d) + · · ·+ (xj + d) = x1 + · · · xj + jd,

Caso estas duas somas fossem iguais teríamos que k = j =19

2, um absurdo. Portanto as somas não podem ser iguais.

Page 30: compilacaoModulo1Nivel2

DRAFT

30 Combinatória– Paridade – Professor Fábio Brochero

Exemplo 16 (OBM) Pintamos de vermelho ou azul 100 pontos em uma reta.

• Se dois pontos vizinhos são vermelhos, pintamos o segmento que os une de vermelho;

• Se pontos vizinhos são azuis, pintamos o segmento de azul;

• Finalmente, se dois pontos vizinhos têm cores distintas, pintamos o segmento de verde.

Feito isto, existem exatamente 20 segmentos verdes. O ponto mais à esquerda é vermelho. É possível determinar com

estes dados a cor do ponto mais à direita? Em caso afirmativo, qual a cor deste ponto?

Solução: Somente existem mudanças de cor entre pontos consecutivos quando o segmento que os une é de cor

verde. Além disso, se temos um número par de segmentos verdes entre dois pontos, vamos ter um número par de

mudanças de cor entre eles, assim estes dois pontos têm a mesma cor e de igual forma, se o número de segmentos

verdes entre dois pontos é ímpar, vamos ter um número ímpar de mudanças de cor entre eles e assim estes dois

pontos têm cores diferentes.

No caso particular em que temos 20 segmentos verdes, concluímos que o ponto da ponta esquerda e o ponto da

ponta direita tem a mesma cor, portanto o ponto da ponta direita é vermelho.

Exemplo 17 Lema de Sperner. Um triângulo T é dividido em triângulos menores de modo que quaisquer dois

triângulos pequenos ou não possuem pontos em comum, ou possuem um ponto em comum, ou possuem um lado completo

em comum. Os três vértices de T são etiquetados com A, B, C. Todos os vértices dos triângulos pequenos também são

etiquetados com A, B, C, com a única restrição de que vértices sobre o lado AB não recebem a etiqueta C, vértices

sobre o lado AC não recebem a etiqueta B e vértices sobre o lado BC não recebem a etiqueta A. Mostre que dentre os

triângulos pequenos, existe no mínimo um cujos vértices estão etiquetados com A, B e C.

................

A

.B .

B

.

C

.

A

.

A

.

A

.

B

.

A

.

A

.

B

.

C

.

A

.

B

.

C

Solução: Para resolver este problema, vamos atribuir valores a cada um dos lados dos triângulos menores da

seguinte forma: atribuímos um 0 se os extremos do lado estão etiquetados com letras iguais, caso contrário atribuímos

um 1. Desta forma, os lados de cada triângulo podem ter os seguintes valores:

• 0− 0− 0, se os três vértices têm a mesma etiqueta;

• 1− 1− 0, se dos vértices têm uma mesma etiqueta e o terceiro tem uma etiqueta distinta;

• 1− 1− 1, se todos os vértices têm etiquetas distintas.

Page 31: compilacaoModulo1Nivel2

DRAFT

OBMEP • PECI – Preparação Especial para Competições Internacionais 31

................

A

.B .

B

.

C

.

A

.

A

.

A

.

B

.

A

.

A

.

B

.

C

.

A

.

B

.

C

.

1

.

1

.

0

.

1

.

1

.0 .

0

.

0

.

1

.

1

.

1

.

1

.

1

.

1

.

0

.

1

.

1

.

0

.

0

.1

.1 .1

.

1

.1.

0

.

1

.

1

.1

.

0

.

1

Agora podemos atribuir a cada triângulo a soma dos números atribuídos a seus lados. Assim, cada triângulo pode

receber os números 0, 2 ou 3. Observe que se um triângulo recebe o número 3 é porque seus três vértices estão

etiquetados por letras distintas e, neste caso, o problema estaria resolvido.

Para isto, seja S a soma dos números atribuídos aos triângulos. Se mostramos que S é ímpar, então uma das

parcelas é ímpar e, portanto, umas das parcelas é 3. Para determinar a paridade de S, basta observar que cada lado

interior ao triângulo maior é somado duas vezes em S porque este lado pertence a dois triângulos distintos, enquanto

todo lado que pertence ao bordo do triângulo maior é contado apenas uma vez. Logo, a paridade de S é igual à paridade

da soma dos lados que estão na borda do triângulo maior.

Neste ponto, o problema é parecido ao problema anterior: Num lado do triângulo do lado maior estamos atribuindo

0 se não há troca de cor e 1 se há troca de cor. Como o ponto inicial e o ponto final são distintos, então o número de

trocas de cor é ímpar. Assim, o número de uns em qualquer um dos lados do triângulo maior é ímpar e como a soma

de três números ímpares é ímpar, temos que S é ímpar, como queríamos demonstrar.

4.1

Problemas Propostos

Problema 1 (OPM) A seguir, temos a parte de baixo de uma figura.

A figura delimita uma região. O círculo preto está dentro ou fora dessa região? Lembre-se: você deve justificar sua

resposta!

Problema 2 Das 28 peças de um dominó, foram perdidas as 5 peças a seguir:

. . . . .

Bruno notou que com as peças restantes poderia fazer um arranjo de modo que todas as peças fossem enfileiradas,

respeitando as regras do jogo de dominó. Qual a soma dos números das extremidades da fileira?

Page 32: compilacaoModulo1Nivel2

DRAFT

32 Combinatória– Paridade – Professor Fábio Brochero

Problema 3 O produto de 50 inteiros é 1. Mostrar que a soma dos inteiros não pode ser 0. Mostrar que a soma dos

inteiros não pode ser 4 nem 5.

Problema 4 Num tabuleiro de xadrez (8×8) um cavalo parte de uma casa do tabuleiro e faz sucessivos movimentos

até voltar à casa inicial. Mostre que o cavalo realizou um número par de jogadas. Atenção! O cavalo faz movimentos

em L conforme indicado na figura (duas casas em uma direção e uma em outra).

..

Problema 5 Um número de 17 algarismos é escrito. Depois obtemos um número escrevendo os algarismos deste

de forma invertida. Mostre que a soma destes dois números sempre contém um algarismo par.

Problema 6 10 meninos e 11 meninas ficam sentados ao redor de uma mesa circular. Mostre que existe uma pessoa

nesta mesa cujos vizinhos são meninos.

Problema 7 Prove que a igualdade1

a+

1

b+

1

c+

1

d+

1

e+

1

f= 1

não tem soluções no conjunto dos naturais ímpares.

Problema 8 Os números 1, 0, 1, 0, 0, 0 estão dispostos ao redor de uma circunferência e vamos mudando estes

números seguindo a seguinte regra: pegamos dois números adjacentes e somamos 1 a cada um deles. É possível

depois de um número finito de passos obter 6 números iguais ao redor da circunferência?

Problema 9 (Cone Sul – 1995) Há dez pontos marcados sobre uma circunferência, numerados de 1 a 10 em al-

guma ordem. Traçamos em seguida todos os segmentos que esses pontos determinam e os pintamos, uns de vermelho

e os demais de azul. É possível, sem trocar as cores dos segmentos, reenumerar os pontos de 1 a 10 de modo que se

dois números eram unidos por um segmento vermelho agora são unidos por um segmento azul e vice-versa?

Problema 10 É possível dividir um polígono de 13 lados em um número finito de paralelogramos?

Page 33: compilacaoModulo1Nivel2

DRAFT

..

Combinátória

. 5.

Princípio da Casa dos Pombos

O Princípio das Casas dos Pombos é uma ferramenta elementar, mas muito poderosa para resolver problemas. A

idéia do princípio é altamente intuitiva: “Se temos mais pombos que casas, em alguma casa tem que ficar mais que

um pombo”.

.. f. f. f. f.

f. f.

6 pombos em 5 casas: pelo menos 1 casa tem 2 pombos

De um modo mais geral,

“Se temos nk+ 1 ou mais pombos para colocar em n casas, alguma casa ficará k+ 1 ou mais pombos”

..

f. f.

f. f.

f. f.

f. f.

f. f.

f. f. f.

13 pombos em 6 casas: pelo menos 1 casa tem 3 pombos

Observemos que este princípio pode ser provado raciocionando por contradição: de fato, se em cada casa colocarmos

k ou menos pombos, então colocaremos no máximo nk pombos. Como temos nk+ 1 pombos, alguma casa terá pelo

menos k+ 1 pombos.

O Princípio da Casa dos Pombos também é conhecido como Princípio de Dirichlet ou Princípio das Gavetas.

5.1

Exemplos Introdutórios

Exemplo 1 Seja A = 1, 2, . . . , 99, 100 o conjunto dos números de 1 a 100 e deste pegamos ao acaso 51 números.

Demonstrar que entre os números escolhidos sempre existem dois que são consecutivos.

Solução: Para provar isto, primeiro escolhamos gavetas adequadas ao problema.

..g1

.g2

.g3

. . . ..g50

Suponhamos que construímos cinquenta gavetas g1, g2, . . . , g50 para colocar os 51 números escolhidos. Na primeira

irão o 1 e o 2, na segunda o 3 e o 4, e o geral, na i-ésima irão o 2i− 1 e o 2i. Agora, temos construído 50 gavetas e cada

número entre 1 e 100 temos associado alguma gaveta, onde, não importa como escolhamos os 51 números sempre

haverá uma gaveta com dois números e estes, graças a nossa construção, serão consecutivos.

Podemos generalizar este resultado considerando os números 1, 2, . . . , 2n e escolhendo dentre eles n+1 números

ao acaso.

Exemplo 2 Escolhemos n + 1 elementos distintos do conjunto 1, 2, 3, . . . , 3n, onde n é um inteiro positivo dado.

Prove que há dois deles x e y tais que xy+ 1 ou 4xy+ 1 seja um quadrado perfeito.

33

Page 34: compilacaoModulo1Nivel2

DRAFT

34 Combinatória– Princípio da Casa dos Pombos – Professor Fábio Brochero

Solução: Consideremos os n conjuntos

1, 2, 3, 4, 5, 6, . . . , 3n− 2, 3n− 1, 3n.

Pelo Princípio das Casas dos Pombos, ao menos dois números x e y dos n+ 1 escolhidos estão em um mesmo destes

n conjuntos, donde |x− y| = 1 ou 2.

Suponhamos, sem perda de generalidade, que x > y.

• Se x− y = 1, então 4xy+ 1 = 4(y+ 1)y+ 1 = (2y+ 1)2.

• Se x− y = 2, então xy+ 1 = (y+ 2)y+ 1 = (y+ 1)2.

Em qualquer caso, xy+ 1 ou 4xy+ 1 será um quadrado perfeito.

Exemplo 3 Seja A = 1, 2, . . . , 99, 100 o conjunto dos números de 1 a 100 e deste escolhemos ao acaso 55 números.

Demonstrar que entre os números escolhidos sempre existem dois tais que sua diferença é 9.

Solução: Do mesmo modo que no exemplo anterior o problema é descobrir como formar as gavetas. Consideremos

as gavetas 0, 1, 2, . . . , 8, onde o número n é colocado na gaveta i se e só se o resto ao dividir n por 9 é i. Como

escolhemos 55 = 9×6+1 números, isto nos indica que existirá uma gaveta j na qual tenham ficado 7 ou mais números

dos escolhidos. Mas cada gaveta pode conter no máximo 12 números (por exemplo, a gaveta 1 pode conter quaisquer

elementos do conjunto 1, 10, 19, 28, 37, 46, 55, 64, 73, 82, 91, 100. Segue, pelo exemplo 1, que existiram dois números

que serão consecutivos em tal conjunto e portanto sua diferença é 9.

Exemplo 4 Mostre que em qualquer poliedro existem duas faces com a mesma quantidade de lados.

Solução: Considere a face S que tem a maior quantidade de lados, e suponha que esta face tenha N lados. Agora

considere N faces que são adjacentes à face S. Cada uma desta faces tem entre 3 e N lados. Como temos N faces e

N − 2 possíveis valores para o número de seus lados, temos pelo Princípio das Casas dos Pombos que duas destas

faces têm a mesma quantidade de lados.

5.2

Pombos Geométricos

Exemplo 5 São dados 5 pontos no interior de um quadrado de lado 1. Prove que dentre estes pontos existem dois,

digamos A e B, tais que AB <√2/2.

Solução: Divida o quadrado em quatro quadrados de lado 1/2.

.

Pelo Princípio da Casa dos Pombos, existem dois dos cinco pontos em um mesmo quadrado de lado 1/2. A distância

entre estes pontos é menor que a diagonal deste quadrado (prove!). Como a medida da diagonal de um quadrado de

lado ℓ é ℓ√2, existirão dois pontos determinado uma distância menor que

√2/2.

Exemplo 6 Cinco pontos estão no interior de um triângulo equilátero de lado 1. Prove que dentre estes pontos

existem dois, digamos A e B, tais que AB < 1/2.

Page 35: compilacaoModulo1Nivel2

DRAFT

OBMEP • PECI – Preparação Especial para Competições Internacionais 35

.

Solução: Dividimos o triângulo equilátero ligando os pontos médios dos lados. Obtemos assim quatro triângulos

equiláteros de lado 1/2. Dos cinco pontos considerados, pelo Princípio das Casas dos Pombos, existem dois que estão

no mesmo triângulo pequeno, o qual vamos chamar de T1. Os dois pontos que estão em T1 vamos chamar de A e B.

Observe que como os pontos estão no interior do triângulo inicial, eles não podem coincidir com os vértices de T1.

Como o lado de T1 é 1/2, então AB < 1/2.

5.3

Problemas Propostos

Problema 1 Cinquenta e um pontos estão no interior de um quadrado de lado 1. Prove que existem três pontos

que podem ser cobertos por um círculo de raio 1/7.

Problema 2 (Balcânica Júnior – 1997) Mostre que dados quaisquer 9 pontos no interior de um quadrado, mostre

que três deles determinam um triângulo de área menor que 1/8.

Problema 3 (OCM)

(a) Marca-se 151 pontos distintos no interior de um quadrado unitário Q. Divide-se Q em 36 quadrados idênticos e

justapostos e considera-se os círculos circunscritos a estes pequenos quadrados. Prove que existem pelo menos

cinco pontos, dos 151 marcados, que estão no interior de um círculo de raio igual a 2/13;

(b) Marcamos 383 pontos distintos no interior de um cubo unitário. Prove que, entre estes 383 pontos, existem pelo

menos 4 que estão no interior de uma esfera de raio igual a 4/23.

5.4

Ramsey

Exemplo 7 Pintamos cada um dos 15 segmentos determinados por 6 pontos de azul ou vermelho. Mostre que existem

três pontos que determinam um triângulo monocromático, isto é, com os três lados da cor azul ou os três lados da cor

vermelha.

Solução: Sejam A, B, C, D, E e F os pontos, como na figura à esquerda. Vamos representar a cor azul por uma

aresta “cheia” e a cor vermelha por uma aresta tracejada.

..

A

.

F

.

C

.

D

.B . E...... ..

A

.

C

.

D

.

B

.

E

.....

Vamos olhar para uma ponto qualquer, por exemplo, A. Dos cinco segmentos que partem de A pelo menos três

são da cor azul ou pelo menos três são da cor vermelha.

Page 36: compilacaoModulo1Nivel2

DRAFT

36 Combinatória– Princípio da Casa dos Pombos – Professor Fábio Brochero

Suponha que partem três segmentos azuis, sem perda de generalidade, para os pontos C, E e F. Se dois destes

pontos são ligados por um segmento azul, acabou. Caso contrário os segmentos CE, EF e FC são vermelhos e o

problema também acabou.

Este problema pode ser enunciado de outro modo: Prove que em qualquer grupo de seis pessoas existem três

pessoas que se conhecem mutuamente ou seis pessoas que não se conhecem mutuamente. Neste problema admitimos

que a relação conhecer é simétrica, ou seja, se A conhece B então B conhece A.

O desenho à direita mostra que a afirmação não é verdadeira para um grupo de 5 pessoas.

Exemplo 8 Cada ponto do plano é colorido usando uma dentre k cores. Prove que podemos encontrar um retângulo

com todos os vértices da mesma cor

Solução: Antes de resolver o caso geral, ilustraremos a solução com o caso k = 2, isto é, suponhamos que estamos

colorindo todos os pontos do plano de azul ou verde. Consideremos uma configuração formada por 3 retas paralelas

horizontais e 9 retas paralelas verticais. Observe que cada reta horizontal corta cada reta vertical em três pontos, e

temos oito possíveis configurações para as cores desses pontos:

.. V.

V

.

V

. V.

V

.

A

. V.

A

.

V

. A.

V

.

V

. V.

A

.

A

. A.

V

.

A

. A.

A

.

V

. A.

A

.

A

Como temos nove retas verticais, pelo Princípio das Casas dos Pombos temos que uma das configurações anteriores

se repete pelo menos duas vezes. Como em cada uma das configurações anteriores existe uma cor que se repete,

pegando os pontos com a mesma cor, eles forma um retângulo queríamos mostrar.

A ideia do caso geral é exatamente a mesma. Para isto, pegamos k+ 1 retas horizontais e kk+1 + 1 retas verticais.

Assim, cada reta vertical corta cada reta horizontal em k + 1 pontos. Como temos k cores, temos kk+1 possíveis

configurações de cores para k + 1 pontos. Logo, temos duas retas horizontais que cortam as retas verticais em

configurações de cores iguais. Nesta configuração de cores, temos uma cor que ser repete, assim basta pegar os

quatro pontos com a mesma cor na mesma configuração que eles formam um retângulo.

5.4.1

Problemas Propostos

Problema 4 (África do Sul – 1997) Seis pontos são ligados dois a dois por segmentos vermelhos ou azuis. É verdade

que deve existir um caminho fechado formado por quatro segmentos, todos da mesma cor?

Problema 5 (IMO - 1964) Dezessete pessoas se comunicam por carta, cada uma com todas as demais. As cartas

tratam somente de três assuntos. Cada par de pessoas trata em suas cartas de somente um deste temas. Demonstrar

que existem pelo menos 3 pessoas que trocam cartas sobre o mesmo tema.

Problema 6 Numa festa com 10 pessoas mostre que no mínimo uma das afirmações abaixo é verdadeira:

1. existem 4 pessoas que se conhecem mutuamente;

2. existem 4 pessoas tais que uma delas não conhece nenhuma das outras três.

Problema 7 (Cone Sul – 1993) Prove que dado um inteiro positivo n existe um inteiro positivo kn com a seguinte

propriedade:

Dados kn pontos quaisquer no espaço, 4 a 4 não coplanares, e associados números inteiros entre 1 e n a cada aresta

que liga dois desses pontos, há necessariamente um triângulo determinado por 3 deles a cujas arestas está associado o

mesmo número.

Page 37: compilacaoModulo1Nivel2

DRAFT

OBMEP • PECI – Preparação Especial para Competições Internacionais 37

Problema 8 Se os pontos do plano forem coloridos com três cores, sempre existirão dois pontos distanciados de

1 unidade com a mesma cor. Provar.

Problema 9 O conjunto dos inteiros positivos de 1 a 16 é dividido em três subconjuntos. Prove que existem três

inteiros positivos a, b e c que pertencem a um mesmo subconjunto e satisfazem a condição a+ b = c.

5.5

Pombos e Divisibilidade

Observemos que em alguns casos empregamos tal princípio de forma natural, por exemplo: Dados três números

inteiros, existem dois que tem a mesma paridade. De fato como só temos duas possíveis paridades (que fazem o papel

de casas de pombo) e três números (que fazem o papel de pombos), pelo Princípio das Casas dos Pombos temos que

dois o mais estão na mesma casa, isto é, dois números tem a mesma paridade.

De fato este problema pode ser generalizado da seguinte forma: Dados n+ 1 números inteiros, então dois destes

números deixam o mesmo resto quando divididos por n. A idéia é exatamente a mesma, mas neste caso temos que

nossas casas são os possíveis restos quando dividimos por n. Lembrando que o resto sempre é um número inteiro

entre 0 e n− 1, temos n possíveis restos (casas de pombo) e n+ 1 números (pombos), segue que existem dois ou mais

na mesma casa, isto é, temos, dois o mais números que deixam o mesmo resto quando divididos por n.

Exemplo 9 Seja A um subconjunto de 1, 2, . . . , 200 com 101 elementos. Então A possui dois elementos distintos

tais que um é divisível pelo outro. Provar.

Solução: Todo número inteiro positivo n pode ser escrito na forma n = 2aI, no qual a é um inteiro não-negativo e

I um número inteiro positivo ímpar. Dizemos que I é o “fator ímpar” de n. Se n for um número ímpar temos a = 0.

Se n for uma potência de 2, temos I = 1. Exemplos:

15 = 20 · 15 18 = 22 · 9 21 = 20 · 21 32 = 25 · 1

Considerando os elementos de A, o fator ímpar de cada um deles é um elemento do conjunto B = 1, 3, . . . , 199. Como

B possui 100 elementos e A possui 101 elementos, pelo Princípio da Casa dos Pombos, dois deles possuem o mesmo

fator ímpar.

Deste modo, A tem dois elementos distintos n1 = 2a1I e n2 = 2a2I, com partes ímpares iguais. Se a1 < a2, n1

divide n2; se a1 > a2, n2 divide n1.

Observe que a afirmação do problema não é verdade seA possui 100 elementos. De fato, o conjunto 101, 102, . . . , 200

possui 100 elementos e nenhum é múltiplo de outro.

Exemplo 10 Ache o número mínimo de cores necessárias para colorir os números 1, 2, . . . , 2009 de modo que não

exista uma tripla (a, b, c) de números distintos da mesma cor tais que a divide b e b divide c.

Solução: Vamos provar que o número mínimo de cores é 6.

Suponha que 5 cores sejam suficientes. Considere as onze potências de 2 menores que 2009:

1, 2, 22, · · · , 210

Pelo Princípio das Casas dos Pombos, pelo menos uma cor é utilizada 3 vezes para colorir estes onze números. De

fato, se cada cor fosse usada no máximo duas vezes, então teríamos no máximo 5× 2 = 10 números.

Se 2m < 2n < 2p são essas potências pintadas com a mesma cor, temos então 2m | 2n e 2n | 2p, donde segue que

5 cores não são suficientes.

Vamos provar que é possível colorir os números com 6 cores respeitando as condições do problema. Pintemos os

números assim:

Page 38: compilacaoModulo1Nivel2

DRAFT

38 Combinatória– Princípio da Casa dos Pombos – Professor Fábio Brochero

Números Cor

1, 2, 3 1

4, . . . , 15 2

16, . . . , 63 3

64, . . . , 255 4

256, . . . , 1023 5

1024, . . . , 2009 6

Para provar que esta coloração respeita as condições do problema, observe que dados dois números naturais

distintos m e n, tais que m | n, então n ⩾ 2m. Portanto se a, b e c são números distintos tais que a divide b e b divide

c, então c ⩾ 4a.

Suponha que existam x, y e z da mesma cor tais que x | y e y | z. Pela observação segue que z ⩾ 4x. Mas, dados

dois números x e z (x < z) pintados com a mesma cor acima, temos 1 < zx< 4, absurdo.

Exemplo 11 (OBM-1991) Mostre que existe um número da forma 1 99 . . . 9︸ ︷︷ ︸n noves

1 com mais de dois noves que é um

múltiplo de 1991.

Solução: 1 Considere os números 19991; 199991; 1999991;. . .. Pelo Princípio da Casa dos Pombos, dois desses

números deixam o mesmo resto na divisão por 1991. Subtraindo esses números, obtemos um número da forma

19 . . . 980 . . . (n noves) o qual é divisível por 1991.

Assim, como 1991 é primo com 10, o número 19 . . . 98000 (n noves) é divisível por 1991 e, portanto,

1 9 . . . 9︸ ︷︷ ︸n vezes

8000+ 1991 = 1 9 . . . 9︸ ︷︷ ︸n + 3 vezes

1

é divisível por 1991.

Exemplo 12 Demonstrar que qualquer conjunto de n inteiros tem um subconjunto não vazio cuja soma dos ele-

mentos é divisível por n.

Solução: Sejam a1, a2, . . . , an os elementos do conjunto, e definamos

S1 = a1

S2 = a1 + a2

...

Sj = a1 + a2 + · · ·+ aj

...

Sn = a1 + a2 + · · ·+ an

Se algum dos Sj é divisível por n o problema fica resolvido. Se nenhum é divisível por n, então existem dois índices j

e k com j < k tais que Sj e Sk deixam o mesmo resto quando os dividimos por n. Portanto, Sk − Sj = aj+1 + · · ·+ ak

é divisível por n.

1Solução de André Reis Leal – Olimpíadas Brasileiras de Matemática – 9a a 16a – SBM –OBM

Page 39: compilacaoModulo1Nivel2

DRAFT

OBMEP • PECI – Preparação Especial para Competições Internacionais 39

Exemplo 13 Dado um conjunto S formado por inteiros positivos distintos, nenhum dos quais é divisível por nenhum

primo maior que 12.

a. Mostrar que se S tem 33 elementos, então existem dois elementos tais que seu produto é um quadrado perfeito.

b. Mostrar que se S tem 9 elementos, então podemos escolher 4 ou menos elementos de S de forma que seu produto é

um quadrado perfeito.

Solução: Neste problema o mais importante é a paridade dos expoentes na fatoração em primos de cada número.

De fato, cada elemento de S pode ser escrito na forma 2a3b5c7d11e, onde a, b, c, d e e são inteiros não negativos, e o

produto de dois números de S também é da mesma forma. Além disso um número é um quadrado se cada um destes

expoentes é par. Por outra lado, se x = 2a13b15c17d111e1 e y = 2a23b25c27d211e2 são elementos de S, seu produto é

um quadrado perfeito se e somente se a1 tem a mesma paridade que a2, b1 tem a mesma paridade que b2, etc.

Para solucionar a primeira parte, basta classificar os números de S de acordo com paridade destes expoentes, isto

é, dois números estão na mesma classe se seus expoentes tem mesma paridade. Como temos no total 25 = 32 possíveis

classes e o conjunto S tem 33 elementos, segue pelo Princípio das Casas dos Pombos que existem dois elementos na

mesma classe, logo seu produto é um quadrado perfeito.

Para solucionar a segunda parte, considerem o conjunto de conjuntos R = x, y|x, y ∈ S. Observe que R tem(92

)= 36 elementos formado por dois números, e consideremos os 36 números obtidos a partir do produto dos

números em cada elemento de R. Pela primeira parte do problema, o produto de dois destes números é um quadrado

perfeito. Daqui, temos duas possibilidades: Ou este produto é formado por produto de 4 elementos distintos de S,

ou os dois elementos escolhidos eram da forma x, y, x, z, com x2yz igual a um quadrado, mas neste caso yz é um

também um quadrado.

Fica de exercício ao leitor mostrar que a parte (b) do problema continua valida quando o conjunto S possui 8

elementos.

Exemplo 14 Prove que dentre 52 inteiros é possível escolher dois cuja soma ou diferença é divisível por 100.

Solução: Para aplicar o Princípio das Casas dos Pombos neste problema, construíremos 51 casas da seguinte forma:

na casa j, colocamos o números que deixam resto j ou 100 − j quando dividimos por 100 onde j = 0, 1, . . . , 50. Como

temos 52 números, pelo Princípio das Casas dos Pombos, existem dois números que estão na mesma casa. Caso estes

dois números deixem o mesmo resto quando são divididos por 100, sua diferença será divisível por 100. Por outro

lado, se deixam restos distintos, sua soma será divisível por 100.

Exemplo 15

(a) Prove que em qualquer grupo de cinco números inteiros existem três cuja soma é divisível por 3.

(b) Prove que em qualquer grupo de onze números inteiros existem seis cuja soma é divisível por 6.

Solução: Para solucionar o item a. , consideremos dois possíveis situações: Entre os cinco números temos três

números que deixam restos distintos quando dividimos por 3, e o segundo caso é que isto não acontece. No primeiro

caso, basta pegar estes três números que sua soma será divisível por 3. No segundo caso, podemos distribuir os

números em duas classes, dependendo do resto que deixam quando dividimos por 3, como temos cinco números e

dois possíveis restos, segue que existem três números que deixam o mesmo resto e assim a soma destes números é

divisível por 3.

Para solucionar o item b, vejamos como transformar este problema no problema anterior. De fato, como temos 11

números, a pares e b ímpares, podemos formar ⌊a2⌋ pares de números pares e ⌊b

2⌋ pares de números ímpares. Como

ou a ou b é par, temos no total ⌊a2⌋ + ⌊b

2⌋ = ⌊a+b

2⌋ = 5 pares de números da mesma paridade. Com cada par a, b

construímos um número inteiro a+b2

, assim pelo problema anterior destes cinco números inteiros é possível escolher

Page 40: compilacaoModulo1Nivel2

DRAFT

40 Combinatória– Princípio da Casa dos Pombos – Professor Fábio Brochero

três tal que a soma é divisível por 3, e portanto a soma de seis dos números originais é divisível por 6.

Exemplo 16 (Rioplatense) Prove que dentre 101 números inteiros positivos quaisquer, é possível escolher 11 tal que

sua soma seja divisível por 11.

Solução: Observemos que se entre os 101 números temos onze números que deixam restos distintos quando

dividimos por 11, temos que a soma destes onze números é divisível por 11, assim podemos supor sem perda de

generalidade que entre os 101 podemos obter no máximo 10 restos quando dividimos por 11, o que implica pelo

Princípio das Casas dos Pombos que existem um resto que se repete onze vezes ou mais, assim somando onze números

que deixam o mesmo resto quando dividido por 11, esta soma é divisível por 11.

5.5.1

Problemas Propostos

Problema 10 Prove que existe um inteiro positivo n tal que os quatro últimos algarismos de 3n são iguais a 0001.

Problema 11 Seja k ⩾ 1 um número natural. Determine o menor número natural n com a seguinte propriedade:

Para toda escolha de n inteiros existem no mínimo dois cuja soma ou diferença é divisível por 2k+ 1.

5.6

Combinatória Aditiva

Exemplo 17 O conjunto A ⊂ N é tal que existe um elemento de A entre quaisquer 2013 inteiros consecutivos. Mostre

que existem em A quatro elementos distintos a, b, c, d tais que

a+ b = c+ d.

Solução: É claro que A possui infinitos elementos, já que possui elementos arbitrariamente grandes. Se ordenemos

os elementos de A de forma crescente: a1 < a2 < a3 < a4 < · · · , temos que todas as diferenças a2 − a1, a4 − a3,

a6 − a5, . . . são positivas e menores que 2013. Como temos infinitas diferenças, existem diferenças iguais, isto é

a2j − a2j−1 = a2i − a2i−1 com i = j e portanto a2j + a2i−1 = a2i + a2j−1, como queríamos mostrar.

Exemplo 18 Um conjunto de números é dito livre de somas se não contém dois elementos (possivelmente iguais)

cuja soma também esteja no conjunto. Seja A = 1, 2, 3, . . . , 200, 201.

(a) Encontre um subconjunto livre de somas do conjunto A com 101 elementos.

(b) Prove que todo subconjunto de A com mais de 101 elementos não é livre de somas.

Solução:

(a) O subconjunto 1, 3, 5, . . . , 201 tem 101 elementos e é livre de somas, pois a soma de quaisquer dois de seus

elementos é par.

O subconjunto 101, 102, . . . , 201 também tem 101 elementos e é livre de somas, porque a soma de quaisquer dois

de seus elementos é maior que o seu maior elemento.

(b) Vamos provar que todo subconjunto de A com 102 elementos contém três elementos x, y e z tais que x+ y = z.

Considere um subconjunto S = x1, . . . , x102, de A com x1 < x2 < · · · < x102. Observe que

1 ⩽ x2 − x1 < x3 − x1 < · · · < x102 − x1 ⩽ 200.

Page 41: compilacaoModulo1Nivel2

DRAFT

OBMEP • PECI – Preparação Especial para Competições Internacionais 41

Como A− S tem 99 elementos, nem todas essas 101 diferenças podem estar em A− S. Logo, ao menos uma delas

está em S, donde segue que existem i e j, com 1 ⩽ j < i ⩽ 101, xj = xi − x1. Portanto, xi = xj + x1 e segue que S

não é livre de somas.

5.7

Problemas Avançados

O princípio das gavetas é também aplicável à solução de algumas desigualdades. Por exemplo, o seguinte problema

proposto na IMO (Olimpíada Internacional de Matemática) de 1987.

Exemplo 19 (IMO – 1987) Sejam x1, x2, . . . , xn números reais tais que x21 + x22 + · · · + x2n = 1. Demonstrar que

para todo número natural k, k ⩾ 2, existem n inteiros a1, a2, . . . , an, não todos nulos tais que |ai| ⩽ k− 1 e que

|a1x1 + a2x2 + · · ·+ anxn| ⩽ (k− 1)

√n

kn − 1.

Solução: Primeiro vemos que a desigualdade de Cauchy-schwarz 2 nos permite afirmar que

|x1|+ |x2|+ · · ·+ |xn| ⩽√x21 + x22 + · · ·+ x2n

√n =

√n.

Daqui segue se que para qualquer conjunto b1, b2, . . . , bn de números inteiros não negativos menores ou iguais a

k− 1 teremos que

|b1x1 + b2x2 + · · ·+ bnxn| ⩽ (k− 1)(|x1|+ |x2|+ · · ·+ |xn|) ⩽ (k− 1)√n.

Dado que cada bi pode assumir k valores, então teremos kn n-uplas (b1, b2, . . . , bn) tais que |b1x1+b2x2+ · · ·+bnxn|

está entre[0, (k− 1)

√n]. Assim, se dividimos este intervalo em kn−1 intervalos de tamanho k−1

kn−1

√n, pelo princípio

das gavetas existem duas n-uplas (b1, . . . , bn) e (c1, . . . , cn), tais que b1x1 + · · ·+ bnxn e c1x1 + · · ·+ cnxn estão no

mesmo subintervalo. Portanto,

|(b1 − c1)x1 + · · ·+ (bn − cn)xn| ⩽k− 1

kn − 1

√n,

como queríamos demonstrar.

Do mesmo modo podemos resolver o seguinte

Exemplo 20 (IMO–2003) Seja A um subconjunto de S = 1, 2, . . . , 1000000 com 101 elementos. Demonstrar que

existem t1, . . . , t100 elementos de S tais que os conjuntos

Aj = x+ tj | x ∈ A, j = 1, 2, . . . , 100

sejam disjuntos dois a dois.

Solução: Necessitamos escolher ti de tal forma x+ ti = y+ tj para todo x, y ∈ D e i = j, isto é, que x−y = tj− ti.

Consideremos o conjunto D = x− y | x, y ∈ A.

Este conjunto tem no máximo 101×100+ 1 = 10101 elementos. Logo precisamos escolher 100 números ti, tal que

a diferença de dois de eles não pertença a D. Vamos a escolher os ti por indução. Primeiro pegamos um elemento

2Dados b1, b2, . . . , bn, c1, . . . , cn números reais, então

(b21 + · · · + b2n)(c21 + · · · + c2n) ⩾ (|b1c1| + · · · + |bncn|)

2

Page 42: compilacaoModulo1Nivel2

DRAFT

42 Combinatória– Princípio da Casa dos Pombos – Professor Fábio Brochero

arbitrário, e agora suponhamos que já temos escolhido t1, tk com k ⩽ 99 que cumprem as condições do problema.

Agora por cada ti escolhido existem 10101 que não podemos escolher, logo como escolhemos k existem 10101k <

999999 que não podemos escolher, assim simplesmente escolhemos uns dos que sobra.

Observação: O problema pode ser otimizado pegando D = |x − y| | x, y ∈ A que terá no máximo 101×1002

+ 1

elementos, e |ti − tj| /∈ D.

5.8

Problemas Complementares

Exemplo 21 (Torneio das Cidades) Existem 20 alunos em uma classe. Quaisquer dois deles possuem um avô em

comum. Prove que existe um avô que tem no mínimo 14 netos nesta classe.

Solução: Dizemos que uma criança é do tipo [AB] se tem como avôs a A e a B. Considere uma criança do tipo [AB].

Podem existir várias crianças deste tipo. Sabemos que toda criança tem A ou B como avô. Podemos supor que A não

é avô de todos e que B também não é avô de todos, porque neste caso o problema estaria resolvido. Portanto, existe

uma criança que não tem B como avô, e portanto ela é do tipo [AC]. Analogamente, existe uma criança que não tem

A como avô, e então, por um lado ela deve ter B como avô, e como tem um avô em comum com [AC], também deve

ter C como avô. Portanto, essa criança é do tipo [BC]. Concluímos que existem somente três tipos de crianças: [AB],

[AC] ou [AC], e somente três avôs.

Desse modo, as 20 crianças tem 40 avôs contando com multiplicidade e como 40 = 3×13+1, segue pelo Princípio

da Casa dos Pombos que existe um avô que está sendo contado com multiplicidade pelo menos 14, isto é, existe um

avô com pelo menos 14 netos na turma.

Exemplo 22 Ana escolhe vinte e cinco números diferentes dentre os números 1, 2, . . ., 50. Bruno escolhe vinte e

cinco números diferentes dentre os números 51, 52, . . ., 100. Ana e Bruno fazem essa escolha de modo que dentre todos

os números escolhidos por eles não existam dois cuja diferença seja igual a 50.

Calcule a soma de todos os números escolhidos.

Solução: Observemos que se Ana escolhe o número j, então Bruno não escolheu o número 50 + j. Assim como

Ana escolheu 25 números, temos que 25 números entre 51 e 100 que não podem ser escolhidos por Bruno. Em ou-

tras palavras, os números escolhidos por Ana determinam univocamente os números escolhidos por Bruno. Sejam

j1, j2, . . . , j25 os números escolhidos por Ana e A a soma destes números. Segue que a soma dos números escolhidos

por Bruno é

(51+ 52+ · · ·+ 100) − ((50+ j1) + (50+ j2) + · · ·+ (50+ j25))

=151 · 50

2− 50 · 25−A = 2525−A.

Portanto, a soma dos números escolhidos pelos dois é 2525.

Exemplo 23 São dados 40 pesos: 1, 2, . . . , 40 gramas. Dez pesos com massa par foram colocados sobre o prato

esquerdo da balança. Em seguida, dez pesos com massa ímpar foram colocados no prato direito da balança. Os pratos

esquerdo e direito da balança estão equilibrados agora.

Prove que sobre um prato da balança existem dois pesos cuja diferença em massa é exatamente 20 gramas.

Solução: Solução de Gabriel Ilharco.

São pesos pares os pesos 2, 4, 6, . . . , 40. Dividamo-os em dois conjuntos A e B.

A = 2, 4, 6, 8, 10, 12, 14, 16, 18, 20

Page 43: compilacaoModulo1Nivel2

DRAFT

OBMEP • PECI – Preparação Especial para Competições Internacionais 43

e

B = 22, 24, 26, 28, 30, 32, 34, 36, 38, 40.

Para não haver diferença 20 na balança de pesos pares, temos que, para qualquer peso escolhido em A, não podemos

escolher seu “correspondente em B”. Isso só é possível se fizermos da seguinte forma: 2 ou 22, 4 ou 24, 6 ou 26, . . . ,

18 ou 38, 20 ou 40. Em que o ”ou”é exclusivo, isto é, ou escolhemos um ou outro.

Desse modo, a soma dos pesos dessa balança é dada por 2+4+6+ · · ·+20+20k, em que k é inteiro e 0 ⩽ k ⩽ 10,

ou seja, o peso total é 110+ 20k.

Procedendo de modo análogo com os pesos ímpares, temos que a soma será 100 + 20k ′, com k ′ também inteiro.

Deste modo, temos que 110+ 20k = 100+ 20k ′, o que resulta em 2(k ′ − k) = 1, o que é um absurdo já que k ′ e k são

inteiros, e 2(k ′ − k) é par enquanto que 1 é ímpar.

5.9

Problemas Propostos

Problema 12 Dez amigos trocam postais, cada um deles enviando 5 postais para diferentes amigos. Prove que no

mínimo dois deles trocaram postais entre si.

Problema 13 Mostre que em uma festa com 20 pessoas sempre existem duas com a mesma quantidade de amigos

na festa.

Problema 14 Um professor e seus 30 alunos escreveram, cada um, os números de 1 a 30 em uma ordem qualquer.

A seguir, o professor comparou as seqüências. Um aluno ganha um ponto cada vez que um mesmo número aparece na

mesma posição na sua seqüência e na do professor. Ao final, observou-se que todos os alunos obtiveram quantidades

diferentes de pontos. Mostre que a seqüência de pelo menos um aluno coincidiu com a seqüência do professor.

Problema 15 (Cone Sul – 1992) Determine a quantidade de elementos que pode ter um conjunto B contido em

1, 2, . . . , n com a seguinte propriedade:

Para quaisquer a e b elementos de B, com a diferente de b, (a− b) não divide a+ b.

Problema 16 Uma senha de 7 dígitos é chamada boa se todos os dígitos são diferentes. Um cofre tem uma senha

boa. O cofre é aberto se uma senha boa for testada e se um de seus dígitos for igual ao dígito correspondente da senha

do cofre. Existe um método para abrir um cofre cuja senha não conhecemos usando menos que 7 tentativas?

Problema 17 Os números de 1 até 10 são colocados de forma arbitrária formando uma fila. Mostre que existem 4

cartas (não obrigatoriamente consecutivas) que estão em ordem crescente ou decrescente.

Problema 18 Seja A = 1, 2, . . . , 99, 100 o conjunto dos números de 1 a 100 e deste escolhemos 51 números.

Demonstrar que entre os 51 existem dois números tais que um é divisível pelo outro.

Problema 19 Seja α um número irracional. Demonstrar que para todo inteiro n existe um inteiro 0 < k ⩽ n, tal

que a diferença entre kα e seu inteiro mais próximo é menor que 1/n.

Problema 20 Se escolhem 7 pontos no interior de um retângulo de 2×3. Demonstrar que sempre é possível encon-

trar dois pontos tal que sua distancia é menor o igual a√2.

Problema 21 Se escolhem 9 pontos no interior de um quadrado de lado 1. Demonstrar que é possível escolher 3

deles de tal forma que o área do triângulo que formam é menor que 18

.

Problema 22 (OCM) Separamos o conjunto N = 1, 2, . . . como união disjunta N = L ∪ (N − L). O conjunto L é

finito, tem g elementos e se os números naturais a e b são tais que a ∈ L e b ∈ L, então a + b ∈ L. Mostre que o

maior elemento de L é menor ou igual a 2g− 1.

Problema 23 Dado um número irracional u, demonstrar que sempre é possível encontrar um número infinito de

Page 44: compilacaoModulo1Nivel2

DRAFT

44 Combinatória– Princípio da Casa dos Pombos – Professor Fábio Brochero

números racionais pq

de tal forma que ∣∣∣∣u−p

q

∣∣∣∣ < 1

q2.

Um problema mais difícil que este é demonstrar existem racionais pq

de tal forma que∣∣∣∣u−p

q

∣∣∣∣ < 1√5q2

.

Problema 24 (IMO – 1983) Seja ABC um triângulo equilátero e E o conjunto de todos os pontos contidos nos

segmentos AB,BC e CA (incluindo A,B e C). É verdade que, para toda partição de E em dois subconjuntos disjuntos,

no mínimo um dos dois subconjuntos contém os vértices de um triângulo retângulo? Justifique sua resposta.

Problema 25 (IMO – 1985) Dado um conjunto M com 1985 inteiros positivos distintos, nenhum dos quais tem

divisores primos maior do que 23, mostre que M contém um subconjunto de 4 elementos tal que seu produto é uma

quarta potência.

Problema 26 (IMO – 2001) Sejam n1, n2, . . . , nm inteiros com m ímpar. Denotemos por x = (x1, . . . , xm) uma

permutação dos inteiros 1, 2, . . . ,m, e f(x) = x1n1+ · · ·+xmnm. Demonstre que existem duas permutações a e b tais

que f(a) − f(b) é divisível por m!.

Problema 27 Demonstrar que dados 7 números reais sempre é possível escolher 2 deles, digamos a e b, tais que∣∣∣∣ a− b

1+ ab

∣∣∣∣ < 1√3.

Page 45: compilacaoModulo1Nivel2

DRAFT

..

Combinatória

. 6.

Grafos

6.1

Viajando em Pecilândia

Exemplo 1 Pecilândia é um pequeno país que possui 10 cidades e 37 estradas. Cada estrada une exatamente duas

cidades. Duas cidades são ligadas por no máximo uma estrada. Prove que é possível viajar entre duas cidade quaisquer

de Pecilândia, percorrendo uma ou mais estradas.

Solução: Vamos representar as cidades C1, C2, . . . , C10 como pontos no plano e as estradas como segmentos

unindo dois dos pontos.

..

C1

.

C2

.

C3

.

C4

.C5

.

C6

.

C7

.

C8

.

C9

. C10

Por exemplo, na figura acima, o segmento significa que existe uma estrada entre as cidades C4 e C10.

Sabemos que ao todo Pecilândia possui 37 estradas, mas não sabemos quantas estradas partem de cada cidade.

Sejam d1, d2, . . . , d10 as quantidades de estradas que partem de C1, C2, . . . , C10. Podemos supor, sem perda de

generalidade, que d1 ⩽ d2 ⩽ · · · ⩽ d10.

Quanto vale a soma d1 + d2 + · · · + d10? Como cada estrada tem duas extremidades, esta soma é o dobro da

quantidade de estradas. Deste modo,

d1 + d2 + · · ·+ d10 = 2× 37 = 74.

A soma destes dez números inteiros não negativos é igual a 74. O maior deles é no mínimo 8. De fato, se todos fosse

menores ou iguais a 7, a soma dos dez seria no máximo 70.

Portanto, d10 ⩾ 8. Se d10 = 9, o problema acabou, pois todas as cidades estariam ligadas a C10 e para viajar entre

quaisquer duas outras cidades, bastaria passar por C10.

..

C1

.

C2

.

C3

.

C4

.C5

.

C6

.

C7

.

C8

.

C9

. C10

45

Page 46: compilacaoModulo1Nivel2

DRAFT

46 Combinatória– Grafos – Professor Fábio Brochero

Se d10 = 8, então existe uma cidade X que não está ligada diretamente a C10. É possível viajar entre quaisquer duas

cidades diferentes de X passando por C10. Será que X pode ser uma cidade isolada? Se este fosse o caso, existiriam

no máximo 9× 8/2 = 36, estradas em Pecilândia, o que contradiz a hipótese do problema.

..

C1

.

C2

.

C3

.

C4

.C5

.

C6

.

C7

.

C8

.

C9

. C10

Portanto, X está ligada a pelo menos uma outra diferente de C10, digamos, Y. Então é possível viajar para X de uma

cidade qualquer diferente de C10, de X e de Y, fazendo o percurso Ci – C10 – Y – X.

× × ×

Essa estrutura de vértices e arestas que unem pares de vértices é conhecida como grafo. No nosso problema

tínhamos um grafo com 10 vértices e 37 arestas.

Dizemos que dois vértices são adjacentes ou vizinhos, se existe uma arestas que os une.

A quantidade de arestas que incide em cada vértice é o grau de um vértice. Como toda aresta incide em exatamente

em dois vértices, podemos concluir que

d1 + d2 + · · ·+ dn = 2A,

sendo A o número de arestas.

Um caminho é uma sequência de vértices consecutivos são unidos por uma aresta. Se o primeiro vértice de um

caminho é igual ao último dizemos que tal caminho é um ciclo.

Se quaisquer dois vértices do grafos pertencem a um mesmo caminho, dizemos que o grafo é conexo.

O nosso problema poderia ser enunciado então, como: Prove que todo grafo com 10 arestas e 37 vértices é conexo.

6.2

Entre Amigos

Exemplo 2 (Rioplatense – 1997) Em um grupo de pessoas, sabe-se que cada uma conhece exatamente 101 pessoas

do grupo.

(a) É possível que haja exatamente 1997 pessoas no grupo?

(b) É possível que haja exatamente 1998 pessoas no grupo?

Observação: supõe-se que se A conhece B, então B conhece A.

Solução: Vamos representar as pessoas como vértices de um grafo e ligar dois vértices por uma aresta se as pessoas

representadas por estes vértices se conhecem.

(a) Em linguagem de grafos, o problema pergunta em (a) se existe um grafo com 1997 vértices tal que todos possuem

grau 101.

Suponhamos que tal grafo exista. A soma dos graus de todos os vértices é então igual a 1997 × 101. Mas a soma

de todos os graus é igual ao dobro do número de arestas. Como 1997× 101 é ímpar, tal grafo não pode existir.

Page 47: compilacaoModulo1Nivel2

DRAFT

OBMEP • PECI – Preparação Especial para Competições Internacionais 47

(b) Neste caso queremos um grafo com 1998 vértices tal que cada um tem grau 101. O argumento usado em (a) não se

aplica, pois a soma de todos os graus é igual a 1998× 101 e a quantidade de arestas é então 999× 101.

Tal grafo existe sim. Considere as pessoas P1, P2, . . . , P1998. Suponha que elas estão ao redor de um círculo

nesta ordem e olhando para o centro. Se cada pessoa conhece as 50 pessoas mais próximas à esquerda, as 50 mais

próximas à direita e a pessoa que está diametralmente oposta, então cada uma conhecerá exatamente 101 pessoas.

Não é difícil verificar que esse exemplo realmente funciona.

O diagrama abaixo mostra uma situação análoga para 12 pessoas, cada uma conhecendo exatamente 5 outras.

Duas à direita, duas à esquerda e a pessoa mais distante.

..

P1

.

P2

.

P3

.

P4

.

P5

.P6

.

P7

.

P8

.

P9

.

P10

.

P11

. P12

× × ×

Como a soma dos graus de todos os vértices de um grafo é um número par, a quantidade de vértices de grau ímpar

tem que ser par. De fato, se a quantidade de vértices de grau ímpar fosse ímpar, a soma de todos os graus seria ímpar,

um absurdo.

Vamos mais uma vez aplicar esse resultado no problema a seguir.

Exemplo 3 Cada um dentre os 102 estudantes em uma escola é amigo de pelo menos outros 68 estudantes. Prove

que existem quatro estudantes que possuem o mesmo número de amigos.

Solução: Os estudantes são os vértices do nosso grafo e amizade entre dois estudantes é representada por uma

aresta.

O grau de cada vértice pertence ao conjunto A = 68, 69, . . . , 101 de 34 elementos. Como existem 102 vértices e

34 possíveis graus, se supusermos que não existem quatro com o mesmo grau, teremos que cada elemento de A está

associado à exatamente 3 vértices (pois 102 = 34 × 3). Como existem 17 números ímpares no conjunto A, o número

de vértices de grau ímpar do nosso grafo será 17× 3 que é um número ímpar, absurdo.

Portanto, deverão existir quatro vértices com o mesmo grau, ou seja, quatro estudantes com a mesma quantidade

de amigos.

× × ×

O nosso próximo problema é do Torneio Internacional das Cidades, uma competição que teve origem na Rússia e

da qual participam cidades dos cinco continentes. A solução abaixo foi apresentada nesta prova pelo aluno do PECI,

Victor de Oliveira Bitarães de Betim – MG.

Page 48: compilacaoModulo1Nivel2

DRAFT

48 Combinatória– Grafos – Professor Fábio Brochero

Exemplo 4 2000 pessoas registraram-se em um novo website. Cada uma destas convidou 1000 pessoas (dentre

as registradas no website) para ser sua amiga. Duas pessoas são consideradas amigas se e somente se elas enviaram

convite uma para a outra. Qual pode ser o número mínimo de pares de amigos neste website?

Solução: Podemos contar a quantidade de segmentos que podem ser formados por 2000 pontos num plano, sendo

que não existem 3 colineares. Esse número é equivalente a(2000

2

).

A quantidade total de convites que serão enviados é 2000 × 1000, pois cada pessoa envia 1000 convites. Então

temos que um par de amigos é um dos (2000

2

)=

2000× 1999

2

segmentos que é “preenchido” com 2 convites (um da primeira pessoa para a segunda e outra da segunda para a

primeira).

O mínimo de pares de amigos é obtido quando “preenchemos” todos os segmentos pelo menos uma vez, restando

assim

2000× 1000−2000× 1999

2= 1000

convites.

Como todos os segmentos já foram preenchidos uma vez, podem ser preenchidos no máximo duas vezes e resta-

ram 1000 pares de amigos.

Podemos dar um exemplo no qual há exatos 1000 pares de amigos. Chamamos as pessoas de p1, p2, . . . , p2000 e

usamos pi → pj para indicar que pi enviou um convite para pj. Então fazemos:

p1 → p2, p3, . . . , p1001

p2 → p3, p4, . . . , p1002

...

p1000 → p1001, p1002, . . . , p2000

p1001 → p1002, p1003, . . . , p1

...

p2000 → p1, p2, . . . , p1000

Pela representação acima, vemos que os pares de amigos são p1, p1001, p2, p1002, . . . , p1000, p2000 e que há exatos

1000 pares de amigos, que é, portanto o número mínimo de pares de amigos.

Exemplo 5 A população de Pecília, capital da Pecilândia, é formada por n pessoas que respeitam as seguintes leis:

(i) Dentre quaisquer três pessoas existem pelo menos duas pessoas que não se conhecem;

(ii) Quaisquer duas pessoas que não se conhecem possuem exatamente duas pessoas conhecidas em comum.

Mostre que existe um inteiro k tal que n = 1 + k(k+1)2

. Conclua que todas as pessoas têm a mesma quantidade de

conhecidos.

Solução: Seja x uma pessoa qualquer e x1, x2, . . . , xm os conhecidos de x.

Como dentre três pessoas quaisquer existem duas que não se conhecem, então quaisquer duas pessoas xi e xj

conhecidas de x não se conhecem.

Page 49: compilacaoModulo1Nivel2

DRAFT

OBMEP • PECI – Preparação Especial para Competições Internacionais 49

Assim, pela condição (ii), xi e xj possuem exatamente dois conhecidos em comum, um dos quais é x. Seja yi,j o

outro amigo comum.

É claro que x e yi,j não se conhecem porque caso contrário teríamos que quaisquer entre x, xi e yi,j se conheceriam,

assim xi e xj são os únicos amigos comuns aos dois.

Por outro lado, se y não conhece x, existem exatamente dois conhecidos comuns, que claramente estão no conjunto

x1, x2, . . . , xm. Concluímos que cada par xi, xj determina exatamente uma pessoa que x não conhece e vice-versa.

Portanto o número total de pessoas é 1+m+(m2

)= 1+ m(m+1)

2.

Agora se a pessoa z conhece k pessoas, pelo mesmo processo temos que o número total de pessoas é 1+ k(k+1)2

,

donde concluímos que m(m+1)2

= k(k+1)2

, assim k = m. Portanto, todas as pessoas conhecem exatamente a mesma

quantidade de outras em Pecilândia.

× × ×

Para terminar os exemplos resolvidos, apresentamos um problema com a solução do aluno André Macieira, de

Belo Horizonte – MG.

Exemplo 6 Cientistas estão reunidos para um congresso matemático. Sabe-se que dois cientistas com o mesmo

número de amigos não possuem amigos em comum. Se existem cientistas que se conhecem, prove que existe um cientista

que possui apenas um amigo.

Solução: Construímos um grafo como “quase” sempre, ou seja vértices representam cientistas e dois vértices estão

ligados se cientistas se conhecem.

Considere dentro deste grafo o vértice (ou um dos vértices) de maior grau e seja k este grau. Veja que existem

exatamente outros k vértices ligados ao de maior grau e estes vértices apresentam como grau um número entre 1 e k

e todos estes vértices tem grau diferente, uma vez que possuem um “amigo” em comum.

Suponha que nenhum destes vértices possui grau 1. Então teremos k − 1 opções de escolha para o grau destes

vértices, e pelo Princípio das Casas dos Pombos haverá dois com mesmo grau, o que nos leva a uma contradição, pois

ambos possuem um amigo comum. Portanto existe um vértice de grau 1, como queríamos demonstrar.

× × ×

Um grafo é dito completo quando quaisquer dois vértices são ligados por uma aresta. O grafo completo com n

vértices é denotado por Kn.

..

K3

..

K4

..

K5

..

K6

No próximo exemplo mostraremos que todo grafo com 2n vértices e mais de n2 arestas possui um K3.

6.3

Triângulos em Grafos

Exemplo 7

(a) Considere um grafo com 2n vértices, n > 1 e pelo menos n2 + 1 arestas. Mostre que existem três vértices determi-

nando um triângulo.

(b) Mostre que para cada natural n existe um grafo com 2n vértices e n2 arestas sem um triângulo.

Page 50: compilacaoModulo1Nivel2

DRAFT

50 Combinatória– Grafos – Professor Fábio Brochero

Solução:

(a) Para resolver esta parte vamos utilizar o princípio de indução finita.

• Para n = 2, é fácil perceber que qualquer grafo com 4 vértices e 5 arestas possui um triângulo.

• Vamos admitir todo grafo com 2n vértices e pelo menos n2 + 1 arestas possui um triângulo.

• Considere um grafo com 2(n+ 1) vértices e pelo menos (n+ 1)2 + 1 arestas. Vamos provar que pelo menos um

triângulo é formado. Suponha que nenhum triângulo seja formado.

Considere dois vértices A e B conectados por uma aresta.

.......

A

.

B

Se o subgrafo determinado pelos outros 2n vértices tiver pelo menos n2 + 1 arestas, pela hipótese de indução

teremos um triângulo. Admita, então, que este subgrafo de 2n vértices possui no máximo n2 arestas.

Cada um dos 2n pontos é vizinho de no máximo um dentre A e B porque se for vizinhos de ambos encontramos

um triângulo. Deste modo utilizamos no máximo n2 + 2n + 1 arestas: (n2 no subgrafo com 2n vértices, 2n

ligando A e B ao subgrafo e 1 para ligar a A a B). Mas n2 + 2n+ 1 < (n+ 1)2 + 1. Absurdo! Então, pelo menos

um triângulo é formado e o resultado segue por indução.

(b) Para a parte (b) basta dividir os vértices em dois grupos de n e ligar cada vértice de um grupo a todos os vértices

do outro. Deste modo, obtemos n×n = n2 arestas e nenhum triângulo é formado. De fato, se considerarmos três

vértices quaisquer, pelo menos dois deles estarão no mesmo grupo e portanto não são adjacentes.

.

Exemplo 8 (Rioplatense) Patrícia desenhou em uma folha de papel vários pontos ao redor de uma circunferência.

Em seguida traçou alguns segmentos com extremidades nestes pontos. Ao final, observou que em sua figura partiam

pelo menos três segmentos de cada ponto e que não existiam triângulos nem quadriláteros com vértices nos pontos

desenhados.

Determine o menor número possível de pontos desenhados por Patrícia e uma possível representação dos segmentos

traçados.

Solução: Vamos provar que o menor número possível de pontos é 10.

Considere um ponto qualquer A. Deste devem sair no mínimo 3 segmentos, para três outros pontos. Estes três

pontos não podem estar ligados por um segmento, pois não há triângulos na figura de Patrícia. Portanto, de cada um

desses três pontos devem sair no mínimo 2 outros segmentos. As extremidades desses segmentos devem ser todas

distintas, pois caso contrário haveria a formação de um quadrilátero. Segue que o número de pontos é no mínimo

1+ 3+ 3× 2 = 10.

Page 51: compilacaoModulo1Nivel2

DRAFT

OBMEP • PECI – Preparação Especial para Competições Internacionais 51

............A

Um exemplo com 10 pontos é mostrado na figura.

...........

6.4

Problemas Propostos

Problema 1

(a) Mostre que os números de 1 a 16 podem ser escritos numa reta, de tal modo que a soma de quaisquer dois

números vizinhos seja um quadrado perfeito.

(b) Mostre que os números de 1 a 16 não podem ser escritos ao redor de uma circunferência, de tal modo que a

soma de quaisquer dois números vizinhos seja um quadrado perfeito.

Problema 2 (OBM – 1997) A professora de Matemática propôs o seguinte problema para seus alunos:

“Marquem 6 pontos sobre uma circunferência. Eu quero que vocês pintem o maior número de cordas determinadas

por estes pontos, de modo que não existam quatro dos pontos sobre a circunferência determinando um quadrilátero

com todos os lados e diagonais coloridos.”

(a) Edmilson encontrou uma solução correta colorindo 12 cordas. Exiba uma maneira de como fazer isto.

(b) Gustavo afirmou ter encontrado uma solução na qual pintara 13 cordas. Mostre que a solução de Gustavo não está

correta.

Problema 3 (USAMO – 1982) Ema festa com 1982 pessoas, em qualquer grupo de quatro deles existe no mínimo

uma pessoa que conhece as outras três. Qual é o número mínimo de pessoas na festa que conhecem todas as demais?

(Assuma que se A conhece B, então B conhece A)

Problema 4 (Torneio das Cidades) Em Shvambrania existem n cidades, duas quaisquer ligadas por uma estrada.

Estas estradas não se intersectam. Se necessário, alguma delas passa sobre ou sob outra por meio de pontes. Um

mágico mau estabeleceu regras de mão - única nas estradas de tal modo que se alguém sai de uma certa cidade ele

não conseguirá voltar. Prove que

Page 52: compilacaoModulo1Nivel2

DRAFT

52 Combinatória– Grafos – Professor Fábio Brochero

(a) É possível estabelecer tais regras

(b) Existe uma cidade a partir da qual podemos chegar a qualquer outra, e existe uma cidade que não podemos sair.

(c) O mágico pode realizar seu objetivo de n! maneiras.

Problema 5 Em um país há 1995 cidades. Duas quaisquer delas se comunicam diretamente, por via aérea ou

terrestre (mas nunca por ambos os meios). Há também duas cidades X e Y tais que não é possível viajar de uma à

outra, mesmo com paradas em outras cidades, só por via terrestre. Prove que, dadas duas cidades quaisquer das 1995,

é possível ir de uma à outra por via aérea, fazendo no máximo uma escala (as rodovias são de mão dupla e os vôos em

ambos os sentidos).

Problema 6 (Hungria – 1977) Três escolas possuem cada uma n estudantes. Cada estudante conhece no total n+1

estudantes das outras duas escolas. Prove que existem três estudantes das diferentes escolas que se conhecem.

Problema 7 A cintura de um grafo é o tamanho do menor ciclo no grafo. Seja G um grafo com cintura 5 para o

qual todos os vértices tem grau maior ou igual a d. Mostre que G tem no mínimo d2 + 1 vértices.

Problema 8 (Cone Sul – 1998) O prefeito de uma cidade deseja estabelecer um sistema de transportes com pelo

menos 1 linha de ônibus, no qual:

(i) Cada linha passe por exatamente 3 paradas.

(ii) Cada duas linhas distintas tenham exatamente uma parada em comum.

(iii) Para cada duas paradas distintas exista exatamente uma linha que passe por ambas.

Determine o número de paradas de ônibus da cidade.

Page 53: compilacaoModulo1Nivel2

DRAFT

PARTE III

Teoria dos NúmerosProfessor Sávio Ribas

..Assuntos do Módulo: Divisibilidade; Indução; Algoritmo de

Euclides.

.

Pré-requisitos: Capítulos 1, 2 e 3 da Apostila 1 do PIC.

Page 54: compilacaoModulo1Nivel2

DRAFT

..

Números

. 7.

Ferramentas Básicas

7.1

Aquecendo os motores

7.1.1

Critérios de divisibilidade

• 2, se for par.

• 4, se o número formado pelos dois últimos algarismos da direita for divisível por 4.

• 8, se o número formado pelos três últimos algarismos da direita for divisível por 8.

• 3, se a soma dos seus algarismos for divisível por 3.

• 9, se a soma dos seus algarismos for divisível por 9.

• 5, se terminar em 0 ou 5.

• 10, se terminar em 0.

• 25, se terminar em 00, 25, 50 ou 75.

• 11, se a soma dos algarismos de ordem ímpar menos a soma dos algarismos de ordem par for um número divisível

por 11.

• 6, 15, 22, . . . , se for divisível simultaneamente pelos seus dois fatores primos.

• 12, se for divisível simultaneamente por 3 e 4.

7.1.2

Exercícios para esquentar

Problema 1 (OMM 2005) Mostre que todos os números inteiros de 1 a 100 que não são divisíveis por 2, 3, 5 nem 7

são primos.

Problema 2 Quantos divisores tem o número 72? Qual é a soma desses divisores?

Problema 3 Demonstre que a equação 1a+ 1

b+ 1

c+ 1

d+ 1

e+ 1

f= 1 não tem solução nos inteiros ímpares.

Problema 4 Os alunos da turma de Pedro praticam a soma e a multiplicação de números inteiros. A professora

escreve os números de 1 a 9 em nove fichas, uma para cada número, e as coloca em uma urna. Pedro retira três fichas

e deve calcular a soma e o produto dos três números correspondentes. Ana e Julián fazem o mesmo, esvaziando a

urna. Pedro informa à professora que retirou três números consecutivos cujo produto é 5 vezes a soma. Ana informa

que não tem nenhum primo, mas sim dois consecutivos e que o produto desses três números é 4 vezes a soma dos

mesmos. Quais números retirou Julián?

54

Page 55: compilacaoModulo1Nivel2

DRAFT

OBMEP • PECI – Preparação Especial para Competições Internacionais 55

Problema 5 (Olimpíada de Maio) Na minha calculadora, uma das teclas de 1 a 9 está com defeito. Ao pressioná-

la, aparece na tela um dígito entre 1 e 9 que não é o correspondente. Quando tentei escrever o número 987654321

apareceu na tela um número divisível por 11 e que deixa resto 3 ao ser dividido por 9. Qual é a tecla defeituosa? Qual

é o número que apareceu na tela?

7.2

Ferramentas preliminares

7.2.1

Princípio da Indução Finita - 1ª forma

Seja P(n) uma propriedade do número natural n. Uma maneira de provar que P(n) é verdadeira para todo natural

n ⩾ n0 é utilizar o Princípio da Indução Finita (PIF), que é um dos axiomas que caracterizam o conjunto dos números

naturais. O PIF consiste em verificar duas coisas:

1. (Base da Indução) P(n0) é verdadeira;

2. (Passo Indutivo) Se P(n) é verdadeira para algum número natural n ⩾ n0 então P(n+ 1) também é verdadeira.

Na base da indução, verificamos que a propriedade é válida para um valor inicial n = n0. O passo indutivo consiste

em mostrar como utilizar a validade da propriedade para um dado n (a chamada hipótese da indução) para provar a

validade da mesma propriedade para o seu consecutivo n+ 1.

Exemplo 7.2.1. Para todo inteiro positivo n temos:

1+ 2+ · · ·+ n =n(n+ 1)

2. (7.3)

De fato, para n = 1 temos 1 = 1(1+1)2

, que é verdade (essa é a base!). Suponha que a iguadade acima seja válida para

algum inteiro positivo n = k, ou seja, suponha que temos:

1+ 2+ · · ·+ k =k(k+ 1)

2(essa é a hipótese!).

Somando k+ 1 a ambos os lados da igualdade acima (esse é o passo!) obtemos:

(1+ 2+ · · ·+ k) + (k+ 1) =k(k+ 1)

2+ (k+ 1) =

(k+ 1)(k+ 2)

2,

que é justamente a equação 7.3 para n = k+ 1. Logo, pelo PIF, a igualdade vale para todo n ⩾ 1.

7.2.2

Princípio da Indução Finita - 2ª forma

Uma variação do PIF consiste em diferenciar um pouco a hipótese e mostrar que:

1. P(n0) é verdadeira;

2. se P(k) é verdadeira para todo k satisfazendo n0 ⩽ k ⩽ n então P(n+ 1) é verdadeira;

então podemos concluir que a propriedade P é válida para todo natural n ⩾ n0.

Exemplo 7.2.2. A sequência de Fibonacci Fn é definida recursivamente por F0 = 0, F1 = 1 e se n ⩾ 2 então Fn =

Fn−1 + Fn−2. Podemos escrever Fn em função apenas de n como:

Fn =αn − βn

α− β, (7.4)

Page 56: compilacaoModulo1Nivel2

DRAFT

56 Teoria dos Números– Ferramentas Básicas – Professor Sávio Ribas

onde α = 1+√5

2e β = 1−

√5

2são as raízes de x2 = x+ 1.

De fato, para n = 0 e n = 1 em 7.4 temos F0 = α0−β0

α−β= 0 e F1 = α−β

α−β= 1 (base). Seja n ⩾ 1 e suponha que a

igualdade 7.4 é válida para todo 0 ⩽ k ⩽ n. Temos:

Fn+1 = Fn + Fn−1 =αn − βn

α− β+

αn−1 − βn−1

α− β=

(αn + αn−1) − (βn + βn−1)

α− β=

αn+1 − βn+1

α− β,

que é justamente a equação 7.4 para n+ 1, uma vez que x2 = x+ 1 implica xn+1 = xn + xn−1.

No exemplo acima, observe que o passo indutivo utiliza os valores de dois termos anteriores da sequência de

Fibonacci, logo precisamos verificar a base de indução para os dois primeiros termos F0 e F1 e não apenas para o

primeiro.

7.2.3

Princípio da Boa Ordenação

O Princípio da Boa Ordenação (PBO) é equivalente ao PIF. Ele diz que todo subconjunto não vazio dos números naturais

possui um elemento mínimo.

Exemplo 7.2.3. Toda função f : N → N não-crescente é constante a partir de um certo número natural. De fato, seja

A ⊂ N a imagem de f. Pelo PBO, A possui um elemento mínimo a0. Seja n0 um natural tal que f(n0) = a0. Como f é

não-crescente, f(n) ⩽ f(n0) = a0 para todo n ⩾ n0. Por outro lado, f(n) ⩾ a0. Logo, f(n) = a0 para todo n ⩾ n0.

7.2.4

Princípio da Casa dos Pombos

O Princípio da Casa dos Pombos (PCP) diz que se n+ 1 pombos forem alocados em n casas então ao menos uma casa

conterá 2 pombos. A demonstração desse fato é por absurdo: suponha que cada casa contenha no máximo um pombo,

então o número de pombos é no máximo n, contradição.

Podemos generalizar o PCP da seguinte forma: se mais do que kn pombos forem alocados em n casas então ao

menos uma casa conterá mais que k pombos, e a demonstração é a mesma.

Exemplo 7.2.4. Ao escolher 51 números do conjunto A = 1, 2, . . . , 99, 100 sempre existem 2 que são primos entre si.

De fato, escreva A =∪50

n=1 An, onde An = 2n − 1, 2n. Como são 50 conjuntos menores e 51 números escolhidos de

A, algum Aj foi todo escolhido e portanto 2j− 1 e 2j satisfazem o que queríamos pois mdc(2j− 1, 2j) = 1.

7.2.5

Exercícios de fixação

1. Demonstrar que:

(a) 12 + 22 + · · ·+ n2 = n(n+1)(2n+1)6

.

(b) 13 + 23 + · · ·+ n3 = (1+ 2+ · · ·+ n)2.

(c) 6 divide n3 − n para todo n inteiro positivo.

(d) 3 divide 2n + 1 para todo n ímpar.

(e) 1+ q+ q2 + · · ·+ qn−1 = qn−1q−1

para todo n natural.

(f) F1 + F2 + F3 + · · ·+ Fn = Fn+2 − 1 para todo n inteiro positivo, onde (Fn) é a sequência de Fibonacci dada

por F1 = F2 = 1 e Fn+2 = Fn+1 + Fn(n ⩾ 1).

2. Prove que toda sequência com n2 + 1 números reais contém ou uma subsequência crescente de tamanho n+ 1

ou uma subsequência decrescente de tamanho n + 1. Além disso, mostre que isso pode ser evitado com n2

números reais.

Page 57: compilacaoModulo1Nivel2

DRAFT

OBMEP • PECI – Preparação Especial para Competições Internacionais 57

3. Mostre que os princípios da indução finita e da boa ordenação são equivalentes. Dica: mostre que: PIF (1ª forma)

=⇒ PIF (2ª forma) =⇒ PBO =⇒ PIF (1ª forma).

7.3

Divisibilidade

7.3.1

Múltiplos e divisores

Dados dois inteiros d e a, dizemos que d divide a ou que d é divisor de a ou que a é múltiplo de d se existir q ∈ Z tal

que a = qd. No caso, denotamos d | a. Caso contrário, d | a.

Exemplo 7.3.1. −5 | 10 (tomando q = −2), mas 10 | 5.

Lema 7.3.2. Sejam a, b, c, d inteiros. Temos:

1. Se d | a e d | b então d | ax+ by para todos x, y ∈ Z.

2. Se d | a então a = 0 ou | d |⩽| a |.

3. Se a | b e b | c então a | c.

Demonstração. Demonstração: Exercício. Dica: use a definição de “ser divisível”.

Definição 7.3.3. Dados a, b ∈ Z, não ambos nulos, definimos o máximo divisor comum como ... Adivinha!? O maior

número inteiro que divide a e b. Denotamos esse número por mdc(a, b) ou (a, b). Quando (a, b) = 1 dizemos que a e

b são primos entre si.

Definição 7.3.4. Analogamente, definimos mínimo múltiplo comum como o menor número inteiro positivo que é múl-

tiplo tanto de a quanto de b. Denotamos esse número por mmc(a, b).

7.3.2

O algoritmo de Euclides

Existe um método para calcular o mdc de dois números, que é chamado Algoritmo de Euclides. Vamos apresentá-lo a

seguir.

Definição 7.3.5. Seja x ∈ R. Definimos o piso de x, ⌊x⌋, como sendo o único k ∈ Z tal que k ⩽ x < k + 1 (ou seja, é o

maior inteiro menor ou igual a x). Definimos também o teto de x, ⌈x⌉, como sendo o único k ∈ Z tal que k− 1 < x ⩽ k

(ou seja, é o menor inteiro maior ou igual a x).

Exemplo 1

• ⌊5⌋ = ⌈5⌉ = 5;

• ⌊−3⌋ = ⌈−3⌉ = −3;

• ⌊−2, 5⌋ = −3 e ⌈−2, 5⌉ = −2;

• ⌊π⌋ = 3 e ⌈π⌉ = 4.

Proposição 7.3.6 (divisão de Euclides). Dados a, b ∈ Z com b = 0, existem q, r ∈ Z com a = bq+ r e 0 ⩽ r < |b|. Além

disso, q, r são únicos.

Dizemos que o q nas condições acima é o quociente da divisão de a por b e r é o resto.

Demonstração. Basta tomar:

Page 58: compilacaoModulo1Nivel2

DRAFT

58 Teoria dos Números– Ferramentas Básicas – Professor Sávio Ribas

q =

⌊a/b⌋, se b>0

⌈a/b⌉, se b<0e r = a− bq.

É fácil verificar que 0 ⩽ r < |b|. Além diso, se a = bq1 + r1 = bq2 + r2 então r2 − r1 = b(q1 − q2) é múltiplo de b

com |r2 − r1| < |b|, logo r2 − r1 = 0 =⇒ r2 = r1 =⇒ q2 = q1, que prova a unicidade.

Lema 7.3.7 (Euclides). Se a = bq+ r então mdc(a, b) = mdc(b, r).

Demonstração. Demonstração: Exercício. Dica: mostre que todos os divisores comuns de a e b também são divisores

comuns de r e todos divisores comuns de b e r também são divisores de a.

Note que esse lema vale mesmo que não tenhamos 0 ⩽ r < |b|. O algoritmo de Euclides consiste na aplicação

sucessiva do lema acima, onde q e r são o quociente e o resto na divisão de a por b. Como os restos formam uma

sequência estritamente decrescente de inteiros, o algoritmo termina quando atingirmos o 0.

Exemplo 7.3.8. Vamos calcular mdc(36, 28). Temos:

36 = 28× 1+ 8

28 = 8× 3+ 4

8 = 4× 2+ 0

Logo, mdc(36, 28) = mdc(28, 8) = mdc(8, 4) = mdc(4, 0) = 4.

Teorema 7.3.9 (Bézout). Sejam a, b ∈ Z. Existem x, y ∈ Z com ax+ by = mdc(a, b).

Demonstração. Seja I(a, b) := ax+by; x, y ∈ Z. Seja d = ax0+by0 o menor elemento positivo de I(a, b), que existe

(pense a respeito!). Afirmamos que d divide todos os elementos de I(a, b). De fato, dado m = ax+by ∈ I(a, b), sejam

q, r o quociente e o resto na divisão euclidiana de m por d. Temos:

r = m− dq = a(x− qx0) + b(y− qy0) ∈ I(a, b).

Como r < d e d é o menor elemento positivo de I(a, b), segue que r = 0 e portanto d | m.

Em particular, como a, b ∈ I(a, b) temos d | a e d | b, logo d ⩽ mdc(a, b). Note ainda que se c | a e c | b então

c | ax0 + by0 ⇐⇒ c | d. Tomando c = mdc(a, b) temos que mdc(a, b) | d. Logo, d = mdc(a, b).

Corolário 7.3.10. Se c ∈ Z é tal que c | a e c | b então c | mdc(a, b).

Demonstração. Exercício.

Proposição 7.3.11. Se mdc(a, b) = 1 e a | bc então a | c.

Demonstração. Pelo Teorema de Bézout, existem x, y ∈ Z tais que ax+ by = 1 =⇒ a.cx+ (bc)y = c. Notemos que a

divide o lado esquerdo, logo deve dividir o direito.

Corolário 7.3.12. Sejam p um número primo e a1, a2, . . . , am ∈ Z. Se p | a1a2 . . . am então p | aj para algum

j, 1 ⩽ j ⩽ m.

Demonstração. Exercício.

7.3.3

O Teorema Fundamental da Aritmética

Teorema 7.3.13. Seja n ⩾ 2 um número natural. Podemos escrever n de forma única como um produto n = p1p2 . . . pm

onde m ∈ N∗ e p1 ⩽ p2 ⩽ . . . ⩽ pm são primos.

Page 59: compilacaoModulo1Nivel2

DRAFT

OBMEP • PECI – Preparação Especial para Competições Internacionais 59

Demonstração. A existência segue pelo PBO. De fato, seja A ⊂ N− 0, 1 o conjunto dos números que não podem ser

escritos como produto de primos. Note que 2 ∈ A. Suponha que A = ∅ =⇒ A possui um menor elemento n. É claro

que n não pode ser primo. Assim, n = ab, com 1 < a, b < n e portanto a, b ∈ A. Logo, n é o produto dos primos

de a pelos primos de b, absurdo pois n não possui fatoração em primos. Sendo assim, A = ∅ e todos os números

naturais maiores que 1 possuem fatoração em primos.

Alternativamente, poderíamos provar a existência da fatoração usando a 2ª forma do PIF: se n é primo não há o

que fazer (basta tomar m = 1 e p1 = n). Se n é composto podemos escrever n = ab, a, b ∈ N, 1 < a < n, 1 < b < n.

Por hipótese, a e b se decompõe como produto de primos. Juntando as fatorações de a e b e reordenando os fatores

obtemos uma fatoração de n.

Quanto à unicidade, também seguiremos pelo PIF (1ª forma). Suponha n = p1 . . . pm = q1 . . . qk, com p1 ⩽. . . ⩽ pm e q1 ⩽ . . . ⩽ qk. Como p1 | q1 . . . qk temos p1 | qi para algum i e como qi é primo, p1 = qi e p1 ⩾ q1.

Analogamente, q1 ⩾ p1, logo p1 = q1. Por hipótese:

n

p1

= p2 . . . pm = q2 . . . qk

admite fatoração única, logo m = k e pi = qi para todo i.

Outras formas de escrever a fatoração são n = pe1

1 pe2

2 . . . pemm com p1 ⩽ p2 ⩽ . . . ⩽ pm e n = 2e23e35e5 . . . pep . . .

onde apenas um número finito de expoentes são positivos.

A partir do Teorema Fundamental da Aritmética podemos mostrar que todo número inteiro positivo pode ser

escrito como ab2, onde a ∈ N é livre de quadrados (isto é, não existe p primo tal que p2 | a) e b ∈ N.

Teorema 7.3.14. Existem infinitos primos.

Demonstração. Exercício. Dica: suponha que existem finitos e considere o número (produto de todos os primos)+1.

Observemos que isso não prova que p1 . . . pm + 1 é primo para todo conjunto finito de primos. Contra-exemplo:

2× 3× 5× 7× 11× 13+ 1 = 30031 = 59× 509.

É interessante notar que existem cadeias arbitrariamente grandes de números compostos consecutivos. Por exem-

plo, a sequência:

(k+ 1)! + 2, (k+ 1)! + 3, (k+ 1)! + 4, . . . , (k+ 1)! + (k+ 1)

possui k termos, nenhum dos quais é primo.

Novamente, mdc e mmc

Podemos calcular o mdc e o mmc entre dois números de outra forma:

• mdc: fatoramos os dois números e tomamos os fatores comuns com os menores expoentes;

• mmc: fatoramos os dois números e tomamos os fatores que aparecem em alguma das fatorações com os maiores

expoentes.

Exemplo 2 Vamos calcular mdc(36, 14) e mmc(36, 14). Temos 36 = 22 × 32 e 14 = 2× 7. Logo, mdc(36, 14) = 2 e

mmc(36, 14) = 22 × 32 × 7 = 252.

Proposição 7.3.15. 1. Se a | c então mdc(a, b) = mdc(c, b).

2. Se mdc(a, b) = 1 então mdc(ac, b) = mdc(c, b).

3. Se mdc(a,n) = mdc(b, n) = 1 então mdc(ab, n) = 1.

Demonstração. Demonstração: Exercício. Dica: use o Teorema Fundamental da Aritmética.

Page 60: compilacaoModulo1Nivel2

DRAFT

60 Teoria dos Números– Ferramentas Básicas – Professor Sávio Ribas

7.4

Problemas Propostos

Problema 6 Uma urna contém 101 bolinhas. Paulo e Robério disputam um jogo, jogando alternadamente um de

cada vez (Paulo começa), eles retiram no mínimo 1 e no máximo 10 bolinhas da urna. Quando a urna esvaziar, eles

irão contar a quantidade de bolinhas que cada um retirou da urna. Se os números forem primos entre si, Paulo ganha.

Caso contrário, Robério ganha. Quem irá vencer esse jogo e qual é a estratégia vencedora?

Problema 7 Seja n = pe1

1 . . . pemm a fatoração de n em potências de primos distintos pi. Mostre que n possui

(e1 + 1) . . . (em + 1) divisores.

Problema 8 Sejam n = pe1

1 . . . pemm a fatoração de n em potências de primos distintos pi e σ(n) a soma dos seus

divisores. Mostre que:

σ(n) =pe1+11 − 1

p1 − 1. . .

pem+1m − 1

pm − 1.

Problema 9 Seja p um primo e α a maior potência de p que divide n!. Mostre que:

α =

⌊n

p

⌋+

⌊n

p2

⌋+

⌊n

p3

⌋+ . . .

Problema 10 Determine com quantos zeros termina 2013!.

Problema 11 Sejam m = n números naturais. Demonstre que:

mdc(a2m

+ 1, a2n

+ 1) =

1, se a é par,

2, se a é ímpar.

Dica: suponha m > n e fatore a2m

− 1.

Problema 12 Demonstre que mdc(2a − 1, 2b − 1) = 2mdc(a,b) − 1 para todos a, b ∈ N.

Problema 13 Mostre que existe um inteiro positivo a tal que a29−1a−1

tem pelo menos 2007 fatores primos distintos.

Problema 14 Sejam a, b, c ∈ Z. Mostre que a equação ax + by = c tem solução inteira em x e y se, e somente se,

mdc(a, b) | c. No caso de admitir solução, determine todas.

Problema 15 Sejam a, b inteiros positivos com mdc(a, b) = 1. Mostre que para todo c ∈ Z com c > ab− a− b a

equação ax+ by = c admite soluções inteiras com x, y ⩾ 0.

Problema 16 Sejam an = 100+ n2 e dn = mdc(an, an+1). Calcule dn.

Problema 17 Sejam a, b ∈ N∗. Mostre que mdc(a, b).mmc(a, b) = ab.

Problema 18 Sejam m,n inteiros positivos tais que mdc(m,n) + mmc(m,n) = m + n. Prove que um dos dois

números é divisível pelo outro.

Problema 19 Sejam a e b números inteiros positivos primos entre si. Suponha que o produto ab é um quadrado

perfeito. Mostre que a e b são quadrados perfeitos.

Problema 20 (Ibero 1987) A sequência pn é definida da seguinte forma:

• p1 = 2;

Page 61: compilacaoModulo1Nivel2

DRAFT

OBMEP • PECI – Preparação Especial para Competições Internacionais 61

• Para todo n ⩾ 2, pn é o maior divisor primo da expressão

p1p2p3 . . . pn−1 + 1.

Demonstrar que pn é diferente de 5.

Problema 21 (IMO 1959) Mostre que a fração21n+ 4

14n+ 3é irredutível para todo n natural.

Problema 22 Determine todas as soluções de a2 + b2 = c2 com a, b, c ∈ Z.

Problema 23 Determine todas as soluções de a2 + b2 = 2c2 com a, b, c ∈ Z.

Problema 24 Mostre que:

1. 215 − 1 e 210 + 1 são primos entre si;

2. 232 + 1 e 24 + 1 são primos entre si;

3. se m | a− b então m | ak − bk para todo natural k;

4. se k é natural ímpar então a+ b | ak + bk.

Problema 25 Encontre todos os inteiros positivos tais que:

1. n+ 1 | n3 − 1.

2. 2n2 + 1 | n3 + 9n− 17.

3.1

a+

1

b+

1

c= 1

Problema 26 Mostre que se n é um número natural composto então n é divisível por um primo p com p ⩽ ⌊√n⌋.

Problema 27 (OBM 1988) Determine todos os primos que são a soma e a diferença de dois primos.

Problema 28 (OBM 1990) Mostre que a equação x3 + 1990y3 = z4 tem infinitas soluções inteiras com x, y, z > 0.

Problema 29 (OBM 1991) Mostre que existe um número da forma:

1 99 . . . 9︸ ︷︷ ︸n noves

1

com mais de dois noves que é múltiplo de 1991.

Problema 30 (OBM 1992) Prove que existe um natural n tal que a expansão decimal de n1992 começa com 1992

algarismos iguais a 1.

Problema 31 (OBM 1992) Seja d(n) o número de divisores positivos de n. Prove que:

n

(1

2+

1

3+ · · ·+ 1

n

)⩽

n∑k=1

d(k) ⩽ n

(1+

1

2+

1

3+ · · ·+ 1

n

)

Page 62: compilacaoModulo1Nivel2

DRAFT

PARTE IV

GeometriaProfessor Paulo Rodrigues

..Assuntos do Módulo: Ângulos, Base Média, Medianas, Bari-

centro

.

Pré-requisitos: Ângulos, Congruência de Triângulos, Para-

lelismo. Será útil conhecimento de conteúdos da apostila 8 do

PIC – Uma Introdução às Construções Geométricas.

Page 63: compilacaoModulo1Nivel2

DRAFT

..

Geometria

. 8.

Problemas de Aquecimento

Problema 1 Prove que a medida da mediana AM do

triângulo ABC é maior que (AB+AC− BC)/2.

Problema 2 DEFG é um quadrado no exterior do pen-

tágono regular ABCDE. Quanto mede o ângulo EAF?

.........A

.

B

.

C

.

E

.D

.

G

.

F

Problema 3 Construímos dois triângulos equilateros:

ABE interno e BFC externo ao quadrado ABCD. Prove

que os pontos D, E e F se localizam na mesma reta.

........A.

B.

C

.

D

.

E

.

F

Problema 4 Demonstre que, num quadrilátero qual-

quer, os pontos médios dos lados são vértices de um pa-

ralelogramo.

Problema 5 O ponto O é a intersecção das diagonais

do trapézio ABCD. São conhecidas as seguintes áreas:

[AOB] = 2 e [COD] = 8. Determine a área do trapézio.

Problema 6 Os catetos de um triângulo retângulo me-

dem 7√2 e 42

√2. Calcule o comprimento da bissetriz do

ângulo reto.

Problema 7 ABCD é um retângulo, AD = 5 e CD = 3.

........

A

.

D

.

P

.

N

.B.

C

Se BN é perpendicular a AP, calcular AP × BN.

Problema 8 No triângulo ABC, AB = 13, BC = 14 e

CA = 15. Além disso, M é o ponto médio do lado AB e H

o pé da altura relativa ao lado BC. Determine as medidas

dos segmentos HM, AH, BH e CH.

.......B.

H.

C.

A

.

M

Problema 9 As circunferências C1 e C2 são tangentes a

reta ℓ nos pontos A e B e tangentes entre si no ponto C.

Prove que o triângulo ABC é retângulo.

.......A

.B

.

O1

.

O2

.

C

...ℓ

Problema 10 Um quadrado ABCD possui lado 40cm.

Uma circunferência contém os vértices A e B e é tangente

63

Page 64: compilacaoModulo1Nivel2

DRAFT

64 Geometria– Problemas de Aquecimento – Professor Paulo Rodrigues

ao lado CD. Determine a medida do raio desta circunfe-

rência.

........C

.D.

M.

B

.

A

Problema 11 A figura abaixo é formada por arcos de

circunferências. Os centros destas são os vértices de um

quadrado e todos os arcos passam pelo centro do qua-

drado. Calcule a área sombreada, sabendo que o lado do

quadrado mede 1 cm.

.

Problema 12 Uma circunferência de raio r está inscrita

em um setor circular de raio R. O comprimento da corda

do setor é 2a.

....... 2a.

B

.

A

.

R

Prove que1

r=

1

R+

1

a.

Problema 13 As circunferências de centros A e B tem

raios 3 e 8, respectivamente. Uma tangente comum in-

terna interscta as circunferências em C e D, respectiva-

mente. As retas AB e CD intersectam-se em E e AE = 5.

.......A.

B.

C

.

D

. E

Qual a medida de CD?

Problema 14 Os vértices de um triângulo de lados 3,

4 e 5 são centros de três círculos mutuamente tangentes,

como mostra a figura.Calcule a soma das áreas dos três

círculos.

.....B.

C.

A

.5

.

3

.

4

Problema 15 O triângulo retângulo ABC tem hipote-

nusa AC e contém um ponto P para o qual PA = 10,

PB = 6 e APB = BPC = CPA.

......B.

C.

A

.

P

Determine a medida do segmento PC.

Problema 16 Como mostra a figura abaixo o ABC

está dividido em seis triângulos por retas que passam

pelo vértice e por um ponto comum no interior do tri-

ângulo. As áreas de quatro destes triângulos estão indi-

cadas. Determine a área do ABC.

........A

.B

.P.

C

.

M

.

N

.40

.30

.

84

.

35

Page 65: compilacaoModulo1Nivel2

DRAFT

..

Geometria

. 9.

Bases Médias, Medianas e Tesouros

9.1

A Base Média

Nesta seção vamos resolver problemas utilizando uma elemento muito simples da geometria elementar: a base média.

Em um triângulo ABC, considere os pontos médios M, N e P dos lados BC, CA e AB, respectivamente. O triângulo

tem três bases médias, obtidas ligando os pontos médios de dois lados. Por exemplo, uma das bases médias é o

segmento NP. Diremos que esta base média é relativa ao lado BC. Analogamente, MN e P são as bases médias

relativas aos lados AB e CA, respectivamente.

........B.

M.

C.

A

.

N

.

P

Cada base média é paralela ao lado que corresponde e tem comprimento igual a metade do comprimento do lado.

Uma consequência deste resultado é o Teorema de Varignon.

Teorema 9.1.1. Os pontos médios dos lados de um quadrilátero são vértices de um paralelogramo.

Este resultado é válido mesmo em um quadrilátero não-convexo ou até mesmo entrecruzado.

..........A.

M.

B.

C

.

P

.

D

.

N

.

Q

..........A.

M.

B.

C

.

P

.

D

.

N

.

Q

Demonstração. Seja ABCD um quadrilátero convexo e M, N, P e Q os pontos médios dos lados AB, BC, CD e DA,

respectivamente. Vamos provar que MNPQ é um paralelogramo.

Considerando o triângulo ABC, o segmento MN é base média relativa ao lado AC, sendo paralelo ao mesmo e

medindo a metade de AC.

Analogamente, olhando para o triângulo CDA, o segmento PQ é base média relativa ao lado AC, e portanto é

paralelo a AC e mede a metade de AC.

Segue que os segmentos MN e PQ são iguais e paralelos, mostrando que o quadrilátero MNPQ é um paralelo-

gramo.

65

Page 66: compilacaoModulo1Nivel2

DRAFT

66 Geometria– Bases Médias, Medianas e Tesouros – Professor Paulo Rodrigues

Os demais casos são análogos e deixados para o leitor.

Problema 1 Prove que a área de MNPQ é a metade da área de ABCD.

Definição 9.1.2. Dado um trapézioABCD de basesAB eCD, a base média do trapézio é o segmento de reta determinado

pelos pontos médios dos lados BC e AD.

Teorema 9.1.3. A medida da base média de um trapézio é paralela às bases e tem por medida a média aritmética das

medidas das bases.

Demonstração. Considere um trapézio ABCD de bases AB e CD, com CD ⩾ AB. Construa X sobre CD de tal modo

que AB = DX. Então o quadrilátero ABXD possui dois lados opostos iguais e paralelos (AB = DX e AB ∥ DX), sendo

portanto um paralelogramo. Desse modo, AD = BX e AD ∥ BX.

..........A. B.

C

.

D

.

X

.

N

.

P

.

M

Sejam M e N os pontos médios de BC e AD. Marque também o ponto P, médio de BX. O quadrilátero NDXP é um

paralelogramo porque ND = PX e ND ∥ PX.

Por outro lado, PM é base média do triângulo BXC, donde PM mede a metade de XC e é paralelo à CD.

Desse modo podemos concluir que N, P e M são colineares e

MN = MP + PN =XC

2+DX =

CD−DX

2+DX =

CD+DX

2=

AB+ CD

2.

9.2

O Primeiro Tesouro

Exemplo 1 Um pirata resolveu enterrar um tesouro em uma ilha. Para tal, ele caminhou da árvore A para a rocha

R1, e depois a mesma distância e na mesma direção até o ponto X. Ele fez o mesmo em relação a entrada da caverna

C e em relação à rocha R2, alcançando os pontos Y e Z, respectivamente. Ele enterrou o tesouro em T , ponto médio de

AZ.

..........

X

.

C

.

Y

.R1

.R2

.

A

.

Z

.

T

Ao voltar à ilha para desenterrar o tesouro, o pirata encontrou as rochas e a caverna, mas não encontrou a árvore.

Como o pirata pode descobrir o tesouro?

Solução: A chave para o pirata encontrar o tesouro está no fato de que em todo quadrilátero, os pontos médios dos

lados são vértices de um paralelogramo.

Page 67: compilacaoModulo1Nivel2

DRAFT

OBMEP • PECI – Preparação Especial para Competições Internacionais 67

Isto significa que a posição T do tesouro independe da posição da árvore. No quadrilátero AXYZ, R1, C, R2 e T

são os pontos médios dos lados. Portanto, R1CR2T é um paralelogramo.

O pirata pode começar de um ponto qualquer e repetir os procedimentos, ou pode determinar T traçando uma

reta paralela a R1C por R2 e uma paralela a CR2 por R1. O ponto de interseção das paralelas é o ponto T , localização

do tesouro.

9.3

Outro Tesouro Perdido

Exemplo 2 Um pirata encontrou uma mapa acompanhado de instruções para localizar um tesouro em uma ilha

deserta. As instruções eram as seguintes:

(1) Andar da palmeira P até a rocha R, girar 90 à direita e caminhar a mesma distância palmeira-rocha, chegando a

um ponto no qual deve-se fazer uma marca M1 .

(2) Voltar a palmeira, caminhar desta à caverna C e girar 90 à esquerda. Caminhar a mesma distância palmeira-

caverna, chegando a um ponto no qual faz-se uma segunda marca M2.

(3) O tesouro está enterrado no ponto médio das duas marcas.

PC

T

R

M1

M2

Chegando, à ilha, o pirata conseguiu localizar a rocha e a caverna, mas não viu palmeira alguma. Como ele pode

encontrar o tesouro?

Solução: Na resolução de problemas de geometria plana o começo ideal é a construção de uma boa figura. Fixadas

as posições da rocha e da caverna, variamos a posição da palmeira, nos pontos P e P1.

...........

P

.

P1

.

C

.

T

.

R

.

M1

.

M3

.

M2

.

M4

Page 68: compilacaoModulo1Nivel2

DRAFT

68 Geometria– Bases Médias, Medianas e Tesouros – Professor Paulo Rodrigues

Esta construção leva a seguinte conjectura: a posição do tesouro independe da localização de P. Se isto for verdade,

o pirata facilmente poderá refazer o procedimento escolhendo arbitrariamente a posição da palmeira.

Vamos provar algo mais forte: o triângulo determinado pelas posições da rocha, da caverna e do tesouro é retângulo

e isósceles, o que fixa a posição do tesouro.

...........

P

.

C

.

T

.

R

.

Y

.

M1

.

M2

.

X

Sejam X o ponto médio de M1P e Y o ponto médio de PM2. O segmento TY é base média do triângulo M2PM1 e

então TY mede a metade de M1P. O segmento RX também mede a metade de M1P pois é mediana relativa a hipotenusa

do triângulo retângulo M1RP. Portanto, TY = RX.

Analogamente, considerando o lado M2P do triângulo M1PM2, concluímos que XT = YC.

Além disso, os ângulos TXR e CYT são congruentes, pois TXM1 = YPX = M2YT e M1XR = 90 = CYM2.

Segue que os triângulos TXR e CYT são congruentes pelo caso de congruência lado-ângulo-lado e então TR = CT .

Portanto, provamos que o triângulo TRC é isósceles de base RC. Provemos que é também retângulo:

RTC = RTX+ XTY + YTC =

= RTX+ XPY + XRT =

= RTX+M1XT + XRT = 180 −M1XR = 90,

pois TYPX é um paralelogramo e YTC = XRT pela congruência provada anteriormente.

9.4

Problemas Propostos

Problema 2 (Torneio das Cidades - 1983) Construa um quadrilátero, dadas as medidas de seus lados e a medida do

segmento que une os pontos médios das diagonais.

Problema 3 (Austrália – 1983) P é um ponto no interior do triângulo ABC tal que os ângulos PAC e PBC são

congruentes. As perpendiculares de P a BC e CA intersectam estes lados em L e N respectivamente, e D é o ponto

médio de AB. Prove que DL = DM.

Problema 4 (OBM 2002 – Nível 3 – 2ª Fase) Em um quadrilátero convexo ABCD, os lados opostos AD e BC são

congruentes e os pontos médios das diagonais AC e BD são distintos. Prove que a reta determinada pelos pontos

médios das diagonais forma ângulos iguais com AD e BC.

Problema 5 (Torneio das Cidades 1994) Três ângulos de um quadrilátero não convexo e não entrecruzado medem

45. Demonstrar que os pontos médios de seus lados são vértices de um quadrado.

Problema 6 ADB eAEC são triângulos retângulos isósceles (AB eAC como hipotenusas) construídos externamente

sobre os lados AB e AC do triângulo ABC; F é o ponto médio de BC. Prove que DEF é um triângulo retângulo isósceles.

Problema 7 Desafio! Seja ABCDEF um hexágono convexo no qual AC = DF, CE = FB e EA = BD. Prove que as

retas que unem os pontos médios dos lados opostos do hexágono são concorrentes.

Page 69: compilacaoModulo1Nivel2

DRAFT

OBMEP • PECI – Preparação Especial para Competições Internacionais 69

9.5

Medianas

Definição 9.5.1. Chamamos de mediana de um triângulo um segmento que une um vértice ao ponto médio do lado

oposto.

......

A

.B.

C.

M

Um triângulo possui então três medianas, cada uma partindo de um vértice.

......

A

.B.

C.

M......

A

.B.

C.

N

......

A

.B.

C.

P

Teorema 9.5.2. As três medianas de um triângulo contêm um ponto comum, chamado de baricentro do triângulo.

........

A

.B.

C.

M.

P

.

N

..

G

Demonstração. Trace as medianas AM e BN no triângulo ABC. Seja G o ponto de interseção destas medianas. Trace

a base média MN do triângulo ABC e a base média XY do triângulo ABG, com X em AG e Y em BG.

.......

A

.B.

C.

M.

N

..

G

...

X

.Y

Page 70: compilacaoModulo1Nivel2

DRAFT

70 Geometria– Bases Médias, Medianas e Tesouros – Professor Paulo Rodrigues

Sabemos que MN é base média do triângulo ABC e então a reta MN é paralela à reta AB e o segmento MN tem

comprimento igual à metade da medida do segmento AB.

Por outro lado, o segmento XY é base média do triângulo ABG e portanto, a reta XY é paralela à reta AB e o

segmento XY tem por medida a metade da medida do segmento AB.

Portanto, as retas XY e MN são paralelas e os segmentos XY e MN possuem o mesmo comprimento.

Pelo paralelismo, os ângulos YXG e GMN são iguais (alternos internos). Pelo mesmo motivo também são iguais

os ângulos GYX e GNM.

.......

A

.B.

C.

M.

N

..

G

...

X

.Y

....

Segue que os triângulos GXY e GMN são congruentes pelo caso de congruência ALA. Portanto, XG = MG e

YG = NG. Portanto, como sabemos que AX = XG, podemos concluir que AX = XG = GM e então o ponto G divide

a mediana AM na razão 2 : 1. Analogamente, este ponto divide a mediana BN na razão de 2 : 1.

De modo análogo, podemos provar que o ponto de interseção das medianas AM e CP divide cada uma destas na

razão 2 : 1. Assim, o ponto G é comum as três medianas e este ponto é chamado de baricentro do triângulo ABC.

9.6

Mediana em um triângulo retângulo

Um resultado muito útil é o seguinte:

Teorema 9.6.1. Em todo triângulo retângulo, a medida da mediana relativa a hipotenusa tem comprimento igual a

metade da medida da hipotenusa.

Demonstração. 1ª – Formando triângulos isósceles.

......A

.B

.

C

.

X

...β

......A

.B

.

C

.

X

...β

.

90 − β

.

90 − β

..

Considere o triângulo ABC, retângulo em A. Construa o ponto X sobre CB de tal modo que AX = XB. Seja β a

medida do ângulo XBA. Então, BAX = β, XAC = 90 −β (porque o ângulo A é reto) e ACX = 90 −β (lembre-se que

ABC é retângulo).

Portanto, AX = CX e como por construção, AX = XB, temos que AX = BX = CX e portanto, X é o ponto médio

de CB e a mediana AX mede a metade da hipotenusa BC.

Page 71: compilacaoModulo1Nivel2

DRAFT

OBMEP • PECI – Preparação Especial para Competições Internacionais 71

Demonstração. 2ª – Completando o retângulo.

Dado o triângulo ABC retângulo em A construa o ponto D de tal modo a obter o retângulo ABDC. Construa a

diagonal AD.

.......A

.B

.

C

.

D

.

M

Como um retângulo é um paralelogramo, o ponto de interseção das diagonais é o ponto médio de ambas. Deste

modo, AM = MD e BM = MC. Como as diagonais de um retângulo são iguais, temos que AM = MC = MB = MD

e portanto, M é o ponto médio de BC e a medida da mediana AM é igual a metade da medida da hipotenusa BC.

Problema 8 (OCM) Sejam ABC um triângulo qualquer e G o ponto de encontro das suas medianas. Veja que uma

reta r qualquer que passe pelo ponto G, excetuando-se as medianas, separa um dos vértices do triângulo, por exemplo

A, dos outros dois B e C. Prove que a soma das distâncias de B e C à reta é igual a distância de A à reta r.

Solução: A situação do problema está representada na figura abaixo:

.........

A

.

Y

.

Z

.

G

.B.

X

.C

Sejam X, Y e Z os pés das perpendiculares traçados dos pontos A, B e C com relação a reta r. Devemos provar

que AX = BY + CZ.

Observe que YBCZ é um trapézio de bases BY e CZ (pois estes são perpendiculares a reta r). Sendo M o ponto

médio de BC, traçamos a base média do trapézio MT (veja figura abaixo).

.........B.

X

.C

.M

.

G

..........

A

.

Y

.

Z

.

T

MT é perpendicular a r, pois é paralelo a cada uma das bases BY e CZ. Como MT é a base média do trapézio,

sabemos que 2MT = BY + CZ. Basta então provar que 2MT = AX.

Observe os triângulos AXG e MTG. São semelhantes, pois as retas AX e MT são paralelas. Lembrando que o

baricentro G divide a mediana em segmentos cuja razão é de 2 : 1, obtemos:

AX

TM=

AG

GM= 2 =⇒ AX = 2TM.

Page 72: compilacaoModulo1Nivel2

DRAFT

72 Geometria– Bases Médias, Medianas e Tesouros – Professor Paulo Rodrigues

Segue que AX = 2TM = BY + CZ.

9.7

Problemas Propostos

Problema 9 Sobre o lado BC do triângulo ABC o ponto A1 é marcado de tal modo que BA1 : A1C = 2 : 1. Qual é

a razão em que a mediana CP divide o segmento AA1?

Problema 10 Trace as medianas AM, BN e CP do triângulo ABC de baricentro G. Prove que os seis triângulos

AGP, PGB, BGM, MGC, CGN e NGA possuem áreas iguais.

Problema 11 No triângulo ABC traçamos as medianas AM, BN e CP. Por M traçamos MD paralelo e igual a BN

de modo que A e D estejam no mesmo semiplano determinado pela reta BC. Prove que AD = CP.

Problema 12 As medianas AM e BN do triângulo ABC são perpendiculares e medem 12 cm e 9 cm, respectiva-

mente. Determine a medida do lado AB.

Problema 13 Construa um triângulo dadas as medidas das três medianas.

Problema 14 Considere um triângulo equilátero ABC; escolha sobre os lados AB,BC e AC pontos A1, B1, C1 tais

que

AA1 = BB1 = CC1 =1

3AB.

(a) Prove que o triângulo A1B1C1 é equilátero.

(b) Prove que cada uma das retas A1B1, B1C1 e C1A1 é perpendicular a um dos lados do triângulo.

Page 73: compilacaoModulo1Nivel2

DRAFT

..

Geometria

. 10.

Problemas Extras

Problema 1 Considere dois pontos A e B do mesmo

lado de uma reta r. Determine um ponto P sobre a reta

de modo que a soma das distâncias AP+PB seja a menor

possível.

....

A

.

B

.r

Solução: Observe que se a reta r separasse os pontos

A e B não haveria o que fazer. O menor caminho seria o

segmento de reta AB e a o ponto P seria a intersecção de

AB com a reta r. Podemos transformar o problema dado

neste outro do seguinte modo:

.......

A

.

B

.r

.P

.P1

.

B1

Seja B1 o simétrico de B em relação a reta r e P1 a

intersecção de AB1 e r. Afirmamos que o caminho AP1B

é o menor possível.

De fato, sendo P um ponto qualquer de r diferente de

P1, temos

AP+PB = AP+PB1 > AB1 = AP1+P1B1 = AP1+P1B.

Problema 2 Mostre como (em que posição) devemos

construir uma ponte sobre um rio de modo que uma pes-

soa que saia de A e caminhe até o ponto B percorra o me-

nor caminho possível. (A ponte deve ser perpendicular às

margens do rio - veja a figura abaixo).

....

A

.B

Problema 3 (Canadá - 91) Seja C um círculo e P um

ponto do plano. Cada reta através de P que intersecta Cdetermina uma corda em C. Mostre que os pontos médios

destas cordas estão sobre uma circunferência.

Problema 4 (Olimpíada do Cone Sul – 92) Seja P um

ponto fora de uma circunferência C. Determinar dois

pontos Q e R na circunferência tais que P, Q, R estão ali-

nhados e Q é o ponto médio do segmento PR. (discutir o

número de soluções)

Problema 5 Um mosca está pousada em um dos vér-

tices de um cubo de madeira. Qual o menor caminho que

ela pode seguir para chegar ao vértice oposto?

Problema 6 Seja P um ponto no interior de um triân-

gulo equilátero ABC. Prove que a soma das distâncias de

P aos lados de ABC é constante, isto é, não depende da

localização de P.

73

Page 74: compilacaoModulo1Nivel2

DRAFT

PARTE V

Sessões de ProblemasProfessor Paulo Rodrigues

Page 75: compilacaoModulo1Nivel2

DRAFT

..

Problemas

. 11.

Sessão 1

1. A soma de treze inteiros positivos distintos é igual a

92. Determine estes números.

2. Mostre como formar um quadrado utilizando quatro

figuras idênticas à mostrada na figura 2.1.

..

Figura 2.1

3. Cristiane dobrou uma folha retangular de papel de tal

modo que um vértice coincidiu com o ponto médio de um

lado, como indicado na figura 3.1. Ela descobriu que os

triângulos I e II são iguais.

..I

.

II

.Figura 3.1

Determine a medida do maior lado da folha, sabendo que

o lado mais curto mede 8 cm.

4. Pedro nasceu no século 19, enquanto seu irmão Paulo

nasceu no século 20. Certa vez os irmãos se encontra-

ram em uma festa comemorando o aniversário de ambos.

Pedro disse, “Minha idade é igual à soma dos dígitos do

meu ano de nascimento”. “A minha também”, respondeu

Paulo. Quantos anos Paulo é mais jovem que Pedro?

5. Encontre todas as ternas (x, y, z) de números reais que

satisfazem o sistemax(x+ y+ z) = 26

y(x+ y+ z) = 27

z(x+ y+ z) = 28.

6. Seguem alguns exemplos nos quais a soma dos quadrados de k números positivos consecutivos é igual à soma dos

quadrados dos k− 1 inteiros seguintes:

32 + 42 = 52,

362 + 372 + 382 + 392 + 402 = 412 + 422 + 432 + 442,

552 + 562 + 572 + 582 + 592 + 602 = 612 + 622 + 632 + 642 + 652.

Encontre uma fórmula geral para todos os casos.

75

Page 76: compilacaoModulo1Nivel2

DRAFT

..

Problemas

. 12.

Sessão 2

7. Pinte de preto seis casas de um tabuleiro branco 6×6,

de tal modo que não seja possível cortar um retângulo

branco 1× 6 ou um quadrado branco 3× 3.

8. Temos 9 moedas, uma das quais é falsa (ela é mais

leve do que as outras). Encontre a moeda falsa utilizando

duas pesagens em uma balança de pratos.

9. Uma sequência numérica é formada de acordo com

a seguinte regra: o primeiro número é 7 e cada número,

a partir do segundo, é igual a soma dos dígitos do qua-

drado do número anterior, aumentada em uma unidade.

Por exemplo, o segundo número é 14, porque 72 = 49 e

4+9+1 = 14. O terceiro número é 17 e assim por diante.

Qual o milésimo número da sequência?

10. Escreva um dos números de 1 a 12 em cada um dos

doze triângulos equiláteros pequenos da figura de modo

que, em cada triângulo equilátero formado por quatro tri-

ângulos pequenos, a soma dos números escritos seja igual

a 20.

..

Figura 10.1

11. O triângulo retângulo ABC tem ângulo reto em C e

o ângulo A mede 30. O centro da circunferência inscrita

no triângulo é ponto I e D é o ponto de interseção desta

circunferência com o segmento BI. Prove que os segmen-

tos AI e CD são perpendiculares.

12. No quadrado ABCD, a linha poligonal KLAMN é tal

que os ângulos KLA, LAM e AMN medem 45.

...

45

.........

B

.

L

.

C

.A.

D.

N.

M

.

K

...

Figura 12.1

Demonstre que KL2 +AM2 = AL2 +MN2.

76

Page 77: compilacaoModulo1Nivel2

DRAFT

..

Problemas

. 13.

Sessão 3

13. O rei pretende construir seis castelos em seu reino

e ligar dois quaisquer deles por uma estrada. Faça um

diagrama dos castelos e das estradas de modo que elas

se cruzem ao todo três vezes e exatamente duas estradas

passem em cada cruzamento.

14. A soma dos algarismos de um número é igual a 2010.

Este número pode ser um quadrado perfeito?

15. Sete estudantes contam de 1 a 1000 como segue:

• André diz todos os números, com exceção do número

do meio em cada grupo de três números consecutivos.

Isto é, André diz 1, 3, 4, 6, 7, 9, . . . , 997, 999, 1000.

• Bruno diz todos os números que André não disse, ex-

ceto que ele também salta o número do meio em cada

grupo de três números consecutivos.

• Clara diz todos os números que André e Bruno não dis-

seram, exceto que ele também salta o número do meio

em cada grupo de três números consecutivos.

• Daniel, Estevão e Fábio dizem todos os números que

nenhum dos estudantes com o primeiro nome come-

çando antes do seu no alfabeto disseram, exceto que

eles também saltam o número do meio em cada grupos

de três números consecutivos.

• Finalmente, Gabriel diz o único número que ninguém

disse.

Que número Gabriel disse?

16. O treinador da equipe de natação decidiu organi-

zar uma série de competições entre os 7 integrantes da

equipe. Em cada dia será realizado uma única prova com

a participação de três nadadores. Cada nadador compe-

tirá exatamente uma vez com cada um dos outros.

(a) Quantos dias durará esta série de competições? Expli-

que ou justifique por que não pode durar nem mais

dias, nem menos dias que o número afirmado.

(b) Mostre uma possível distribuição indicando os três na-

dadores que competem em cada dia.

17. Considere os pontos M e N sobre os lados BC e CD

do quadrado ABCD, tais que o ângulo MAN mede 45.

........

B

.

M

.

C

.A.

D.

N

..

45

.

Figura 17.1

Prove que a diagonalBD divide o triânguloAMN em duas

partes de mesma área.

18. Existe um hexágono que pode ser dividido em quatro

triângulos congruentes por um único corte reto?

77

Page 78: compilacaoModulo1Nivel2

DRAFT

..

Problemas

. 14.

Sessão 4

19. O campo do jogo Batalha Naval é um tabuleiro

10 × 10, o qual contém um “navio” oculto no formato de

um retângulo 1×3. É sempre possível acertar o navio com

até 33 tentativas?

20. O primeiro termo de uma sequência é 439 e cada

termo, a partir do segundo, é igual à soma dos algaris-

mos do termo anterior, multiplicada por 13. Qual é o 100o

termo desta sequência?

21. A lei pirata estabelece que para repartir as moedas

de um tesouro o capitão deve escolher um grupo de pira-

tas e repartir igualmente as moedas entre estes até que

não possua moedas suficientes para dar uma a mais a

cada pirata. As moedas que sobram são a parte do ca-

pitão.

O capitão Morgan deve repartir um tesouro que con-

tém menos de 1000 moedas de ouro. Ele sabe que se es-

colhe 99 piratas ficará com 51 moedas e se escolhe 77 pi-

ratas caberão a ele apenas 29 moedas. Determinar quan-

tos piratas deve escolher Morgan para ficar com a maior

quantidade de moedas, e para essa quantidade de piratas,

quantas moedas ele ganhará. Observação: cada pirata es-

colhido deve receber pelo menos uma moeda.

22. Você possui 6 moedas de pesos 1, 2, 3, 4, 5 e 6 gramas

que parecem iguais, exceto por seus rótulos que indicam

o respectivo peso de cada uma. Como determinar se to-

das as indicações dos rótulos estão corretas, usando uma

balança de pratos somente duas vezes?

23. Duas pessoas realizam um truque. A primeira retira

5 cartas de um baralho de 52 cartas (previamente emba-

ralhado por um membro da plateia), olha-as, e coloca-as

em uma linha da esquerda para a direita: uma com a face

para baixo (não necessariamente a primeira), e as outras

com a face para cima. A segunda pessoa deve adivinhar

a carta que está com a face para baixo. Prove que elas

podem combinar um sistema que sempre torna isto pos-

sível.

24. No triângulo ABC, o ângulo B mede 60. Traçamos

as bissetrizes AD e CE, sendo D um ponto do lado BC e

E um ponto do lado AB. As bissetrizes intersectam-se no

ponto I. Prove que ID = IE.

78

Page 79: compilacaoModulo1Nivel2

DRAFT

..

Problemas

. 15.

Sessão 5

25. Um polígono é legal se seus vértices estão sobre uma

grade retangular de pontos e cada um de seus lados é ho-

rizontal ou vertical. A distância entre dois pontos vizi-

nhos da grade é 1 cm. Por exemplo, o polígono da figura

seguinte é legal.

..Figura 25.1

(a) Existe um polígono legal com perímetro igual a 22

cm e área igual a 14 cm2? Em caso afirmativo, mos-

tre um exemplo e caso contrário justifique.

(b) Existe um polígono legal com perímetro igual a 21

cm e área igual a 14 cm2? Em caso afirmativo, mos-

tre um exemplo e caso contrário justifique.

26.(a) Existem dois números naturais consecutivos tais que

as somas de seus algarismos são ambas divisíveis por

7?

(b) Existem dois números naturais consecutivos tais que

as somas de seus algarismos são ambas divisíveis por

9?

Em ambos os casos, se a resposta for afirmativa, dê um

exemplo. Se a resposta for negativa, justifique.

27. O ponto K é marcado sobre a hipotenusa AB do

triângulo retângulo ABC de modo que CK = BC. O seg-

mento CK divide a bissetriz interna AL em dois segmen-

tos de mesma medida (L é um ponto do lado BC). Deter-

mine as medidas dos ângulos do triângulo ABC.

28. Em uma folha quadriculada marcamos os pontos A,

B, C, D, M e N, como mostra a figura 28.1.

........

A

.

B

.

C

.

D

.M.

N.....

Figura 28.1

Prove que a soma dos ângulos MAN, MBN, MCN e

MDN é igual a 45.

29. Um país tem 12 ministros. Cada ministro é amigo

de 5 ministros e inimigo dos outros 6. Cada comitê é for-

mado por 3 ministros. Um comitê é considerado legítimo

se todos os seus membros são amigos ou se todos são ini-

migos. Quantos comitês legítimos podem ser formados?

30. Uma professora de matemática pensou em um in-

teiro positivo de dois algarismos. Ela deseja que seus dois

inteligentes alunos Daniela e Adriano determinem o valor

exato do número pensado.

Para tal, informa reservadamente a Daniela a quan-

tidade de divisores positivos do número e confidencia a

Adriano a soma dos algarismos do número.

Uma breve conversa entre Daniela e Adriano é trans-

crita abaixo:

• Adriano: Eu não posso determinar o número.

• Daniela: Nem eu, mas posso dizer se ele é par ou ímpar.

• Adriano: Agora eu sei qual é o número.

• Daniela: Você sabe? Então eu também sei.

Suponha que os estudantes são honestos e existe lógica

perfeita em tudo o que falaram. Determine o número pen-

sado pela professora justificando sua resposta.

79

Page 80: compilacaoModulo1Nivel2

DRAFT

PARTE VI

Avaliações e SimuladosProfessor Paulo Rodrigues

Page 81: compilacaoModulo1Nivel2

DRAFT

..

Problemas

. 16.

Prova de Seleção 2013

Problema 1 André possui n pesos cujas massas em gramas são dadas pelos números 1, 2, . . . , n.

(a) É possível que André consiga dividir estes pesos em três grupos de mesma massa se n = 2011?

(b) É possível que André consiga dividir estes pesos em três grupos de mesma massa se n = 2010?

Problema 2 Fábio escreveu um número inteiro em cada uma das quarenta e nove casas de um tabuleiro 7× 7. Ele

fez isso de modo que, em cada quadrado 2× 2 e em cada quadrado 3× 3, a soma dos números escritos foi igual a 0.

Prove que a soma dos números escritos por Fábio nas 24 casas que formam o bordo do tabuleiro também é igual

a 0.

Problema 3 Prove que a equação a2 + b2 = c2 + 3 possui infinitas soluções com a, b e c inteiros positivos.

Problema 4 Os pontos A, B, C, D e E são vértices consecutivos de um polígono regular de 18 lados inscrito em uma

circunferência de centro O. O ponto P está sobre a semirreta OA de tal modo que o triângulo OPB é isósceles de base

OP.

.........O.

A

.

B

.

C

.

D

.

E

.

P

(a) Determine a medida do ângulo CEB.

(b) Prove que PB = BE.

(c) Determine as medidas dos ângulos do triângulo PBC.

Problema 5 Em uma fila existem 30 crianças marcadas com os números 1, 2, . . . , 30 da esquerda para a direita. Para

cada criança marcada com o número i tal que 2 ⩽ i ⩽ 15, a quantidade de amigos com número maior que i é igual a 1

81

Page 82: compilacaoModulo1Nivel2

DRAFT

82 Avaliações e Simulados– Prova de Seleção 2013 – Professor Paulo Rodrigues

mais a quantidade de amigos com número menor que i. Para cada criança marcada com número i tal que 16 ⩽ i ⩽ 29,

a quantidade de amigos com número menor que i é igual a 2 mais a quantidade de amigos com número maior que i.

A criança 1 tem 19 amigos. Quantos amigos possui a criança 30?

Problema 6 Colocamos 15 cavalos em um tabuleiro 15× 15 de modo que não existam dois cavalos em uma mesma

linha ou em uma mesma coluna. Cada cavalo faz então um movimento. Prove que agora existem dois cavalos na

mesma linha ou na mesma coluna.

Observação: O cavalo movimenta-se em L. Assim, um cavalo na casa marcada com

.

pode mover-se para as casas destacadas.

.

Page 83: compilacaoModulo1Nivel2

DRAFT

..

Problemas

. 17.

Avaliação 1

• Avaliação aplicada no primeiro encontro presencial no dia 13/01 com duração de 4 horas e 30 minutos.

17.1

Álgebra

Problema 1 Determine m para que as raízes da equação x2 − 3x+ 2m = 0 sejam reais e iguais.

Problema 2 Se x é um número real tal que x+1

x= k, determine, em função de k

(a) x2 +1

x2

(b) x3 +1

x3

Problema 3 Resolver, em R, o sistema de equações abaixo.

y2 + u2 + v2 +w2 = 4x− 1

x2 + u2 + v2 +w2 = 4y− 1

x2 + y2 + v2 +w2 = 4u− 1

x2 + y2 + u2 +w2 = 4v− 1

x2 + y2 + u2 + v2 = 4w− 1

17.2

Combinatória

Problema 4 Dados 50 pontos arbitrários dentro de um quadrado de lado 1, mostre que existem dois desses pontos

cuja distância entre eles é menor que2

9.

Problema 5 Em um tabuleiro de xadrez 8×8 são retirados os cantos superior esquerdo e o inferior direito, obtendo-

se um tabuleiro truncado. Explique se é possível um cavalo, partindo de uma casa qualquer, percorrer todas as casas

do tabuleiro truncado passando por cada casa uma única vez.

Problema 6 Prove que dentre 52 inteiros é possível escolher dois cuja soma ou diferença é divisível por 100.

17.3

Geometria

Problema 7 A figura mostra parte de um polígono regular de 20 lados (icoságono) ABCDEF..., um quadrado BCYZ

e um pentágono regular DEVWX.

83

Page 84: compilacaoModulo1Nivel2

DRAFT

84 Avaliações e Simulados– Avaliação 1 – Professor Paulo Rodrigues

..

A

.

B

.

C

.

D

.

E

.

F

.

X

............

Y

.

Z

.

V

.

W

(a) Determine a medida do ângulo YDC.

(b) Mostre que o vértice X está sobre a reta DY.

Problema 8 Na figura, ABCD e CEFG são quadrados e o lado do quadrado CEFG mede 12 cm.

.........

A

.

D

.

F

.B.

C.

E.

G

Quais são os possíveis valores da área do triângulo AEG?

Problema 9 Mostre que é possível construir um pentágono com todos os lados de mesma medida e cujos ângulos

internos meçam 60, 80, 100, 140 e 160, em alguma ordem.

17.4

Números

Problema 10 Existe um número natural tal que o produto de seus algarismos é igual a 1980?

Problema 11 Determine todos os inteiros positivos n tais que 3n− 4, 4n− 5 e 5n− 3 são todos primos.

Problema 12 Um número escrito com cem algarismos 0, cem algarismo 1 e cem algarismos 2 pode ser um quadrado

perfeito?

Page 85: compilacaoModulo1Nivel2

DRAFT

..

Problemas

. 18.

Simulado 1

• Simulado aplicado no primeiro encontro presencial no dia 16/01 com duração de 4 horas e 30 minutos.

Problema 1

(a) Simplifique a expressão

x2 − (x+ 1)2 − (x+ 2)2 + (x+ 3)2.

(b) Prove que, para todo inteiro positivo n, existe um inteiro positivo k e uma escolha adequada de sinais + e − para

a qual

n = ±12 ± 22 ± · · · ± k2.

(por exemplo, 12 = −12 + 22 + 32).

Problema 2 No triângulo ABC com AB = BC, traçamos a bissetriz BD, com D sobre AC. Marcamos o ponto E

sobre BD, diferente de D, de tal modo que CE = CD. Prove que o ponto médio de DE está sobre a base média de

ABC, paralela ao lado AB.

..

B

.C.

D.

A.

E

.......

Problema 3 Existem n jogadores participando de um torneio de xadrez. Cada jogador deve disputar uma partida

com cada um dos outros n − 1 jogadores, e não mais que uma partida por dia. Determine o menor número de

dias necessários para terminar o torneio. Mostre como poderia ser realizado o torneio com a quantidade de dias

encontrada.

85

Page 86: compilacaoModulo1Nivel2

DRAFT

..

Problemas

. 19.

Simulado 2

• Simulado aplicado no primeiro encontro presencial no dia 16/01 com duração de 4 horas e 30 minutos.

Problema 1 Calcule a área do triângulo ABC abaixo, dados BD = 4, DE = 2, EC = 6, BF = FC = 3.

..

A

.

B

.

D

.

C

.F .E......

Problema 2 Estão alinhados N gafanhotos (N ⩾ 3). A cada segundo, um dos gafanhotos (exceto os dois primeiros)

pula e passa à frente dos dois imediatamente à sua frente. Após K segundos, os N gafanhotos estão novamente na

ordem original. Quais os possíveis valores de K?

Problema 3 Em Terra Brasilis existem n casas, onde vivem n duendes, cada um em uma casa. Nesse país temos

estradas de mão única satisfazendo as seguintes condições:

(i) Cada estrada une duas casas.

(ii) Em cada casa começa exatamente uma estrada.

(iii) Em cada casa termina exatamente uma estrada.

Todos os dias, a partir do dia 1, cada duende sai da casa onde está e chega à casa vizinha. Uma lenda de Terra Brasilis

diz que, quando todos os duendes regressarem às suas casas originais, o mundo acabará.

(a) Mostre que o mundo acabará.

(b) Se n = 98, mostre que é possível que os duendes construam e orientem as estradas de modo que o mundo não

acabe antes de 300 000 anos.

86

Page 87: compilacaoModulo1Nivel2

DRAFT

..

Soluções

. 20.

Gabarito da Prova de Seleção

Problema 1 André possui n pesos cujas massas em gramas são dadas pelos números 1, 2, . . . , n.

(a) É possível que André consiga dividir estes pesos em três grupos de mesma massa se n = 2011?

(b) É possível que André consiga dividir estes pesos em três grupos de mesma massa se n = 2010?

Solução: de Thiago Lucas Faustino da Silva de Itumbiara – Goiás.

(a) Não, para que André consiga dividir os pesos em três grupos de mesma massa, a soma da massa de todos os pesos

deveria ser um múltiplo de 3, porém, se n = 2011, a soma das massas não será um múltiplo de 3. Veja:

S =(1+ 2011) · 2011

2=

2012

2· 2011 = 1006 · 2011 = 2023066.

Vamos ver se este número é múltiplo de 3 somando os algarismos:

Sa = 2+ 2+ 3+ 6+ 6 = 19

19 não é um múltiplo de 3, logo, S não é múltiplo de 3, por isso é impossível dividir S em três parcelas iguais, que

seriam os três grupos de pesos.

(b) Sim, pois a soma das massas será um múltiplo de 3. Veja:

S =(1+ 2010) · 2010

2= 1005 · 2011 = 2021055.

Se somarmos os algarismos, teremos certeza que a soma é múltiplo de 3.

Sa = 2+ 2+ 1+ 5+ 5 = 15,

que é um múltiplo de 3. Assim, podemos dividir em três grupos cuja soma das massas é 673685.3

Uma maneira de dividir os pesos neste caso seria formar 335 grupos de 6 números consecutivos, que inicia com

um número da forma 6x + 1 e termina num número da forma 6x. Exemplo: 1 − 6, 7 − 12, 13 − 18. Vamos chamar

o menor número de cada grupo (massa do peso de menor massa) de x. O grupo teria:

x, x+ 1, x+ 2, x+ 3, x+ 4, x+ 5

A partir disso, dividimos os grupos A, B e C da seguinte forma:

(a) Grupo A terá números da forma x e x+ 5.

(b) Grupo B terá números da forma x+ 1 e x+ 4.

(c) Grupo C terá números da forma x+ 2 e x+ 3.

3Observe que esta é uma condição necessária, mas não garante que a divisão seja de fato possível.

87

Page 88: compilacaoModulo1Nivel2

DRAFT

88 Avaliações e Simulados– Gabarito da Prova de Seleção – Professor Paulo Rodrigues

Comentários da Equipe do PECI Muitos alunos fizeram confusão em relação ao que é uma condição necessária e

o que é uma condição suficiente.

• Para que seja possível dividir em três grupos de mesma massa é necessário que a soma 1+ 2+ · · ·+n seja divisível

por 3. Assim, no item (a), como a soma não é divisível por 3, não é possível.

• Ja no item (b), o simples fato da soma ser divisível por 3 não implica que seja possível fazer a divisão em três grupos

de mesma massa. Por exemplo, se n = 3, a soma é 1+ 2+ 3 = 6, múltiplo de 3, mas não é possível separar em três

grupos de mesma massa.

• Existem muitas maneiras de construir o exemplo do item b. Uma maneira diferente mas que utiliza o mesmo

princípio da solução apresentada é considerando os conjuntos:

• A = 1, . . . , 335 ∪ 1676, . . . 2010

• B = 336, . . . , 670 ∪ 1341, . . . , 1675

• C = 671, . . . , 1005 ∪ 1006, . . . 1340

Dividimos os números de 1 a 2010 em 6 blocos com a mesma quantidade de inteiros consecutivos e colocamos num

mesmo grupo o primeiro e o último blocos, o segundo e o penúltimo blocos, o terceiro e o quarto blocos. É fácil

verificar que os três grupos possuem a mesma soma.

Problema 2 Fábio escreveu um número inteiro em cada uma das quarenta e nove casas de um tabuleiro 7× 7. Ele

fez isso de modo que, em cada quadrado 2× 2 e em cada quadrado 3× 3, a soma dos números escritos foi igual a 0.

Prove que a soma dos números escritos por Fábio nas 24 casas que formam o bordo do tabuleiro também é igual

a 0.

Solução: de Guilherme Goulart Kowalczuk de Porto Alegre – RS.

..

a

.Figura 2.1

..

a

.Figura 2.2

..

a

.

b

.

c

.

d

.

Figura 2.3

Chamamos de a a soma dos seis números indicados da borda do tabuleiro na figura 2.1. Também analisaremos com

um todo, as duas partes 3× 3 assinaladas no desenho, cuja soma é 0.

Vamos analisar a parte destacada no desenho anterior, como na figura 2.2. A soma de todos os quadradinhos da

figura ao lado tem que ser 0. A soma dos quadrados 2 × 2 assinalados também tem que ser 0. Portanto, a soma dos

quadrados sombreados é a = 0− 0, ou seja, a = 0.

Da mesma forma podemos encontrar os valores de b, c e d como na figura 2.3.

Comentários da Equipe do PECI: Existem várias maneiras de resolver o problema. Uma é observando as formas

“L” de quadrados 3× 3 e 4× 4, mostradas abaixo.

Na figura 2.4, a soma dos números do L3 é igual a diferença entre a soma dos números do tabuleiro 3×3 e a soma

dos números do tabuleiro 2× 2 (veja a figura 2.5), ou seja, igual a zero.

Page 89: compilacaoModulo1Nivel2

DRAFT

OBMEP • PECI – Preparação Especial para Competições Internacionais 89

Na figura 2.6, a soma dos números do L4 é igual a diferença entre a soma dos números do tabuleiro 4×4 e a soma

dos números do tabuleiro 3× 3. Como um tabuleiro 4× 4 pode ser formado por quatro tabuleiros 2× 2 (figura 2.7), a

soma dos números é igual a zero e então a soma dos números do L4 também é igual a zero.

Para concluir o problema, basta observar que o bordo do tabuleiro é a união de dois L3 e dois L4, como mostrado

na figura 2.8.

..Figura 2.4

.

L3

..Figura 2.5

..Figura 2.6

.

L4

..Figura 2.7

..Figura 2.8

Problema 3 Prove que a equação a2 + b2 = c2 + 3 possui infinitas soluções com a, b e c inteiros positivos.

Solução: de Renan Felipe Bergamaschi de Morais de Bariri – SP (Adaptada pela Banca).

O problema pode ser escrito da seguinte forma: a soma de dois quadrados perfeitos é igual a um quadrado perfeito

mais três. Então

a2 + b2 = c2 + 3 ⇐⇒ a2 + b2 − c2 = 3.

Fixamos um valor positivo para a, com a > 3. Por exemplo, se a = 4, temos que a2 = 16, e 16 − 3 = 13. Assim a

equação ficaria b2 + 13 = c2.

Para encontrar os valores de b e c, fazemos um quadro com os quadrados perfeitos

.. 12. 22. 32. 42. 52. 62. 72.

82

.

92

.

102

.

112

.

122

.

132

.

142

.

+3

.

+5

.

+7

.

+9

.

+11

.

+13

.

+17

.

+19

.

+21

.

+23

.

+25

.

+27

.

+11

Perceba que a diferença entre dois quadrados perfeitos consecutivos é sempre ímpar, e aumenta de 2 em 2 a cada

quadrado. Na equação b2+13 = c2, b2 = c2−13 temos que a diferença entre os quadrados é 13, então esses números

são 62 e 72, formando a igualdade 42 + 62 = 72 + 3.

Perceba que no caso anterior a = 4, e se a = 5, a2 = 25, então 25 − 3 = 22, sendo assim, não há dois quadrados

perfeitos consecutivos com essa diferença. Generalizando, um (número ímpar)2 = número ímpar, menos 3 o resultado

é par, que não soluciona o problema para quadrados consecutivos.

Assim a próxima escolha é a = 6, e a2− 3 = 33, ou seja a diferença entre os quadrados de b e c é 33, então b = 16

e c = 17, formando a igualdade 62 + 162 = 172 + 3.

Como há infinitos números pares, há infinitos quadrados perfeitos pares, e assim infinitas soluções para esta

equação.

Comentários da Equipe do PECI: Existem várias formas de resolver o problema. Algebricamente, a solução de

Renan pode ser escrita como a = 2k, para algum inteiro k. Substituindo na equação dada obtemos c2 − b2 = 4k2 − 3.

Escolhendo c = b+ 1, obtemos (b+ 1)2 − b2 = 4k2 − 3, donde b = 2k2 − 2 e c = 2k2 − 1. De fato,

a2 + b2 = (2k)2 + (2k2 − 2)2 = 4k2 + 4k4 − 8k2 + 4 = 4k4 − 4k2 + 1+ 3 = (2k2 − 1)2 + 3 = c2 + 3,

e então obtemos as infinitas soluções. A tabela abaixo mostra soluções com alguns valores de k:

Page 90: compilacaoModulo1Nivel2

DRAFT

90 Avaliações e Simulados– Gabarito da Prova de Seleção – Professor Paulo Rodrigues

k a b c

2 4 6 7

3 6 16 17

4 8 30 31

5 10 48 49

6 12 70 71

Nem todas as soluções da equação são geradas por está fórmula, como por exemplo, a solução a = 12, b = 22 e c = 25.

Problema 4 Os pontos A, B, C, D e E são vértices consecutivos de um polígono regular de 18 lados inscrito em uma

circunferência de centro O. O ponto P está sobre a semirreta OA de tal modo que o triângulo OPB é isósceles de base

OP.

(a) Determine a medida do ângulo CEB.

(b) Prove que PB = BE.

(c) Determine as medidas dos ângulos do triângulo PBC.

Solução: de Luan Lima Freitas – Rio de Janeiro – RJ.

.........O.

A

.

B

.C

.

D

.

E

.

P

Podemos dividir a circunferência em 18 setores, ou triângulos isósceles cujos lados são raios, demarcados por

dois pontos consecutivos do polígono e o centro O, cada um. Como são 18 triângulos congruentes, o ângulo relativo

ao vértice O de cada um é igual a 360/18 = 20.

b) No triângulo EOB, o ângulo O corresponde a 3 setores, ou seja, vale 3∠O = 60. Como o triângulo EOB é

isósceles (pois seu lados são raios) de base EB, EBO e BEO valem, cada um, 180−602

= 60. Vemos assim que ele

é equilátero. Como PBO é isósceles, PB = OB, e sendo EBO equilátero, EB = BO, logo, PB = BE.

a) Sabemos que BEO mede 60. No DEO, O = 20 (um setor) e D = E = 180−202

= 80. O ângulo CEB é

igual a DEO− (DEC+ BEO)

..I

. Sabemos que DEO = 80 e BEO = 60. Analisemos um triângulo um pouco

diferente, DEC: D = 160, um ângulo do polígono de 18 lados. DEC = DCE = 180−1602

= 10. Substituindo

os valores em..I ,

CEB = DEO− (DEC+ BEO) = 80 − (60 + 10) = 10.

c) Sendo OPB isósceles, P = O = 20. Assim, B = 180 − (20 + 20) = 140. O ângulo CBO mede 80, pois

CBO é isósceles e COB = 20. O ângulo CBP, então medirá 360 − (140 + 80) = 140.

O triângulo EPB é isósceles, pois BE = PB, pelo item b). Sabemos que CEB = 10, porém não está claro se C

está sobre o segmento PE ou não. Vamos fazer uma figura caricata:

Page 91: compilacaoModulo1Nivel2

DRAFT

OBMEP • PECI – Preparação Especial para Competições Internacionais 91

......B

.C

.

E

.

P

B = CBP+CBE (da figura original): logo B = 140+CBE. CBE, por sua vez, é igual a CBO−EBO = 80−60 =

20. Também temos que CEB = 10. Bem, se CEB = 10 e PEB = 10 também, isso significa que CEP = 0, ou

seja C está sobre PE. Assim provado, fica óbvio que CPB = 10 e PCB = 180 − (10 + 140) = 30.

Comentários da Equipe do PECI: A dificuldade do problema consistia em não usar sem provar que os pontos P, B

e E estão alinhados como sugeria a figura. Muitos alunos erraram no item (c) ao usar esta condição com base apenas

na figura e alguns usaram também para resolver o item (a).

A maneira mais direta para calcular o ângulo CEB no item (a) é observar que este é um ângulo inscrito na circun-

ferência e portanto, mede a metade do arco CB.

Uma solução alternativa para o item (c), também usa ângulos inscritos na circunferência, porém não a construída

no enunciado:

Como a distância de P, O e E a B é a mesma, então podemos traçar uma circunferência com centro B e raio BO que

vai passar pelos pontos P e E. Assim o ângulo EPO = 12EBO = 30. Por outro lado, como o triângulo OPB é isósceles,

temos que BP0 = BOP = 20, segue que PEB = EPB = 30 − 20 = 10. Pelo item a) sabemos que CEB = 10, donde

se deduz que P, C e E estão alinhados.

Concluímos que CPB = EPB = 10

PCB = CBE+ CEB = 20 + 10 = 30

PBC = 180 − CPB− PCB = 140

.

Problema 5 Em uma fila existem 30 crianças marcadas com os números 1, 2, . . . , 30 da esquerda para a direita. Para

cada criança marcada com o número i tal que 2 ⩽ i ⩽ 15, a quantidade de amigos com número maior que i é igual a 1

mais a quantidade de amigos com número menor que i. Para cada criança marcada com número i tal que 16 ⩽ i ⩽ 29,

a quantidade de amigos com número menor que i é igual a 2 mais a quantidade de amigos com número maior que i.

A criança 1 tem 19 amigos. Quantos amigos possui a criança 30?

Solução:

Page 92: compilacaoModulo1Nivel2

DRAFT

92 Avaliações e Simulados– Gabarito da Prova de Seleção – Professor Paulo Rodrigues

..−x

.

−2

.

−2

.

−2

.

1

.

1

.

1

.

19

Construímos um tabuleiro 30×30, onde preenchemos segundo as relações amizade da seguinte forma: se a criança

i é amiga da criança j então a casa na linha i e coluna j será preenchida com 1, e caso não sejam amigas será preenchida

com 0, e a diagonal não será preenchida. Observemos que, como a relação de amizade é recíproca, o tabuleiro será

simétrico em relação à diagonal que vai do canto superior esquerdo ao canto inferior direito, por exemplo, na casa

que esta na fila 3 coluna 26 temos um 1, se e somente se, na casa que esta na fila 26 coluna 3 temos também um 1.

Agora, vamos construir um novo tabuleiro, mudando o sinal dos números que estão abaixo da diagonal, obtendo

uma tabela que tem soma de todas suas entrada igual a 0. Vejamos que a soma das entradas da linha i corresponde ao

número de amigos que tem número maior que i menos o número de amigos com número menor que i. Pelas hipóteses

do problema, esta soma na primeira linha é 19, nas linhas da 2a à 15a esta soma é 1, nas linha da 16a à 29a esta soma é

−2 e na linha 30a esta soma corresponde −x, onde x é o número de amigos da criança 30. Como a soma deste números

é zero temos que

19+ 1+ 1+ · · ·+ 1︸ ︷︷ ︸14 vezes

−(2+ 2+ · · ·+ 2)︸ ︷︷ ︸14 vezes

−x = 0.

Portanto, x = 19− 14 = 5.

Comentários da Equipe do PECI: Outra maneira de escrever basicamente a mesma solução:

Denotemos por xi o número de amigos da criança numerada com i, os quais têm número menor que i, e yi

o número de amigos da criança numerada com i, os quais têm número maior que i. Pelas hipóteses do problema

sabemos que

x1 = 0

y1 = 19

y30 = 0

xi = yi − 1 para 2 ⩽ i ⩽ 15

xi = yi + 2 para 16 ⩽ i ⩽ 29

Além disso, para cada par de amigos a ⩽ b, esta relação está adicionando uma unidade para xa e uma unidade para

yb. Portanto,

x1 + x2 + · · ·+ x30 = y1 + y2 + · · ·+ y30,

donde

(y1 − x1) + (y2 − x2) + · · ·+ (y15 − x15) + (y16 − x16) + · · ·+ (y29 − x29) + (y30 − x30) = 0.

Assim

19+ 1+ 1+ · · ·+ 1︸ ︷︷ ︸14 vezes

−(2+ 2+ · · ·+ 2)︸ ︷︷ ︸14 vezes

−x30 = 0.

Page 93: compilacaoModulo1Nivel2

DRAFT

OBMEP • PECI – Preparação Especial para Competições Internacionais 93

Logo, x30 = 19+ 14− 2 · 14 = 5.

A figura abaixo mostra que a situação descrita no problema é possível. Ligamos os vértices i e j se as pessoas

i e j são amigas.

..1

.

30

.2

.

29

.3

.

28

.4

.

27

.5

.

26

.6

.

25

.7

.

24

.8

.

23

.9

.

22

.10

.

21

.11

.

20

.12

.

19

.13

.

18

.14

.

17

.15

.

16

Problema 6 Colocamos 15 cavalos em um tabuleiro 15× 15 de modo que não existam dois cavalos em uma mesma

linha ou em uma mesma coluna. Cada cavalo faz então um movimento. Prove que agora existem dois cavalos na

mesma linha ou na mesma coluna.

Observação: O cavalo movimenta-se em L. Assim, um cavalo na casa marcada com

.

pode mover-se para as casas destacadas.

.

Solução: de Thiago Lucas Faustino da Silva de Itumbiara – Goiás.

Um dos motivos é que 15 é ímpar, se o número de cavalos fosse par, poderíamos colocar os cavalos de modo que

dois cavalos troquem de posição no movimento, se fizermos isto, sobra um cavalo sem movimentar-se.

Uma maneira de provar é a seguinte, vamos numerar cada cavalo pela posição, se está na linha 1 coluna 2, como

1+ 2 = 3, o número dele é 3. A soma de todos os números será 450 se não houver cavalos diferentes na mesma linha

ou coluna, a cada movimento, o número de cavalos se altera em: 3, 1, −1 ou −3.

São números ímpares, para que continue com apenas 1 cavalo em cada linha e em cada coluna, a soma dos números

dos cavalos deveria ser 450, o que implicaria uma alteração nula nesta soma, ou seja, 0, a alteração total será a soma

das alterações individuais de cada cavalo, porém, 0 é par e a soma de 15 números ímpares não pode ser par, por isso,

é impossível que continue um cavalo em cada linha e um em cada coluna, logo, existem dois cavalos na mesma linha

ou na mesma coluna.

Comentários da Equipe do PECI: Muitos alunos partiram da hipótese que os cavalos deviam estar em uma das

diagonais do tabuleiro, o que é falso. Na verdade existem 15! = 1 307 674 368 000 maneiras de colocar os cavalos no

tabuleiro, de modo que não fiquem dois na mesma linha ou na mesma coluna. A figura abaixo ilustra uma dessas

maneiras.

Page 94: compilacaoModulo1Nivel2

DRAFT

94 Avaliações e Simulados– Gabarito da Prova de Seleção – Professor Paulo Rodrigues

.

Page 95: compilacaoModulo1Nivel2

DRAFT

..

Soluções

. 21.

Gabarito da Avaliação

21.1

Álgebra

Problema 1 Determine m para que as raízes da equação x2 − 3x+ 2m = 0 sejam reais e iguais.

Solução: Basta fazer ∆ = 0. Deste modo, b2 − 4ac = 0 ⇐⇒ 9− 8m = 0 ⇐⇒ m =9

8.

Problema 2 Se x é um número real tal que x+1

x= k, determine, em função de k

(a) x2 +1

x2

(b) x3 +1

x3

Solução:

(a)

(x+

1

x

)2

= x2 + 2 · x · 1x+

1

x2⇐⇒ k2 = x2 + 2+

1

x2⇐⇒ x2 +

1

x2= k2 − 2.

(b)

(x+

1

x

)3

= x3 + 3 · x2 · 1x+ 3 · x · 1

x2+

1

x3⇐⇒ k3 = x3 + 3

(x+

1

x

)+

1

x3⇐⇒ x3 +

1

x3= k3 − 3k.

Problema 3 Resolver, em R, o sistema de equações abaixo.

y2 + u2 + v2 +w2 = 4x− 1

x2 + u2 + v2 +w2 = 4y− 1

x2 + y2 + v2 +w2 = 4u− 1

x2 + y2 + u2 +w2 = 4v− 1

x2 + y2 + u2 + v2 = 4w− 1

Solução: Somando todas as equações, obtemos

4x2 + 4y2 + 4u2 + 4v2 + 4w2 = 4x− 1+ 4y− 1+ 4u− 1+ 4v− 1+ 4w− 1.

Reorganizando essa equação, vem (2x − 1)2 + (2y − 1)2 + (2u − 1)2 + (2v − 1)2 + (2w − 1)2 = 0, o que resulta

x = y = u = v = w =1

2. Voltando ao sistema, observe que a igualdade 1/4 + 1/4 + 1/4 + 1/4 = 4 · 1/2 − 1 de fato

ocorre.

95

Page 96: compilacaoModulo1Nivel2

DRAFT

96 Avaliações e Simulados– Gabarito da Avaliação – Professor Paulo Rodrigues

21.2

Combinatória

Problema 4 Dados 50 pontos arbitrários dentro de um quadrado de lado 1, mostre que existem dois desses pontos

cuja distância entre eles é menor que2

9.

Solução: Dividindo o quadrado em 49 quadrados de lado 1/7, temos pelo Princípio das Casas dos Pombos, que dois

pontos estão no mesmo quadrado. Observemos que a distância máxima dentro de um quadrado é sua respectiva

diagonal, que mede√2/7.

Mas é fácil verificar que√2/7 < 2/9, pois elevando ao quadrado, isto é equivalente a 2

49< 4

81.

Problema 5 Em um tabuleiro de xadrez 8×8 são retirados os cantos superior esquerdo e o inferior direito, obtendo-

se um tabuleiro truncado. Explique se é possível um cavalo, partindo de uma casa qualquer, percorrer todas as casas

do tabuleiro truncado passando por cada casa uma única vez.

Solução: Podemos supor, sem perda de generalidade que o canto superior esquerdo é preto, e portanto o canto

inferior direito também é preto, assim o tabuleiro truncado tem 30 casas pretas e 32 casas brancas. Para percorrer

todo o tabuleiro o cavalo tem que dar 61 pulos, e cada vez que o cavalo pula, as casas de partida e de chegada são de

cores distintas, segue que depois de um número ímpar de pulos ele já percorreu a mesma quantidade de casas brancas

que pretas e depois de um número par de pulos ele percorreu uma casa a mais de uma cor que da outra.

Concluímos que é impossível percorrer todo o tabuleiro, já que o número de casas brancas supera em dois o

número de casas pretas.

Problema 6 Prove que dentre 52 inteiros é possível escolher dois cuja soma ou diferença é divisível por 100.

Solução: Para aplicar o Princípio das Casas dos Pombos neste problema, construíremos 51 casas da seguinte forma:

na casa j, colocamos o números que deixam resto j ou 100 − j quando dividimos por 100 onde j = 0, 1, . . . , 50. Como

temos 52 números, pelo Princípio das Casas dos Pombos, existem dois números que estão na mesma casa. Caso estes

dois números deixem o mesmo resto quando são divididos por 100, sua diferença será divisível por 100. Por outro

lado, se deixam restos distintos, sua soma será divisível por 100

21.3

Geometria

Problema 7 A figura mostra parte de um polígono regular de 20 lados (icoságono) ABCDEF..., um quadrado BCYZ

e um pentágono regular DEVWX.

..

A

.

B

.

C

.

D

.

E

.

F

.

X

............

Y

.

Z

.

V

.

W

(a) Determine a medida do ângulo YDC.

(b) Mostre que o vértice X está sobre a reta DY.

Page 97: compilacaoModulo1Nivel2

DRAFT

OBMEP • PECI – Preparação Especial para Competições Internacionais 97

Solução:

..

A

.

B

.

C

.

D

.

E

.

F

.........

Y

.

Z

.. ..

A

.

B

.

C

.

D

.

E

.

F

.

V

.

W

.

X

..........

(a) O ângulo interno do icoságono regular mede180 × 18

20= 162. Segue que YCD = 162 − 90 = 72. Como

YC = CD, o triângulo YCD é isósceles de base YD. Assim, YDC = DYC =180 − 72

2= 54.

(b) Cada ângulo interno de um pentágono regular mede180 × 3

5= 108. Assim, CDX = 162 − 108 = 54. Como as

retas XD e YD formam o mesmo ângulo com a reta CD, segue que os pontos X, Y e D pertencem a uma mesma

reta.

(c) Este problema não tem item (c), mas poderíamos ter perguntado: Qual a única letra do alfabeto que ainda pode-

ríamos usar nesta figura?

Resposta:

ComoTé a vigésima letra do alfabeto,o icoságono éABCDEF...T.Como usamos tambémV,W,X,YeZ,só

faltou a letraU!Você já tinha visto um problema de geometria com tantas letras?

Problema 8 Na figura, ABCD e CEFG são quadrados e o lado do quadrado CEFG mede 12 cm.

.........

A

.

D

.

F

.B.

C.

E.

G

Quais são os possíveis valores da área do triângulo AEG?

Fatos que Ajudam: Triângulos com mesma base e mesma altura possuem áreas iguais.

..........

A

.

D

.

F

.B.

C.

E.

X

.

G

Solução: Traçamos a diagonal AC do quadrado ABCD. Como as retas AC e GE formam ângulo de 45o em relação

à reta BE, concluímos que AC e GE são paralelas.

Page 98: compilacaoModulo1Nivel2

DRAFT

98 Avaliações e Simulados– Gabarito da Avaliação – Professor Paulo Rodrigues

Seja X um ponto arbitrário sobre AC. Os triângulos AGE e XGE possuem a mesma área, pois ambos têm a mesma

base GE e a mesma altura que corresponde à distância entre as retas paralelas AC e GE. Tomando X = C, concluímos

que a área do triângulo AGE é igual à área de CGE, isto é, 12× 12/2 = 72 cm2.

Problema 9 Mostre que é possível construir um pentágono com todos os lados de mesma medida e cujos ângulos

internos meçam 60, 80, 100, 140 e 160, em alguma ordem.

Fatos que Ajudam: Se um quadrilátero possui os quatro lados de mesma medida, então ele é um losango. Em um

losango, os ângulos opostos possuem a mesma medida.

Solução:

.......A.

E.

D.

B

.

C

..60

.......A.

E.

D

.

B

.

C

..

80

..60

..

140

..

100

..

160

Suponhamos que já construímos o pentágono ABCDE e que o ângulo em A mede 60. Traçando a reta BE,

concluímos que o triângulo ABE é equilátero, pois AB = AE e EAB = 60. Logo, BE = AB e, portanto, BCDE tem

todos os seus lados com a mesma medida, isto é, BCDE é um losango.

Em particular, os ângulos opostos do losango são iguais. Isto implica que, no pentágono, o ângulo em B é igual

ao ângulo em D mais 60 e o ângulo em E é igual ao ângulo em C mais 60.

Como 160 = 100 + 80 e 140 = 80 + 60, concluímos que os ângulos em C e D devem assumir os valores 80

e 100, não necessariamente nessa ordem, enquanto B e E assumem os respectivos valores de D e C, adicionados de

60.

Portanto, para construir tal pentágono basta construir um triângulo equilátero ABE e um losango BCDE com

ângulos de medidas 100 e 80.

21.4

Números

Problema 10 Existe um número natural tal que o produto de seus algarismos é igual a 1980?

Solução: Como 1980 = 22 × 32 × 5× 11 segue que 11 é fator primo de 1980 e possui mais que um algarismo. Logo,

não pode existir um número natural cujo produto de seus algarismos seja 1980.

Problema 11 Determine todos os inteiros positivos n tais que 3n− 4, 4n− 5 e 5n− 3 são todos primos.

Solução: Note que (3n− 4) + (4n− 5) + (5n− 3) = 12n− 12 = 2(6n− 6), ou seja, a soma dos três números é par.

Se os três fossem ímpares então a soma seria ímpar, logo algum deles é par. Como 2 é o único primo par, algum deles

é igual a 2. Temos 3 casos:

1. Se 3n− 4 = 2 então n = 2. Assim, uma solução é 2, 3 e 7.

2. Se 4n− 5 = 2 então n não é inteiro, absurdo.

3. Se 5n− 3 = 2 então n = 1. Porém, para n = 1 temos 3n− 4 = −1, que não é positivo, absurdo.

Page 99: compilacaoModulo1Nivel2

DRAFT

OBMEP • PECI – Preparação Especial para Competições Internacionais 99

Logo, a única solução é n = 2.

Problema 12 Um número escrito com cem algarismos 0, cem algarismo 1 e cem algarismos 2 pode ser um quadrado

perfeito?

Solução: A soma dos algarismos de qualquer número dessa forma é 0 × 100 + 1 × 100 + 2 × 100 = 300. Portanto,

qualquer número dessa forma é múltiplo de 3, pois 3 divide 300, mas não é múltiplo de 9 pois 9 não divide 300. Assim,

se um número dessa forma é um quadrado perfeito, como 3 divide esse número teríamos que ter 32 = 9 dividindo

esse número. Logo, nenhum número dessa forma pode ser um quadrado perfeito.

Page 100: compilacaoModulo1Nivel2

DRAFT

..

Soluções

. 22.

Gabarito do Simulado 1

Problema 1

(a) Simplifique a expressão

x2 − (x+ 1)2 − (x+ 2)2 + (x+ 3)2.

(b) Prove que, para todo inteiro positivo n, existe um inteiro positivo k e uma escolha adequada de sinais + e − para

a qual

n = ±12 ± 22 ± · · · ± k2.

(por exemplo, 12 = −12 + 22 + 32).

Solução: (a)

x2 − (x+ 1)2 − (x+ 2)2 + (x+ 3)2 =

x2 − (x2 + 2x+ 1) − (x2 + 4x+ 4) + (x2 + 6x+ 9) = 4

Logo, x2 − (x+ 1)2 − (x+ 2)2 + (x+ 3)2 = 4 para todo x.

(b) Vejamos para n = 1, n = 2, n = 3 e n = 4.

• 1 = +12;

• 2 = −12 − 22 − 32 + 42;

• 3 = −12 + 22;

• 4 = −12 − 22 + 32.

Considere agora n > 4. Indutivamente, suponha o resultado válido para n−4. Existem k inteiro positivo e uma escolha

de sinais + e − tais que:

n− 4 = ±12 ± 22 ± · · · ± k2.

Pelo item anterior e pela hipótese de indução temos:

n = ±12 ± 2

2 ± · · · ± k2 + (k + 1)2 − (k + 2)2 − (k + 3)2 + (k + 4)2.

Logo, o resultado também é válido para n. Pelo Princípio da Indução Finita segue que o resultado vale para todos os

números inteiros positivos.

100

Page 101: compilacaoModulo1Nivel2

DRAFT

OBMEP • PECI – Preparação Especial para Competições Internacionais 101

Problema 2 No triângulo ABC com AB = BC, traçamos a bissetriz BD, com D sobre AC. Marcamos o ponto E

sobre BD, diferente de D, de tal modo que CE = CD. Prove que o ponto médio de DE está sobre a base média de

ABC, paralela ao lado AB.

Solução 1

............

B

.C.

D.

A.

F.

E

.

H

.

G

....

β

.

β

.

β

.

90 − β

Seja H o ponto médio de DE e traçe, por H, o segmento

FG com F ∈ AC e G ∈ BC tal que FG ∥ AB. No triângulo

isósceles DCE, temos que CH é mediana e altura com re-

lação à base DE.

Se ∠CBD = ∠DBA = β, então temos ∠BHG =

∠DBA = β (alternos internos), logo o triângulo BGH

é isósceles e BG = GH. Como o triângulo BCH é re-

tângulo, temos ∠HCB = 90 − ∠CBH = 90 − β e

∠CHG = 90 − ∠BHG = 90 − β, ou seja, o triângulo

CGH é isósceles e CG = GH = BG.

Portanto, G é ponto médio de BC e FG é base média

do triângulo ABC.

Solução 2

..........

B

.

E

.

H

.C.

D.

A.

C ′

Como BD é bissetriz, ao refletir o triângulo BCD com

relação à BD, obtemos um triângulo BC ′D, com C ′ sobre

AB. Temos também que o triângulo C ′DE é uma reflexão

do triângulo CDE em relação à BD, logo eles são congru-

entes.

Se H é ponto médio de DE, então em ambos os triân-

gulos, H é pé da altura com relação à base. Assim, C, H e

C ′ estão alinhados e, em particular, H é ponto médio da

ceviana CC ′, logo H está sobre a base média em relação à

AB (ou de outro modo, traçando FG paralelo a AB, con-

forme a solução anterior, temos queCFG ∼ CAB, com

razão de semelhança igual aCH

CC ′ =1

2, logo FG =

AB

base média).

Page 102: compilacaoModulo1Nivel2

DRAFT

102 Avaliações e Simulados– Gabarito do Simulado 1 – Professor Paulo Rodrigues

Problema 3 Existem n jogadores participando de um torneio de xadrez. Cada jogador deve disputar uma partida

com cada um dos outros n − 1 jogadores, e não mais que uma partida por dia. Determine o menor número de

dias necessários para terminar o torneio. Mostre como poderia ser realizado o torneio com a quantidade de dias

encontrada.

Solução:

É claro que se o número n for ímpar, então pelo me-

nos um jogador deverá descansar em cada rodada e te-

remos no mínimo n rodadas para completar o torneio,

porque o jogador que descansar na primeira rodada pre-

cisará de no mínimo mais n − 1 rodadas para enfrentar

todos os jogadores.

Se n for par, como são realizados no máximo n/2 par-

tidas por dia e o torneio tem n(n− 1)/2 partidas, são ne-

cessários no mínimo

n(n− 1)/2

n/2= n− 1

dias para concluir o torneio.

Vamos mostrar que esses valores são de fato, os mí-

nimos, construindo uma tabela com essas quantidades de

dias em cada caso.

Para tal, vamos escrever na casa que está linha i e na

coluna j o dia em que se jogadores se enfrentam no tor-

neio. É claro que não escreveremos nada nas casas do tipo

(i, i) e que não podem aparecer dois números iguais em

uma mesma linha ou coluna.

Antes de passar para o caso geral, observemos dois

casos particulares:

• n = 3.

..

1

.

1

.

2

.

2

.3

.

3

.

0

.

1

.2

.

1

.

2

.3

.

2

.

3

.1

A tabela para n = 4 pode ser obtida a partir de n = 3. O

jogador 4 enfrentará os jogadores na ordem 3× 4, 2× 4 e

1× 4.

• n = 5.

..

1

.

1

.

2

.

2

.

3

.

3

.

4

.

4

.5

.

5

.

0

.

1

.

2

.

3

.4

.

1

.

2

.

3

.

4

.5

.

2

.

3

.

4

.

5

.1

.

3

.

4

.

5

.

1

.2

.

4

.

5

.

1

.

2

.3

Observe que cada jogador descansa um dia.

Para construir uma tabela para n = 6, basta que o

jogador 6 dispute as partidas com os jogadores que des-

cansavam na tabela de n = 5. Assim, a sequência de jogos

deste jogador será: 4× 6, 2× 6, 5× 6, 3× 6 e 1× 6.

No caso geral para n ímpar, colocamos a partida entre

os jogadores i e j no dia i+ j− 2 (mod n), lembrando de

escrever n no lugar de zero. Todas as linhas da tabela te-

rão números distintos. De fato, suponha que os números

que aparecem em (i, j) e (i, k) sejam iguais. Então tere-

mos i+ j− 2 ≡ i+ k− 2 (mod n), donde j ≡ k mod n e

como 1 ⩽ j, k ⩽ n, teremos j = k.

O jogador i descansa no dia 2i − 2 (mod n). Não

é possível que dois jogadores descansem no mesmo dia

porque se 2i − 2 ≡ 2j − 2 (mod n), então 2(i − j) ≡ 0

(mod n), e como n é ímpar, devemos ter i ≡ j (mod n),

donde i = j.

Assim, a cada dia um jogador diferente descansa e

esse jogador é o que enfrentará o jogador n + 1 nessa

rodada para resolver o caso par.

Page 103: compilacaoModulo1Nivel2

DRAFT

..

Soluções

. 23.

Gabarito do Simulado 2

• Simulado aplicado no primeiro encontro presencial no dia 13/01 com duração de 4 horas e 30 minutos.

Problema 1 Calcule a área do triângulo ABC abaixo, dados BD = 4, DE = 2, EC = 6, BF = FC = 3.

..

A

.

B

.

D

.

C

.F .E......

Solução: Como BD/DE = 2 e D é um ponto da mediana AF, então D é baricentro do triângulo ABC e consequen-

temente E é ponto médio do lado CA. Além disso, os triângulos BCE e ABE têm mesma altura e mesma base, ou seja,

mesma área.

Como o triângulo BCE é equilátero de lado 6, então BCE tem área 9√3 e o triângulo ABC tem área 18

√3.

Problema 2 Estão alinhados N gafanhotos (N ⩾ 3). A cada segundo, um dos gafanhotos (exceto os dois primeiros)

pula e passa à frente dos dois imediatamente à sua frente. Após K segundos, os N gafanhotos estão novamente na

ordem original. Quais os possíveis valores de K?

Solução: Se N = 3 em cada segundo apenas um gafanhoto pode pular e assim K tem que ser múltiplo de 3.

Se N > 3 temos as seguintes possibilidades:

(i) 1234 . . .N→ 1342 . . .N→ 3412 . . .N→ 3124 . . .N→ 1234 . . .N

(ii) 1234 . . .N→ 2314 . . .N→ 3124 . . .N→ 1234 . . .N

Assim é possível voltar à posição original em 3 ou 4 jogadas. Deste modo, também em 3a+4b jogadas, combinando-

as (a e b naturais). Em 1 ou 2 jogadas é impossível pois dois gafanhotos pulados na primeira jogada têm que pular o

primeiro e com esse argumento também descartamos K = 5. Como 3 · 2 = 6, 3 · 1+ 4 · 1 = 7, 4 · 2 = 8 e para obter os

demais valores basta ir acrescentando 3, qualquer natural K ⩾ 6 pode ser escrito como 3a+ 4b. Assim, K pode ser 3,

4 ou qualquer natural maior ou igual a 6.

Problema 3 Em Terra Brasilis existem n casas, onde vivem n duendes, cada um em uma casa. Nesse país temos

estradas de mão única satisfazendo as seguintes condições:

(i) Cada estrada une duas casas.

(ii) Em cada casa começa exatamente uma estrada.

(iii) Em cada casa termina exatamente uma estrada.

103

Page 104: compilacaoModulo1Nivel2

DRAFT

104 Avaliações e Simulados– Gabarito do Simulado 2 – Professor Paulo Rodrigues

Todos os dias, a partir do dia 1, cada duende sai da casa onde está e chega à casa vizinha. Uma lenda de Terra Brasilis

diz que, quando todos os duendes regressarem às suas casas originais, o mundo acabará.

(a) Mostre que o mundo acabará.

(b) Se n = 98, mostre que é possível que os duendes construam e orientem as estradas de modo que o mundo não

acabe antes de 300 000 anos.

Solução:

(a) Numere os duendes de 1 a n e seja f(i) o vizinho do duende número i. Fixe 1 ⩽ i ⩽ n. Como a sequência

f(i), f(f(i)), f(f(f(i))), . . .

assume somente um número finito de valores, existirão inteiros positivos r < s tais que

f(s)(i) = f(r)(i).

Desde que a função f é claramente uma bijeção, temos

f(s−r)(i) = i.

Seja agora g(i) o menor inteiro positivo tal que o duende i retorna à sua casa após g(i) dias. Então após

mmc(g(1), g(2), . . . , g(n))

dias todos os duendes retornarão à posição original e o mundo acabará.

(b) Divida os 98 duendes em 8 ciclos de tamanhos 3, 5, 7, 11, 13, 17, 19, 23 (veja que 98 = 3+5+7+11+13+17+19+23).

Da discussão do item (i) é imediato que os duendes só retornarão simultaneamente às suas posições originais após

um número de dias igual a

3 · 5 · 7 · 11 · 13 · 17 · 19 · 23 = 111546435

> 366 · 300 000,

ou seja, após mais de 300 000 anos.

Alternativamente, podemos dividir os duendes em ciclos de tamanhos 3, 8, 9, 5, 7, 11, 13, 19 e 23, e eles retornarão

à posição original após

mmc(3, 8, 9, 5, 7, 11, 13, 19, 23)

dias, ou aproximadamente 431 444 anos.

Page 105: compilacaoModulo1Nivel2

DRAFT

Apêndice A

Utilizando TEX no Fórum

A.1

O que é TEX?

A.1.1

Knuth inventou o TEX...

O problema de escrever matemática no computador

surgiu na década de 70. Um dos primeiros grandes mate-

máticos a pesquisar em Ciência da Computação, Donald

Knuth da Universidade de Stanford, encontrou uma solu-

ção que continua atual mais de trinta anos depois. Por

volta de 1976, Knuth tinha escrito os dois primeiros vo-

lumes da coleção The Art of Computer Programming e

estava totalmente insatisfeito com o resultado impresso.

Ele não queria apenas que o livro fosse impresso, mas

queria algo belo.

Knuth partiu para a busca de uma solução. Um dos primeiros passos foi a interrupção de sua pesquisa por um

ano para, acompanhado por sua esposa, assistir aulas de design com o professor de arte de Stanford, Matthew Kahn.

A ideia era tentar capturar a essência do design, não apenas seu visual. Por exemplo, como um processador deveria

quebrar as linhas em um parágrafo? Esteticamente, o ideal é que não existam espaços excessivos entre as palavras e

que não existam muitos hifens. Knuth transformou esse problema em combinatória e fez um algoritmo que calcula a

maneira ótima de quebrar as linhas em um parágrafo.

Como resultado do seu trabalho, surgiram o processador de textos TEX e o sistema de descrição de fontes META-

FONT, ambos colocados em domínio público. O TEX foi projetado com dois objetivos principais em mente: permitir

que qualquer pessoa possa produzir livros de alta qualidade com um esforço razoável e dar exatamente o mesmo

resultado em todos os computadores, agora e no futuro.

A.1.2

... e Lamport criou o LATEX

No início da década de 80, o matemático Leslie Lamport planejava escrever o livro Great American Concurrency Book

e digitar utilizando TEX. Ele escreveu um conjunto de macros que facilitaram bastante o trabalho. Essas macros foram

105

Page 106: compilacaoModulo1Nivel2

DRAFT

106 Avaliações e Simulados– Gabarito do Simulado 2 – Professor Paulo Rodrigues

posteriormente colocadas em domínio público. Era o início do LATEX. Lamport até hoje não escreveu o livro pretendido,

mas em 1986 lançou o livro LATEX: A Document Preparation System, que ajudou a popularizar o LATEX. De lá para cá,

vários conjuntos de macros para TEX surgiram, como ConTeXt e JadeTeX, mas sem dúvida o LATEX é o mais utilizado.

Hoje o TEX é popular em todo o mundo, principalmente na área acadêmica, notadamente em matemática, física,

ciência da computação e engenharias.

Uma das vantagens do LATEX é a sua modularização. Qualquer um pode escrever um conjunto de macros que

automatizam determinados procedimentos e facilitam a vida de todos. No caso do LATEX, estas macros são chamadas

de pacotes e existem milhares de pacotes escritos por centenas de usuários ao redor do mundo.

A principal desvantagem inicial do TEX é que não é um editor WYSIWYG (acrônimo da expressão em inglês “What

You See Is What You Get” – O que você vê é o que você recebe). Isto significa que digitamos o texto usando uma

linguagem específica, compilamos e depois vemos o resultado. Isto pode parecer muito estranho para quem está

acostumado a editores WYSIWYG, como o Word, mas é uma barreira que pode ser facilmente superada.

• Curiosidades• Knuth até hoje não terminou a coleção The Art of Computer Programming. Dos sete volumes previstos inicialmente, somente

três volumes completos foram lançados, além de cinco fascículos do volume 4.

• Ele recebeu inúmeros prêmios como pesquisador em Ciência da Computação e em agosto de 1999 seu nome foi dado a um

pequeno planeta descoberto por P. Pravec and P. Kusnirák.

• Desde 2001, Lamport é pesquisador da Microsoft.

• Existem versões de TEX para praticamente todos os sistemas operacionais, incluindo Windows, Mac OS X e Linux.

• O código fonte do TEX foi colocado em domínio público, e Knuth recomenda modificações ou experiências com esse código fonte,

mas, para garantir a mesma saída em todas as versões do TEX, ele deseja que qualquer novo programa obtido tenha outro nome.

Para garantir isso, a American Mathematical Society registrou a marca TEX e qualquer implementação do sistema deve passar por

um teste antes de ser chamada de TEX.

• O nome TEX deve ser pronunciado como “tekh”. O X representa a letra grega χ (chi). TEX é uma abreviação de τεχνη (techn), que

também é a origem da palavra técnico.

• Knuth escreveu cinco livros sobre TEX: The TEX book, TEX: The Program, The METAFONT book, METAFONT: The Program e

Computer Modern Typefaces, todos lançados pela Addison-Wesley.

• As versões de TEX são numeradas como aproximações do número π. A versão atual é a 3.141592. Knuth deseja que, após a

sua morte o TEX não seja alterado, com exceção da versão, que deverá ser a π. Analogamente, as versões de METAFONT são

numeradas como aproximações do número e, base dos logaritmos naturais.

• Atualmente, os grupos de usuários TEX (www.tug.org) de diversos países são responsáveis pela distribuição, manutenção e

atualizações nas macros para TEX.

A.2

Escrevendo e desenhando no fórum

O fórum do Programa Oficinas de Formação está associado a uma instalação do programa LATEX, utilizado para digitar

matemática.

Para tal, você deve digitar [tex] comandos [/tex].

Por exemplo, digitando

[ tex ]\ frac 3 8 [ / tex ]

o sistema converterá seu código para uma imagem contendo 38

. A imagem só é exibida na mensagem a ser visualizada

clicando no botão Prever ou no botão Enviar. Sugerimos que você aprenda inicialmente a escrever os exemplos

básicos abaixo, os quais representam mais de 90% da utilização do LATEX no fórum.

Page 107: compilacaoModulo1Nivel2

DRAFT

OBMEP • PECI – Preparação Especial para Competições Internacionais 107

A.2.1

Exemplos Básicos

3+5 ........................ 3+ 5

7-2 ........................ 7− 2

\times ....................... ×3\times2 ................. 3× 2

3\cdot2..................... 3 · 2\frac38 ................ 3

8

3/8 .......................... 3/8

\dfrac38 .............. 3

820\div3 .................. 20÷ 3

\ne ............................ =

\pm ............................ ±10\% ........................ 10%

a_1 ........................... a1

b_23 ...................... b23

x^11 ...................... x11

\sqrt2 ...................√2

\sqrt[3]2 ............... 3√2

< .............................. <

2<3 ........................ 2 < 3

\le ............................ ⩽

a\le b ................... a ⩽ b

\ge ............................ ⩾a\ge b ................... a ⩾ b

(1,2) ...................... (1, 2)

[1,2] ...................... [1, 2]

\ ..............................

\ ..............................

\1,2\ ................... 1, 2

\mid-3\mid ............... | −3 |

Atenção! A melhor maneira de aprender a digitar em LATEX é praticando. Você não precisa

decorar todos os comandos. Consulte-os neste manual, quando precisar. Após alguma

prática, você já conhecerá os principais comandos e com certeza terá prazer em escrever

usando o LATEX.

A.2.2

Letras Gregas

\pi ............................ π

\Pi ............................ Π

\alpha ........................ α

\beta ......................... β

\gamma ........................ γ

\Gamma ........................ Γ

\Delta ........................ ∆

\delta ........................ δ

\epsilon ..................... ϵ

\phi .......................... ϕ

\lambda ....................... λ

\mu ............................ µ

\rho ........................... ρ

\sigma ........................ σ

\Sigma ........................ Σ

\theta ........................ θ

\Omega ....................... Ω

\omega ....................... ω

A.2.3

Aritmética

\equiv ....................................................................................................... ≡\pmodn ............................................................................................. (mod n)

a\equiv b\pmodn ........................................................................... a ≡ b (mod n)

\phi(n) ................................................................................................... ϕ(n)

\lfloor x\rfloor ....................................................................................... ⌊x⌋\lceil x\rceil ......................................................................................... ⌈x⌉

A.2.4

Geometria

\angle ABC ................................. ∠ABC

\measuredangle ABC ...................... ∡ABC

A\hatBC .................................... ABC

\widehatABC ............................... ABC

r\parallel s ................................. r ∥ s

r\perp s ..................................... r ⊥ s

\circ ............................................. 90^\circ .................................... 90

\overlineAB ................................ AB

Page 108: compilacaoModulo1Nivel2

DRAFT

108 Avaliações e Simulados– Gabarito do Simulado 2 – Professor Paulo Rodrigues

\vecv ........................................... v

\arcoAB....................................... AB1

\triangle ABC ............................. ABC

\cong ............................................. ∼=

\triangle ABC \cong \triangle XYZ

ABC ∼= XYZ

\sim ............................................... ∼

\triangle ABC \sim \triangle XYZABC ∼ XYZ

A.2.5

Setas

\iff ......................................... ⇐⇒\Rightarrow ................................... ⇒\Leftarrow .................................... ⇐\Longrightarrow ............................ =⇒\longrightarrow ............................ −→\mapsto ........................................ 7→\rightarrow ................................... →

\searrow ....................................... \downarrow ..................................... ↓\swarrow ....................................... \leftarrow .................................... ←\nwarrow ....................................... \uparrow ........................................ ↑\nearrow .......................................

A.2.6

Símbolos Diversos

\$ .............................. $

\dots............................ . .\ldots......................... . . .

\vdots..............................\cdots......................... · · ·

\ddots..........................\ell..............................ℓ\infty..........................∞

\therefore ................. ∴\approx ...................... ≈\bullet ..................... •\diamond .................... ⋄\Diamond .................... \Box ......................... \heartsuit ................. \spadesuit ................. ♠\diamondsuit ..............

\clubsuit .................. ♣\star........................... ⋆

\bigstar...................... ⋆\square ..................... \blacksquare .............. \TeX ....................... TEX\LaTeX ................... LATEX\S ............................ §

\P ............................ ¶

A.2.7

Conjuntos

\in ......................... ∈\not\in ..................... ∈\subset....................... ⊂\not\subset ................. ⊂\cap .......................... ∩

\cup .......................... ∪\emptyset .................... ∅\mathbbN .................. N\mathbbZ .................. Z\mathbbQ .................. Q

\mathbbR .................. R\mathbbC .................. C\mathcalP(X)........... P(X)

A.2.8

Matrizes e Determinantes

\beginmatrix1 & 2\\ 3 & 4\endmatrix ...................................................... 1 2

3 4

1Comando personalizado para o fórum

Page 109: compilacaoModulo1Nivel2

DRAFT

OBMEP • PECI – Preparação Especial para Competições Internacionais 109

\beginpmatrix1 & 2\\ 3 & 4\endpmatrix ................................................(1 2

3 4

)

\beginbmatrix1 & 2\\ 3 & 4\endbmatrix .................................................[1 2

3 4

]

\beginBmatrix1 & 2\\ 3 & 4\endBmatrix ................................................1 2

3 4

\beginvmatrix1 & 2\\ 3 & 4\endvmatrix ..................................................∣∣∣∣∣1 2

3 4

∣∣∣∣∣\beginVmatrix1 & 2\\ 3 & 4\endVmatrix .................................................

∥∥∥∥∥1 2

3 4

∥∥∥∥∥\beginpmatrix1 \\ 2 \\ 3\endpmatrix ......................................................

1

2

3

\detA .................................................................................................. detA

A.2.9

Somatórios e Produtórios

\sum_i=1^n i^2.................................................................................. ∑ni=1 i

2

\displaystyle\sum_i=1^n i^2...................................................................n∑

i=1

i2

\sum_i=1^100 i(i+1)...................................................................... ∑100i=1 i(i+ 1)

\displaystyle\sum_i=1^100 i(i+1).......................................................100∑i=1

i(i+ 1)

\prod_i=1^10 \fracii+1................................................................ ∏10i=1

ii+1

\displaystyle\prod_i=1^10 \fracii+1...............................................10∏i=1

i

i+ 1

\displaystyle\sum_i\ge 1 \frac1i^2 = \frac\pi^26.......................∑i⩾1

1

i2=

π2

6

A.2.10

Diversos

\begincasesx+y=10\\ x-y=4\endcases................................................

x+ y = 10

x− y = 4

1\overbrace22\dots2^405 ................................................................... 1

40︷ ︸︸ ︷22 . . . 2 5

\underbrace11\dots1_100 ...................................................................... 11 . . . 1︸ ︷︷ ︸100

(\dfracax+bcx+d).............................................................................. (ax+ b

cx+ d)

\left(\dfracax+bcx+d\right)..............................................................(ax+ b

cx+ d

)

Page 110: compilacaoModulo1Nivel2

DRAFT

110 Avaliações e Simulados– Gabarito do Simulado 2 – Professor Paulo Rodrigues

A.2.11

Construindo figuras com LATEX

É possível fazer desenhos dos mais variados tipos usando LATEX. Existem centenas de pacotes para fazer figuras. No

fórum foram instalados os pacotes pstricks e pst-eucl. Os comandos para desenhar figuras devem estar entre as tags

[teximg] e [/teximg].

Apresentamos a seguir alguns exemplos básicos de figuras feitas com pstricks. Em todos os casos, a imagem

mostrada à direita contém, para facilitar a compreensão, uma grade de pontos que não consta no código exibido.

Traçando segmentos de reta

Os comandos abaixo definem uma caixa com extremidades (0,0) e (4,4) e três segmentos de reta que têm por

extremidades os pontos designados. O parâmetro opcional no segundo segmento ([linestyle=dashed]) muda o

estilo da linha para tracejado.

\beginpspicture(0,0)(4,4)\psline(0,1)(1,4)\psline[linestyle=dashed](2,0)(2,4)\psline->(1,0)(4,3)\endpspicture

0 1 2 3 4

0

1

2

3

4

Circunferências

Para construir uma circunferência, devemos informar o centro e o raio com o comando \pscircle(x,y)r, sendo

(x,y) as coordenadas do centro da circunferência e r o raio.

Na segunda circunferência, aumentamos a espessura da linha com o parâmetro [linewidth=2pt].

\beginpspicture(0,0)(4,4)\pscircle(2,2)2\pscircle[linewidth=2pt](3,3)1\endpspicture

0 1 2 3 4

0

1

2

3

4

Rotulando Pontos

O pacote pst-euclides nos permite dar nome aos pontos e depois utilizar os nomes dados para fazer outras cons-

truções. O comando \pstGeonode[PosAngle=α](x,y)Nome marca um ponto de coordenadas (x,y) com o

rótulo Nome. O ângulo do rótulo em relação ao ponto é dado pelo parâmetro opcional PosAngle.

\beginpspicture(0,0)(4,4)\pstGeonode(4,3)A\pstGeonode[PosAngle=90](1,4)B\pstGeonode[PosAngle=180](0,1)C\pstGeonode[PosAngle=-90](3,0)D

\psline(A)(B)(C)(D)(A)\endpspicture

Page 111: compilacaoModulo1Nivel2

DRAFT

OBMEP • PECI – Preparação Especial para Competições Internacionais 111

0 1 2 3 4

0

1

2

3

4

b A

bB

bC

b

D

Utilizando coordenadas polares

É possível utilizar coordenadas polares para definir os pontos. Neste caso, devemos indicar a distância r do ponto à

origem e o ângulo θ formado entre o segmento de reta que une esse ponto à origem e o eixo x.

As coordenadas polares devem ser separadas por ponto e vírgula: (r;θ).

\beginpspicture(-2,-2)(2,2)\psline(2;0)(2;72)\psline(2;72)(2;144)\psline(2;144)(2;216)\psline(2;216)(2;288)\psline(2;288)(2;0)\psline[linestyle=dashed](2;0)(2;144)\pscircle(0,0)2\endpspicture

-2 -1 0 1 2

-2

-1

0

1

2

Page 112: compilacaoModulo1Nivel2

DRAFT

112 Avaliações e Simulados– Gabarito do Simulado 2 – Professor Paulo Rodrigues

Page 113: compilacaoModulo1Nivel2

DRAFT

Apêndice B

Competições e Prêmios

B.1

Torneio Internacional das Cidades

O Torneio das Cidades é uma competição organizada em Moscou desde 1980 e que permite a participação de estudan-

tes de todo o mundo, sem haver deslocamento. As provas elaboradas pelo comitê organizador são enviadas para as

cidades participantes. Após a aplicação, há uma primeira correção das provas e as melhores são traduzidas e envia-

das para Moscou. Lá as provas são corrigidas novamente e são anunciados os ganhadores de diplomas. Existem duas

modalidades: Júnior e Sênior.

2009

1. Tadeu Pires de Matos Belfort Neto – Fortaleza (CE) – Diploma Modalidade Júnior (14,67)

2. Vinícius Canto Costa – Salvador (BA) – Diploma Modalidade Júnior (14,67)

3. Vitor Ramos de Paula – Belo Horizonte (MG) – Diploma Modalidade Júnior (13,33)

4. Marina Pessoa Mota – Fortaleza (CE) – Diploma Modalidade Júnior (14)

5. Mateus Henrigue Ramos de Souza – Pirapora (MG) – Diploma Modalidade Júnior (12)

6. Victor de Oliveira Bitarães – Betim (MG) – Diploma Modalidade Júnior (12)

7. Maria Clara Mendes Silva – Pirajuba (MG) – Diploma Modalidade Sênior (16,25)

B.2

Asian Pacific Mathematical Olympiad

É uma competição de caráter internacional realizada em diversos países asiáticos e da America dedicada a estudantes

do Ensino Médio. No Brasil a olimpíada APMO é aplicada apenas aos alunos que tenham sido premiados na Olimpíada

Brasileira de Matemática (medalhas de ouro, prata, bronze e menções honrosas). As provas dos alunos selecionados

são enviadas para a comissão organizadora no Japão onde é dada a classificação final.

2010

8. Maria Clara Mendes Silva – Pirajuba (MG) – Medalha de Bronze

113

Page 114: compilacaoModulo1Nivel2

DRAFT

114 Avaliações e Simulados– Gabarito do Simulado 2 – Professor Paulo Rodrigues

2011

9. André Macieira Braga Costa – Belo Horizonte (MG) – Medalha de Prata

10. Henrique G. Fiuza do Nascimento – Brasília (DF) – Medalha de Bronze

11. Maria Clara Mendes Silva – Pirajuba (MG) – Medalha de Bronze

12. Victor de Oliveira Bitarães – Betim (MG) – M. Honrosa

2012

13. Maria Clara Mendes Silva – Pirajuba (MG) – Medalha de Prata

14. André Macieira Braga Costa – Belo Horizonte (MG) – Medalha de Bronze

15. Henrique G. Fiúza do Nascimento – Brasília (DF) – Medalha de Bronze

16. Alessandro de Oliveira Pacanowsky – Rio de Janeiro (RJ) – M. Honrosa

B.3

Olimpíada de Matemática do Cone Sul

Competição realizada anualmente entre 8 países sul-americanos. Os 4 estudantes de cada país não podem ter comple-

tado 16 anos até 31 de Dezembro do ano anterior ao da realização da prova. Começou em 1988 e as últimas edições

aconteceram no Brasil (2010), Bolívia (2011) e Peru (2012). Em 2013 acontecerá no Paraguai.

2010 – Brasil

17. Maria Clara Mendes Silva – Pirajuba (MG) – Prata

2011 – Bolívia

18. Henrique Fiuza do Nascimento – Brasília (DF) – Prata

19. Vinícius Canto Costa – Rio de Janeiro (RJ) – Medalha de Prata

2012 – Peru

20. Henrique Gasparini Fiuza do Nascimento – Brasília (DF) – Medalha de Prata

B.4

Olimpíada Iberoamericana de Matemática

Competição entre mais de 20 países Iberoamericanos. Os 4 estudantes de cada país não podem ter completado 18 anos

até 31 de Dezembro do ano anterior ao da realização da prova. Começou em 1985 e as últimas edições aconteceram

no Paraguai (2010), na Costa Rica (2011) e na Bolívia (2012). Em 2013 acontecerá no Panamá.

Page 115: compilacaoModulo1Nivel2

DRAFT

OBMEP • PECI – Preparação Especial para Competições Internacionais 115

2011 – Costa Rica

21. Henrique Gasparini Fiúza do Nascimento – Brasília (DF) – Medalha de Prata

22. Maria Clara Mendes Silva – Pirajuba (MG) – Medalha de Prata

23. André Macieira Braga Costa – Belo Horizonte (MG) – Medalha de Bronze

2012 – Bolívia

24. André Macieira Braga Costa – Belo Horizonte (MG) – Medalha de Prata

B.5

Romanian Masters in Mathematics

Desde 2010 o Brasil participa da Romanian Master in Mathematics, olimpíada que convoca apenas os melhores paí-

ses do mundo em competições internacionais do gênero. A Romanian Master in Mathematics (RMM) é organizada

desde 2007 pela Escola Nacional de Informática Tudor Vianu em colaboração com a Sociedade Científica Romena de

Matemática e o Ministério de Educação Investigação e Juventude.

2011 – Romênia

25. Maria Clara Mendes Silva – Pirajuba (MG) – Menção Honrosa

2012 – Romênia

26. Maria Clara Mendes Silva – Pirajuba (MG) – Medalha de Bronze

27. Henrique Gasparini Fiúza do Nascimento – Brasília (DF) – Menção Honrosa

28. André Macieira Braga Costa – Belo Horizonte (MG) – Menção Honrosa

B.6

Olimpíada de Matemática dos Países de Língua Portuguesa

2012 – Brasil

29. Daniel Santana Rocha – Rio de Janeiro (RJ) – Medalha de Ouro

B.7

Olimpíada Rioplatense de Matemática

Competição realizada anualmente na Argentina em 4 níveis com a participação de cidades ou países convidados. São

Paulo e Fortaleza participam há mais de 15 anos desta competição.

Page 116: compilacaoModulo1Nivel2

DRAFT

116 Avaliações e Simulados– Gabarito do Simulado 2 – Professor Paulo Rodrigues

2011 – Argentina

30. Gabriel Fazoli Domingos – (SP) – Medalha de Prata – Nível 1

2012 – Argentina

31. Pedro Henrique Alencar Costa – Fortaleza (CE) – Medalha de Prata – Nível 1

B.8

Olimpíada Internacional de Matemática

A primeira edição foi realizada em 1959 com a participação de países do Leste Europeu. O Brasil começou a participar

em 1979 e sediará a competição pela primeira vez em 2017. Cada país compete com 6 estudantes que ainda não

tenham ingressado no nível superior. Em 2011 a IMO foi realizada na Holanda, em 2012 na Argentina, em 2013 será

na Colômbia, em 2014 na África do Sul e em 2015 na Tailândia.

2011 – Holanda

32. André Macieira Braga Costa – Belo Horizonte (MG) – Medalha de Prata

33. Henrique Gasparini Fiúza do Nascimento – Brasília (DF) – Medalha de Prata

34. Maria Clara Mendes Silva – Pirajuba (MG) – Medalha de Bronze

2012 – Argentina

35. Henrique Gasparini Fiúza do Nascimento – Brasília (DF) – Medalha de Bronze

36. Maria Clara Mendes Silva – Pirajuba (MG) – Menção Honrosa

Page 117: compilacaoModulo1Nivel2

DRAFT

OBMEP • PECI – Preparação Especial para Competições Internacionais 117

..

Principais Competições de Matemática para estudantes do Ensino Básico.

Olimpíada Brasileira de Matemática(OBM)

É realizada em 3 fases (Junho, Setembro e Outubro) com

a participação de alunos do Fundamental II e do Ensino

Médio, divididos em 3 níveis. As duas primeiras fases

são aplicadas pela própria escola do aluno. A terceira

fase é aplicada em local determinado pelo coordenador

regional. Ser premiado nesta competição é pré-requisito

para participar do processo seletivo para as principais

competições internacionais de matemática.

.

Olimpíada de Matemática do Cone Sul

Competição realizada anualmente entre 8 países

sul-americanos. Os 4 estudantes de cada país não

podem ter completado 16 anos até 31 de Dezembro

do ano anterior ao da realização da prova. Começou

em 1988 e as últimas edições aconteceram no Brasil

(2010), Bolívia (2011) e Peru (2012). Em 2013 acon-

tecerá no Paraguai.

.

Olimpíada Iberoamericana deMatemática

Competição entre mais de 20 países Iberoamerica-nos. Os 4 estudantes de cada país não podem tercompletado 18 anos até 31 de Dezembro do anoanterior ao da realização da prova. Começou em1985 e as últimas edições aconteceram no Paraguai(2010), na Costa Rica (2011) e na Bolívia (2012). Em2013 acontecerá no Panamá.

.

Olimpíada Internacional de Matemática (IMO)

A mais tradicional e respeitada competição internacional de Mate-

mática, cuja primeira edição foi realizada em 1959 com a partici-

pação de países do Leste Europeu. O Brasil começou a participar

em 1979 e sediará a competição pela primeira vez em 2017. Cada

país compete com 6 estudantes que ainda não tenham ingressado

no nível superior. Em 2011 a IMO foi rea1lizada na Holanda, em

2012 na Argentina, em 2013 será na Colômbia, em 2014 na África

do Sul e em 2015 na Tailândia.

.

Outras Competições Internacionais: Olimpíada de Maio (para alunos de até 15 anos) e APMO (para alunos no Ensino Médio)

nas quais os estudantes participam sem precisar se deslocar. Além destas, há também a Olimpíada Rioplatense de Matemática,

da qual participam anualmente na Argentina estudantes paulistas e cearenses premiados em competições regionais e o Torneio

Internacional das Cidades, tradicional competição organizada em Moscou e aplicada em cidades dos cinco continentes.